Health Assessment Final Exam

अब Quizwiz के साथ अपने होमवर्क और परीक्षाओं को एस करें!

1. Define and describe the technique of the 4 physical examination skills: inspection, palpation, percussion, auscultation.

-inspection - close, careful scrutiny of the whole individual then each body system; check symmetry -palpation - applies your sense of touch to check texture, moisture, organ location and size -percussion - tapping a persons skin with short, sharp strokes to assess underlying structures. -auscultation - listening to sounds produced by the body.

12b. Fill in the labels indicated on the following illustrations (axillary nodes).

-interpectoral/rotter -Lateral axillary -Central axillary -subscapular posterior axillary -pectoral anterior axillary -flow to subdiaphragmatic nodes and liver -flow to opposite breast -parasternal internal thoracic -supraclavicular -infraclavicular

12a. Fill in the labels indicated on the following illustrations (adipose tissue).

-lactiferous duct -lactiferous sinus -lobule -lobe -pectoralis major -cooper ligaments -adipose tissue -2nd rib

21b. Fill in the labels indicated on the following illustrations (internal eye).

-superior rectus m. -conjunctiva -cornea -anterior chamber -lens -posterior chamber -ciliary body -macula -optic disc -optic nerve -sclera -choroid -retina -vitreous body

Cardinal features of successful long-term weight loss

1.) Getting regular physical exercise (4-5 times week/30 minutes) 2.) Eating a low-calorie (=1400 - 1500 kcal/day), low-fat (20% to 25% of total calories) diet 3.) monitoring daily food intake (food diary, portion, portion size) and weight.

2. Name and describe the 3 concentric coats of the eyeball.

1.) Outer Fibrous Sclera- tough, protective, white covering. 2.) Middle Vascular Choroid- dark pigmentation, heavily vascular. 3.) Inner Nervous Retina- visual receptive layer.

Collecting Four Types of Data

1.Complete (Total Health) Database 2. Focused or Problem-Centered Database 3.Follow-Up Database 4. Emergency Database

Describe the process of nociception using the four phase of : 1. Transduction 2. Transmission 3. Perception 4. Modulation

1.Transduction: Initially the first phase, occurs when a noxious stimulus in the form of traumatic or chemical injury, burn, incision, or tumor takes place in the periphery 2. Transmission: the 2nd phase, the pain impulse moves from the level of the spinal cord to the brain. 3. Perception: the 3rd phase, signifies the conscious awareness of a painful sensation. 4. Modulation:Last phase, pain message is inhibited through this phase

Bone conduction, BC

Bones vibrate and transmit sound signals directly to the inner ear and CN8.

Mastoid

Bony prominence of the skull located just behind the ear

Limbus

Border between the cornea and sclera.

apex of the heart

Bottom of the heart: tip of the heart pointing down toward the 5th left intercostal space

9. Contrast Bouchard's nodes with Heberden's nodes in osteoarthritis.

Bouchard's nodes are hard, bony outgrowths or gelatinous cysts on the proximal inter-phalangeal joints, the middle joints of fingers or toes. They are a sign of either osteoarthritis or Rheumatoid arthritis. Herberden's nodes are hard or bony swellings that can develop in the distal inter-phalangeal joints, DIP, the joints closest to the end of the fingers and toes. Heberden's are at the end of fingers and the Bouchard and are in the middle.

10. Contrast visceral pain and somatic/parietal pain.

Somatic, parietal pain is from inflammation of the overlying peritoneum: sharp, precisely localized, aggravated by movement. Visceral pain is from an internal organ: dull, general, poorly localized.

7. Define the condition benign prostatic hypertrophy, list the usual symptoms experienced with this condition, and describe the physical characteristics.

Caused by hormonal imbalance that leads to benign proliferation of benign adenomas. Gradually will impede urine output

Malocclusion

Caused from tooth loss leading to drifting teeth. The stress causes: (1) further tooth loss (2) muscle imbalance from mandible/maxilla misalignment leading to muscle spasms, tenderness, and chronic headaches (3) stress on temporomandibular joint leading to osteoarthritis, pain, and inability to fully open mouth.

Nursing Diagnosis

Clinical judgments used to evaluate the response of the whole person to actual or potential health problems

First visit to a health care provide for a checkup

Complete database

You are leading a discussion of the planned activities for the day at an adult living center and state, "We will be having snacks at 9:30 and lunch will be at noon." One of the participants responds in a very monotone manner, "Snacks at 9:30, lunch at noon, snacks at 9:30, lunch at noon." This patient is exhibiting signs of:

Echolalia

9. Differentiate the signs and symptoms of these conditions of adnexal enlargement:

Ectopic pregnancy: Abdominal or pelvic pain, spotting or new-onset bleeding, positive urine pregnancy test. Ovarian cyst: Usually asymptomatic

First Level Priority Problems

Emergent, life threatening problems prioritized by A,B,C,V. A: Airway Problems B:Breathing Problems C: Cardiac/Circulation Problems V: Vital Signs Concerns

Variables that can influence skin color:

Emotional: fear, anger, embarrassment Environment: hot, cold, cigarette smoking Physical: prolonged elevation, dependent position, immobilization, prolonged inactivity

External variables influencing skin color

Emotions: fear, anger embarrassment Environment: hot room, chilly room, cigarette smoking Physical: prolonged elevation, dependent position, immobilization, prolonged inactivity

_______ is exhibiting an accurate understanding of the other person's feelings within a communication context.

Empathy

10. The manubriosternal angle is also called the angle of Louis. Why is it a useful landmark?

The angle of Louis is continuous with the second rib; an important landmark.

10. Fill in the grading scale for assessing the force of an arterial pulse: 0 _____; 1+ _______; 2+_______; 3+_______.

Fill in the grading scale for assessing the force of an arterial pulse: • 0 = absent • 1+ , weak • 2+, normal • 3+, increased, • 4+ full, bounding

10. Define gynecomastia.

Growth of breast tissue in males.

19. Trigeminal:

H. Mastication and sensation of face, scalp, cornea

24. Retraction:

H. Moving a body part backward and parallel to the ground

SKIN Anatomy

Hair shaft Horney cell layer Basal cell layer Melanocyte Sebaceous gland Eccrine sweat gland Appocrine sweat gland Blood vessels Nerve Adipose tissue Epidermis Dermis Subcutaneous tissue Connective tissue Arrector pili muscle Hair follicle

Spinal cord

Long, cylindric structure of nervous tissue; occupies upper two-thirds of the vertebral canal from the medulla to lumbar vertebrae L1-L2. Its white matter is bundles of myelinated axons that form main highway for ascending and descending fiber tracts that connect brain to spinal nerves; it mediates reflexes of posture control, urination, pain response; its nerve cell bodies, or gray matter, are arranged in a butterfly shape with anterior and posterior "horns".

16. Adduction:

I. Moving a limb toward the midline of the body.

23. Glossopharyngeal:

I. Phonation, swallowing, tasting posterior third of tongue.

4 Stages of Pressure Ulcers:

I: red, unbroken skin, does not blanch II: partial thickness skin erosion, loss of dermis / epidermis III: full thickness skin erosion, extends into subcutaneous tissue, resembles crater IV: full thickness skin erosion involving all skin layers, exposes muscle, tendon, bone - eschar or sloughing may be present

Differences of skin at various developmental stages:

INFANTS: the immature skin cannot effectively prevent fluid loss o regulate temperature PREGNANCY: causes pigment changes and stretch marks AGING: causes changes in the stratum corneum that give chemicals easier access to the body, and causes other changes that lead to wrinkling

Age related macular degeneration, AMD

Loss of central vision caused by yellow deposits, drusen, and neovascularity in the macula; Women at slightly higher risk.

Arterioscerosis cause

Loss of elasticity of the walls of blood vessels

Depersonalization

Loss of identity

5. Describe the environmental conditions to consider in preparing the examination setting.

The area should be warm, comfortable, quiet, private, and well lit. Be sure that all equipment is within easy reach and laid out in an organized fashion.

Fetal circulation

Oxygenation happens from the placenta at week 3 of gestation; arterial blood returned to the right side of the heart; oxygenated blood shunted through the foremen alveoli to the left side of the heart-pumped out through the aorta. Remaining oxygenated blood goes through the right side of the heart via the pulmonary artery with a detour through the ductus arterious back to the aorta. Both sides pump into the systemic circulation. At birth the lungs are inflated and aeration begins and oxygenated blood starts to flow through the lungs and not the placenta.

Seborrhea

OILY: an inflammatory skin disorder affecting the scalp, face, and torso. Typically, seborrheic dermatitis presents with scaly, flaky, itchy, and red skin. It particularly affects the sebaceous-gland-rich areas of skin. In adolescents and adults, seborrhoeic dermatitis usually presents as scalp scaling similar to dandruff or as mild to marked erythema of the nasolabial fold

6. List 4 conditions that may alter normal percussion notes heard over the abdomen.

Obesity, Air or Gas, Ascites, large Ovarian Cyst

1b. What abnormal conditions may affect the location of the apical impulse?

Obesity, thick chest walls, anxiety, fever, hyperthyroidism and anemia might increase amplitude and duration.

Annulus

Outer fibrous rim encircling the eardrum

Extropia

Outward turning of the eyes

Bulbar conjunctiva

Overlays the eyeball, with the white sclera showing through.

1. Describe the structure and function of arteries and veins.

The heart pumps freshly oxygenated blood through the arteries to all body tissues. Artery walls are strong, tough, and tense to withstand pressure demands; contain elastic fibers, which allow their walls to stretch with systole and recoil with diastole. Veins are often closer to the skin and contain valves to help keep blood flowing toward the heart, while arteries carry blood away from the heart. Most veins carry deoxygenated blood from the tissues back to the heart; exceptions are the pulmonary and umbilical veins that carry oxygenated blood to the heart. Arteries are more muscular than veins.

Systole

The heart's contracting and pumping phase; 1/3 the heart cycle AV valves shut making the S1, first heart sound heard at the base or bottom of the heart.

Heart failure

The hearts ability to pump enough blood such as low 02 stats. Kidney's will try to compensate with reduced urine output. The heart doesn't pump well so blood begins to back up. S O B on exertion is an early sign.

Illness

The imbalance of the person, both within one's being (physical, mental, and/or spiritual) and in the outside world (natural, communal, and/or metaphysical)

6. Describe the Ortolani maneuver for checking an infants hips.

The infant lays supine, flex the knees holding your thumbs on the inner mid thighs and your fingers outside on the hips touching the greater trochanters. Adduct the legs until your thumbs touch. Then gently lift abduct, moving the knees apart and down so their lateral aspects touch the table.

Eardrum: Blue or dark red color

Indicates blood behind the drum. Could be trauma or skull fracture.

Eardrum: Black or white dots on drum or canal

Indicates bulging of eardrum. Could be acute otitis media.

Eardrum: Black or white dots on drum or canal

Indicates colony of growth. Could be a fungal infection.

Eardrum: Bright red color

Indicates infection in middle ear. Could be acute otitis media.

Eardrum: Dark, round or oval areas

Indicates perforation. Could be drum rupture.

Eardrum: Prominent landmarks

Indicates retraction of drum. Could be a vacuum in the middle ear from an obstructed Eustachian tube.

Eardrum: White dense areas

Indicates scarring. Could be sequelae of infections.

Eardrum: Air/fluid level or air bubbles

Indicates serous fluid. Could it be otitis media with effusion

Eardrum: Yellow-amber color

Indicates serum or pus. Could be: Otitis media with effusion or chronic otitis media.

Eardrum: Diminished or absent landmarks

Indicates thickened eardrum. Could be chronic otitis media.

List 10 traps of interviewing

Providing false assurance Giving unwanted advice Using authority Using avoidance language Keeping distance Using professional jargon Asking biased questions Talking too much Interrupting the patient Asking "why" questions

Common variations on aging adult's skin:

LENTIGINES: liver spots SEBORRHEIC KERATOSIS: dark, greasy, "stuck on" - trunk, face, hands ACTINIC KERATOSIS" scaly, silver-white plaques - pre-malignant ACROCHORDONS: Skin tags, over growth of normal skin SEBACEOUS HYPERPLASIA: raised yellow papules w/central depression (mostly men), over forehead, nose, cheeks, pebbly look

Flat Affect

Lack of Emotional Response

9b. Fill in the labels indicated on the following illustrations (knee).

Lateral epicondyle of femur Lateral collateral ligament Lateral meniscus Lateral condyle of tibia fibula Tibial tuberosity Anterior cruciate ligament, joint opened to show detail Medial condyle of tibia Patella ligament Patella Quadriceps muscle

18. C. 6th rib, midclavicular line

Lateral left

19. D. 5th intercostal

Lateral right

Folk Healer

Lay healer in the person's culture apart from the biomedical/scientific health care system

Describe how you would validate the following data. When auscultating the heart, you hear a blowing, swooshing sound between the first and second heart sounds.

Listen again for sound, if heard ask Physician for second opinion

Auscultation of the Carotid artery

Listen for bruit with the bell while the patient holds their breath on exhalation; don't press too hard so the patient doesn't pass out and you don't create false bruit. 1) angle of the jaw, 2) midcervicular area, 3) base of the neck.

13a. Fill in the labels indicated on the following illustrations (abdominal organs).

Liver gallbladder ascending colon cecum appendix bladder pubic symphysis sigmoid colon descending colon descending colon small intestines stomach spleen

7.Name the organs that are normally palpable in the abdomen.

Liver, Spleen, Kidneys, Aorta

Right upper quadrant tenderness pathology

Liver, pancreas, or ascending colon.

9. List and describe the characteristics to consider when a mass is noted in the breast.

Location: clock face Size: width x length x thickness Shape: oval, round, lobulated, or indistinct Consistency: soft, firm, hard Movable: freely or fixed Distinctness: solitary or multiple Nipple: displaced or retracted Note skin over lump: erythematous, dimpled, or retracted Tenderness: tender on palpation Lymphadenopathy: regional lymph nodes palpable?

During the evaluation phase of the nursing process, what action would be included

Providing information to the patient and family members

You are the triage nurse in the emergency department and perform the initial intake assessment on a patient who does not speak English. Based on your understanding of linguistic competence, what action would present as a barrier to effective communication

Providing the patient with a paper and pencil so he or she can write down the answers to the questions that you are going to ask

Facial characteristics of hypothyroidism

Puffy face around the eyes dry skin dry course hair dry facial hair

Chancre

Red, round, superficial ulcer with a yellowish serous discharge that is a sign of syphilis

19c. Fill in the labels indicated on the following illustrations (neck vessels).

Right external jugular vein right common carotid artery sternomastoid muscle superior vena cava aorta left internal jugular vein left common carotid artery left external jugular vein

Semilunar valves, SL

Right pulmonic. Left aortic. Close for hearts filling phase. Open for the hearts pumping phase.

Atrioventricular valves, AV

Right tricuspid. Left bicuspid or mitral. Open for hearts filling phase. Close for the hearts pumping phase.

Tinnitus

Ringing in the ears

12. Screening mammography starts at age 40, monthly breast self-examination (BSE), and regular clinical breast examination, CBE.

Risk factors are as follows: Female gender, increasing age, family history, personal history first degree, biopsy proven atypical, prolonged estrogen effect aka early menarche, late menopause, nulliparity, late first pregnancy.

6. Describe the lacrimal system.

The lachrymal apparatus continuously washes the eye. It irrigates and keeps the conjunctiva and sclera clean. Tears wash across the eye and drain into the lachrymal puncta. That then drains into the nasal duct.

duration

The length of time a sound/note lingers

4. Relate the anatomic differences that place the infant at greater risk of middle ear infections.

The low set Eustachian tube is angled upwards, shorted, wider and more horizontal than adults. This does not allow fluids to drain easily.

13. List the major risk factors for heart disease and stroke as identified in this text.

The major risk factors for heart disease and stroke are hypertension, smoking, high cholesterol, obesity, and diabetes. Physical inactivity, family hist of heart disease, and age are other risk factors.

scoliosis

S-Shaped curvature of the thoracic spine

5. Describe the characteristics of the 2nd heart sound and its intensity at the apex of the heart and at the base.

S2- "Dub"- is the loudest at the base.

4th heart sound, S4

S4 gallop, atrial gallop; very soft, low-pitched ventricular filling sound that occurs in late diastole; best heard at the apex, apical pulse area with patient laying on left side with timing at the end of the exhale.

stage II hypertension

S: more than 160 D: more than 100

Pacemaker of the heart

SA node, with it's own electrical impulses, read at the surface of the heart with an EKG.

Difference between sebaceous, eccrine, and apocrine glands:

SEBACEOUS: Oil glands - secrete protective lipid substances. Located everywhere except palms and soles. Mostly in scalp, forehead, face & chin. ECCRINE: sweat glands - dilute saline solution. APOCRINE: sweat glands - thick, milky secretion, opens to hair follicles (axillae, anogenital area, nipple, nasal

The most reliable indicator of pain in the adult is: A. The degree of physical functioning B. Nonverbal behaviors C. The MRI findings D. The patient's self-reporting

The patient's self-report

Describe ethnocentrism

The tendency to view your own way of life as the most desirable

6. Explain the positioning of normal ear alignment in the child.

The top of the pinna should match an imaginary line extending from the corner of the eye to the occiput. The ear should also be positioned within 10 degrees of vertical.

Environment

The total of all collections and elements that make up all the surrounding and influence the development of a person.

Avoidance language

The use of euphemisms to avoid reality or to hide feelings

Distancing

The use of impersonal speech to put space between the self and the threat

Senile purpura

The vascular fragility increases in the aging adult, where a minor trauma may produce dark red discolored areas on the skin

What statement is an example of a patient with dysarthria?

The word choice and grammar are appropriate, but the sounds are distorted so speech is unintelligible.

normal abdominal sounds

There are a wide range of normal bowel sounds and are high pitched, gurgling, cascading sounds occurring irregularly anywhere from 5 to 30 times per minute.

What would you say to a colleague who remarks that the individual with Alzheimer disease does not feel pain and therefore does not require an analgesic?

There are no studies that show older adults or people with diminished cognitive abilities have less pain.

2. Describe the anatomy of the breast.?

They are located between the second and 6th rib. Areolas surround nipples with sebaceous glands Glandular Tissue Fibrous Tissue, suspensory ligaments, Adipose tissue Broken up into four quadrants. The upper outer quadrant is the site of most breast tumors.

How would you asses for pain in an individual with dementia?

They communicate pain through their behavior. Such as: Agitation, pacing, repetitive yelling, cry, hit, kick, or moan

Pars Tensa

Thick, taut, central-inferior section of tympanic membrane

Organ or Corti

Sensory organ of hearing

What is a priority assessment for aging adults?

Sensory perceptive abilities

ascites

Shifting dullness is a test for:

S O B

Shortness of breath

Sensorineural hearing loss

Signifies pathology of the inner ear, CN8 or the auditory areas of the cerebral cortex: impacted earwax, foreign bodies, stabbed eardrum, middle ear puss, otoclerosis.

Because of adolescents' developmental level, not all interviewing techniques can be used with them. Which techniques should be avoided?

Silence and reflection

Critical Thinking

Simultaneously problem-solving while self-improving one's own thinking-ability.

Diastolic murmur

Soft, high pitch, early aortic regurgitation as the person sits and leans forward.

3. List a few examples of high-fiber foods of the soluble type and the insoluble type. What advantages do these foods have for the body?

Soluble: vegetables, fruits, legumes Insoluble: seeds and skins of fruits and vegies Advantages: normalize BMs, maintain bowel health, decrease cholesterol levels, help control blood sugar levels, aids in achieving healthy weight

9. Describe the tympanic membrane and temporal artery thermometers, and compare their use with other forms of temperature measurements.

Tympanic membrane and temporal artery thermometer are fast and comfortable. Oral and rectal thermometers are unsafe.

8. What changes in height and weight distribution would you expect from an adult in his or her 70s or 80s.

Weight-Body weight decreases. Muscles shrinkage. Sucutaneous fat is lost from face and periphery. Additional fat deposited on abdomen and hips. Height-Shorter. Shorter trunk with longer extremities

The Primary counselor tells you Ellen is depressed and angry about being admitted to residential treatment in the clinic. How do you evaluate this

Validate the counselors statement w/ Ellen use open minded approach.

During the physical examination, your patient is diaphoretic and pale and complains of dull pain in the LUQ of the abdomen. This is what type of pain? A. Cutaneous Pain B. Somatic Pain C. Visceral Pain D. Psychogenic Pain

Visceral Pain

Objective Data

What a health professional observes by inspecting, percussing, palpating, and auscultating during a physical exam.

Describe how you would validate the following data. Mr. Quinn tells you his weight this morning on the clinic scale was 165 lbs.

Weight him and check his chart

What would be an appropriate statement to use when initiating an assessment of cultural beliefs with an older American Indian patient?

What cultural or spiritual beliefs are important to you?

Pallor

absence of red-pink tones from the oxygenated hemoglobin in blood, excessively pale, whitish pink color to lightly pigmented skin

describe three factors related to socialization

acculturation (adapting to new culture); assimilation (becomes member of dominant culture); biculturalism (divided loyalty)

nociceptive pain is described as

aching if localized and cramping if poorly localized; from somatic sites it is described as throbbing/aching

delirium -

an acute confusional change or loss of consciousness and perceptual disturbance that may accompany acute illness; usually resolves when the underlying cause is treated

Chloasma

an irregular brown patch of hyperpigmentation on the face that may occur with pregnancy

Religion is best described as:

an organized system of beliefs concerning the cause, nature, and purpose of the universe

1. State the length of the anal canal and the rectum in the adult, and describe the location of these structures in the lower abdomen.

anal canal- 3.8 cm; outlet of the GI tract rectum- 12 cm; distal portion of the large intestine

costovertebral angle, CVA

angle formed by the 12th rib and the vertebral column on the posterior thorax, overlying the kidney

Irritability

annoyed easily provoked

Prevention

any action directed toward promoting health and preventing the occurrence of disease.

Prevention

any action directed toward promotion health and preventing the occurrence of disease

Anxiety

apprehensive from the anticipation of a danger whose source is unknown

Pediatric scales, such as the Oucher Scale and the Faces Pain Rating Scale

are used for young children

precordium

area of the chest wall overlying the heart and great vessels

decorticate rigidity

arms adducted and flexed, wrists and fingers flexed; legs extended, internally rotated, plantar-flexed.

decerebrate rigidity

arms stiffly extended, adducted, internally rotated; legs stiffly extended, plantar-flexed.

clubbing

bulbous enlargement of distal phalanges of fingers and toes that occurs with chronic cyanotic heart and lung conditions

neuropathic pain

burning, shooting, and tingling

confusing artifacts in alscultation

by making sure the room is quiet and warm; no listening through clothing

cecum

first or proximal part of large intestine

transduction

first phase of nociception whereby the painful stimulus is changed into an action potential

diaphram of the stethoscope

flat end-piece of the stethoscope used for hearing relatively high-pitched heart sounds

Macule

flat skin lesion with only a color change - freckles

dupuytren contracture

flexion contracture of the fingers due to chronic hyperplasia of the palmar fascia.

phimosis

foreskin is covering the head and stuck; fully covering.

paraphimosis

foreskin pulled back and stuck behind the penis glans; partially covering.

prepuce

foreskin; the hood or flap of skin over the penis glans that often is surgically removed with circumcision.

uvula

free projection hanging down from the middle of the soft palate

Alveoli

functional units of the lung; the thin-walled chambers surrounded by networks of capillaries that are the site of respiratory exchange of carbon dioxide and oxygen

15. Describe abnormal findings of tissue color that are possible on conjunctiva and sclera, and describe their significance.

general reddening, cyanosis of the lower lids, yellowing of the sclera

Hyperthermia

generalized hyperthermia occurs with increased metabolic rate, causing warm, moist skin, such as with fever, or after heavy exercise. Localized hyperthermia occurs with trauma, infection or sunburn.

Hypothermia

generalized hypothermia accompanies central circulatory problems, such as shock. Localized hypothermia occurs in peripheral arterial insufficiency

19a. Fill in the labels indicated on the following illustrations (heart).

internal jugular veins superior vena cava right atrial appendage right atrium right ventricle inferior vena cava thoracic aorta apex left ventricle left atrial appendage pulmonary artery aortic arch common carotid arteries

viscera

internal organs

pagent disease

intraductal carcinoma in the breast

The expert nurse differs from the novice nurse by acting without consciously thinking about the actions. This is referred to as:

intuition

Tremor

involuntary contraction of opposing muscle groups resulting in rhythmic movement of one or more joints

nystagmus

involuntary, rapid, rhythmic movement of the eyeball

complementary interventions

herbal remedies, reflexology, iridology, imagery, distraction, hypnosis, meditation, relaxation techniques, biofeedback, music, therapeutic touch, and acupressure/puncture

Diurnal rhythm-

high in late afternoon and declines to an early morning low

tympany

high-pitched, musical, drumlike percussion note heard when percussing over the stomach and intestine

Wheeze

high-pitched, musical, squeaking adventitious lung sound

4. Hyperactive bowel sound

high-pitched, rushing, tinkling

pericardial friction rub

high-pitched, scratchy extracardiac sound heard when the precordium is inflamed

hypercapnia

hypercarbia; increased levels of carbon dioxide in the blood

Keloid

hypertrophic scar, elevated beyond site of original injury

midclavicular line, MCL

imaginary vertical line bisecting the middle of the clavicle in each hemithorax

ankylosis

immobility, consolidation, and fixation of a joint because of disease, injury, or surgery; most often due to chronic rheumatoid arthritis.

Dysphagia

impairment in speech consisting of lack of coordination and inability to arrange words in their proper order.

paraplegia

impairment or loss of motor and/or sensory function in the lower half of the body

dysarthria

imperfect articulation of speech due to problems of muscular control resulting from central or peripheral nervous system damage.

diastole

in hearts filling phase

characteristic sign of varicose veins

in lower extremities; dilated, tortuous superficial bluish vessels.

Recommended dietary allowance (RDA)

levels of intake of essential nutrients considered to be adequate to meet the nutritional needs of almost all healthy persons

inguinal ligament

ligament extending for pubic bone to anterior superior iliac spine, forming lower border of abdomen

striae

lineae albicantes; silvery white or pink scar tissue formed by stretching of abdominal skin as with pregnancy or obesity

Fissure

linear crack in skin extending into dermis

Zosteriform

linear shape of skin lesion along a nerve route

descriptor scale

list words that describe different levels of pain intensity, such as no pain, mild pain, moderate pain, and severe pain

8 characteristics to describe present health

location - precise site of pain character/quality - description ex: burning, stabbing, throbbing quantity or severity - quantify symptoms, quantify pain (1-10) timing - onset, duration, frequency setting - triggers aggravating and relieving factors associated factors - symptoms that accompany the primary patient's perception - how pt feels about issue

agraphia

loss of ability to express thoughts in writing

apraxia

loss of ability to perform purposeful movements in the absence of sensory or motor damage such as the inability to use objects correctly.

agnosia

loss of ability to recognize importance of sensory impressions

anorexia

loss of appetite for food

bitemporal hemianopsia

loss of both temporal visual fields

Rage

loss of control

amensia

loss of memory

hemiplegia

loss of motor power, paralysis, on one side of the body, usually caused by a stroke; paralysis occurs on side opposite the lesion.

flaccidity

loss of muscle tone, limp

analogesia

loss of pain sensation

aphasia

loss of power of expression by speech, writing, or signs, or loss of comprehension of spoken or written language.

borborygmi

loud, gurgling bowel sounds signaling increased motility of hyperperistalsis; occurs with early bowel obstruction, gastroenteritis, diarrhea

hyperactive abdominal sounds

loud, high pitched rushing, tinkling sounds that signal increased motility

S2

loudest at the base of the heart.

Rhonchi

low-pitched, musical, snoring, adventitious lung sounds caused by airflow obstruction from secretions

costal margin

lower border of rub margin formed by the medial edges of the 8th, 9th and 10th ribs

ectropion

lower eyelid loose and rolling outward

entropion

lower eyelid rolling inward

Harlequin

lower half of body turns red, upper half blanches

Gynoid obesity

majority of fat is in the hips and thighs

angle of Louis

manubriosternal angle, the articulation of manubrium and body of sternum, continous with the second rib

Malnutrition

may mean any nutrition disorder but usually refers to long-term nutritional inadequacies or excesses

Psychiatric Mental Disorders -

in which an organic etiology has not yet been established [e.g., anxiety disorder or schizophrenia]

ataxia

inability to perform coordinated movements.

Protein-calorie malnutrition (PCM)

inadequate consumption of protein and energy, resulting in a gradual body wasting and increased susceptibility to infection

Epidermal appendages

include hair, sebaceous (oil) glands, sweat glands, and nails

Jaundice

increase in bilirubin in the blood causing a yellow color in the skin, palate, and sclera. Dark skinned have yellow outer sclera.

left ventricular hypertrophy, LVH

increase in thickness of myocardial wall that occurs when the heart pumps against chronic outflow obstruction such as aortic stenosis

Linea nigra

increased pigment along the midline of the abdomen - seen with pregnant women.

hyperventilation

increased rate and dept of breathing

pitting edema

indentation left after examiner depresses the skin over swollen edematous tissue

crypts

indentations on surface tonsils

positive Blumberg sign

indicates peritoneal inflammation

List the barriers to evidence-based practice, both on an individual level and on an organizational level.

individual level - nurses lack research skills, isolated from individuals with an expertise in research, and lack confidence to implement change. organizational level - nurses lack time to read and research, healthcare institutions have inadequate library research holdings, and organizational support decreases when nurses wish to implement change in patient care.

conjuctivitis

infection of the conjunctiva, "pinkeye"

chalazion

infection or retention cyst of a meibomian gland, showing as a beady nodule on the eyelid

13b. Fill in the labels indicated on the following illustrations (abdominal structures).

inferior vena cava right kidney duodenum pancreas right ureter sacral promontory external iliac artery external iliac arter uterus bladder pubic symphysis ovary rectum peritoneum common iliac vein common iliac artery left ureter small intestine left kidney aorta

Salpingitis

inflammation of fallopian tubes

blepharitis

inflammation of he glands and eyelash folicules along the margin of the eyelids

peritonitis

inflammation of peritoneum

mastitis

inflammation of the breast

cholecystitis

inflammation of the gallbladder

pharyngitis

inflammation of the throat

cystitis

inflammation of the urinary bladder

thrombophlebitis

inflammation of the vein associated with thrombus formation

Vaginitus

inflammation of vagina

Erythema

intense redness of the skin due to excess blood in the dilated superficial capillaries, as in fever or inflammation. Dark skinned can not see erythema, so must palpate skin for warmth, edema, hardening

Anthropometry

measurement of the body (e.g. height, weight, circumferences, skinfold thickness)

skinfold thickness

measures estimate the body fat stores or the extent of obesity or undernutrition

Hymen

membranous fold of issue partly closing vaginal orifice

turbinate

one of 3 bony projections into nasal cavity filled with blood vessels and mucus membranes to warm, humidify and filter inhaled air. Superior, middle and inferior turbinates.

bronchiole

one of the smaller respiratory passageways into which the segmental bronchi divide

Focused database

one used for a limited problem that concerns one body system

Menarche

onset of first menstruation, usually between 11 and 13 yrs of age

cataract

opacity of the lens that develops slowly with aging, gradually obstructs vision

ulcer

open skin lesion extending into dermis, with sloughing of necrotic inflammatory tissue

paradoxical splitting

opposite of a normal split S2 so that the split is heard in expiration, and in inspiration the sounds fuse to one sound

thrush

oral candidiasis in the newborn

peau d'orange

orange peel appearance of breast due to edema

Spleen, tonsils, and thymus

organs that aid the lymphatic system

visceral pain

originating from interior organs such as the gallbladder or stomach

somatic pain

originating from muscle, bone, joints, tendons, or blood vessels

What conditions are more likely to produce pain in the aging adult?

osteoarthritis, peripheral vascular disease, cancer, osteoporosis, angina, chronic constipation

3. Differentiate among the types of hearing loss and give examples.

otosclerosis - hardening causing foot plate of stapes to become fixed. prebycusis - gradual sensorineural loss caused by nerve degeneration in the inner ear.

7. Define otosclerosis and presbycusis.

otosclerosis- hardening causing foot plate of stapes to become fixed. prebycusis - gradual sensorineural loss caused by nerve degeneration in inner ear.

kyphosis

outward or convex curve of the thoracic spine; hunchback

chronic (persistent) pain

pain continues for 6 months or longer after initial injury

referred pain

pain felt at a particular site but originates from another location

mastalgia

pain in breast

What is an example of a subjective observation?

pain lasting 2 hours

modulation

pain message is inhibited during this last phase of nociception

cutaneous pain

pain originating from skin surface or subcutaneous structures

breakthrough pain

pain restarts or escalates b4 next scheduled analgesic dose

Dyspareunia

painful intercourse

Fissure

painful longitudinal tear in tissue, e.g. in the superficial mucosa at the anal margin.

parotid glands

pair of salivary glands in the cheeks in front of the ears

optic atrophy

pallor of the optic disc due to partial or complete death of optic nerve

thrill

palpable vibration on the chest wall accompanying severe heart mumur; feels like the throat of a purring cat - signifies turbulent bloodflow.

Skene glands

para-urethral glands

Nutritional screening

parameters used for nutritional screening typically include weight and weight history, conditions associated with increased nutritional risk, diet information, and routine laboratory data.

dead space

passageways that transport air but are not available for gaseous exchange; trachea, bronchi

galactorrhea

persistent white discharge of milk between nursing sessions or after weaning

buccal

pertaining to the cheek

atherosclerosis

plaques of fatty deposits formed in the inner layer, intima, of the arteries

apical pulse

point of maximal impulse aka PMI; pulsation created as the left ventricle rotates against the chest wall during systole, normally at the 5th left intercostal space in the mid-clavicular line.

Abstract reasoning -

pondering a deeper meaning beyond the concrete and literal

equilibrium

postural balance of body

Wheal

raised red skin lesion due to interstitial fluid

crackles

rales: abnormal, discontinuous, adventitous lung sounds heard on inspiration

Emergency database

rapid collection of the database, often compiled concurrently with lifesaving measures

fasciculation

rapid continuous twitching of resting muscle without movement of limbs

tachycardia

rapid heart rate, greater than 95 beats per minute in the adult

tachycardia

rapid heart rate, more than 95 beats per minute in the adult

Lability

rapid shift of emotions

myoclonus

rapid sudden jerk of a muscle

Tachypnea

rapid, shallow breathing; more than 24 breaths per minute

clonus

rapidly alternating involuntary contraction and relaxation of a muscle in response to sudden stretch

cup-to-disc ratio

ratio of the width of the physiologic cup to the width of the optic disc, normally less than half

11. Describe rebound tenderness.

rebound tenderness -abdominal pain -Choose site away from painful area -Hold hand at 90 degrees and push down slowly and deeply -Lift hand up quickly - A normal response is no pain.

Chancre

red, round, superficial ulcer with a yellowish serous discharge that is a sign of syphilis.

cheilitis

red, scaling, shallow, painful fissures at corners of mouth

gingivitis

red, swollen gum margins that bleed easily

Purpura

red-purple skin lesion due to blood in tissues from breaks in blood vessels

Hematuria

red-tinged or bloody urine

Vesicular

refers to soft, low-pitched, normal breath sounds heard over peripheral lung fields

plantar

refers to the surface of the sole of the foot

astigmatism

refractive error of vision due to differences in curvature of the cornea and lens

Tic

repetitive twitching of a muscle group at inappropriate times (e.g., wink, grimace)

You have reason to question the reliability of the information being provided by a patient. One way to verify the reliability within the context of the interview is to:

rephrase the questions later in the interview

Ulcer

sloughing of necrotic inflammatory tissue that causes a deep depression in skin, extending into dermis

bradypnea

slow breathing, fewer than 10 breaths per minute, regular rate

bradycardia

slow heart rate, less than 50 beats per minute in the adult

Lethargic -

sluggish and apathetic

Cherry (senile) angiomas

small 1mm to 5 mm, smooth, slightly raised bright red dots that commonly appear on the trunk in all adults older than 30 years. They normally increase in size and number with aging.

lymph nodes

small oval clumps of lymphatic tissue located at grouped intervals along lymphatic vessels

kiplik spots

small, blue-white spots with red halo over oral mucosa; early sign of measles

Caruncle

small, deep red mass protruding from urethral meatus, usually due to urethritis

Clitoris

small, elongated erectile tissue in the female, located at anterior juncture of labia minora

Freckles

small, flat macules of brown melanin pigment that occur on sun-exposed skin

fordyce granules

small, isolated, white or yellow papules on oral mucosa

Alveoli

smallest structures in the mammary gland that make milk

polyp

smooth, pale mgray nodules in the nasal cavity due to chronic allergic rhinitis

3rd heart sound, S3

soft, low-pitched ventricular filling sound that occurs in early diastole/S3 gallop, and may be an early sign of failure; best heard at the apex, apical pulse area with patient laying on left side with timing at the end of the exhale.

Condylomata acuminata

soft, pointed, fleshy papules that occur on the genitalia and are caused by the human papillomavirus, HPV

xanthelasma

soft, raised yellow plaques occurring on the skin at the inner corners of the eyes.

plaque

soft, whitish debris on teeth

Hegar sign

softening of cervix that is a sign of pregnancy, occuring at 10 to 12 weeks gestation

Maceration

softening of tissue by soaking

Papule

solid, elevated, circumscribed skin lesion, palpable, <1 cm diameter

Nodule

solid, elevated, hard or soft skin lesion, >1 cm diameter

intercostal space

space between ribs

nociceptors

specialized nerve endings that detect painful sensations

A patient says she is very nervous and nauseated, and she feels like she will vomit. This data would be what type of data?

subjective

Complaint of sore shoulder

subjective

Dizziness

subjective

Family has just moved to a new area

subjective

earache

subjective

sore throat

subjective

identify the most reliable indicator of a persons pain

subjective report

paroxysmal nocturnal dyspnea

sudden awakening from sleeping, with shortness of breath

Torsion

sudden twisting of spermatic cord; a surgical emergency

chorea

sudden, rapid, jerky, purposeless movement involving limbs, trunk, or face.

19b. Fill in the labels indicated on the following illustrations (inner heart structures).

superior vena cava pulmonary artery pulmonary veins pulmonic valve right atrium tricuspid AV valve Inferior vena cava right ventricle myocardium endocardium papillary muscle left ventricle chordae tendineae mitral AV valve aortic valve left atrium pulmonary veins cut edge of pericardium aortic arch

cooper ligaments

suspensory ligaments; fibrous bands extending from the inner breast surface to the chest wall muscles; breast support

lymphedema

swelling of extremity due to obstructed lymph channel, nonpitting

edema

swelling of the legs or dependent body part due to increased interstitial fluid

Xiphoid process

sword-shaped lower tip of the sternum

Iris

target shape of skin lesion

inching

technique of moving the stethoscope incrementally across the precordium through the auscultatory areas while listening to the heart sounds

syncope

temporary loss of consciousness due to decreased cerebral blood flow/fainting; caused by ventricular asystole, pronounced bradycardia, or ventricular fibrillation

allen test

test that determines the patency of the radial and ulnar arteries by compressing one artery site and observing return of skin color as evidence of patency of the outer artery

Gallop rhythm

the addition of a 3rd or a 4th heart sound; makes the rhythm sound like the cadence of a galloping horse

What is true regarding pain

the cultural background of a patient is important in a nurse's assessment of the patient's pain

What is a description of "review of systems" as part of the health history.

the evaluation of the past and present health state of each body system

an amulet may be used to protect a person from:

the evil eye

diastole

the heart's relaxed filling phase; 2/3 of the heart cycle AV valves are open

systole

the hearts pumping phase

Uremia

the illness accompanying kidney failure (also called renal failure), in particular the nitrogenous waste products associated with the failure of this organ.

aortic valve

the left semilunar valve separating the left ventricle and the aorta

fissure

the narrow crack dividing the lobes of the lungs

The statement "Reason for seeking care" has replaced chief has replaced "Chief complaint." This change is significant because:

the newer term incorporates wellness needs

bronchovesicular

the normal breath sound heard over major bronchi, characterized by moderate pitch and an equal duration of inspiration and expiration

15. When assessing the carotid artery,

the nurse should palpate Medial to the sternoimastoid muscle, one side at a time

Identify the most reliable indicator of a person's pain.

the patient's self-report

5. second heart sound

the result of Closing of the aortic and pulmonic valves

consolidation

the solidification of portions of lung tissue as it fills up with infectious exudate, as in pneumonia

bronchophony

the spoken voice sound heard through the stethoscope, which sounds soft, muffled, and indistinct over normal lung tissue

Environment

the total of all the conditions that make up the surrounding and influence the development of a person

hot/cold theory of health and illness

the treatment of disease consists of adding or subtracting cold, heat, dryness, or wetness to restore the balance of the humors - beverages, foods, herbs, medicines, and diseases are classified as hot or cold according to their perceived effects on the body not on physical characteristics

Holistic health

the view that the mind, body, and spirit are interdependent and function as a whole within the environment

Holistic Health

the view the the mind, body, and spirit are interdependent and function as a whole within the environment.

thought process -

the way a person thinks, the logical train of thought

Crust

thick, dried-out exudate left on skin when vesicles / pustules burst or dry up

arteriosclerosis

thickening and loss of elasticity of the arterial walls

colostrum

thin, yellow fluid, precursor of milk, secreted for a few days after birth

Lichenification

tightly packed set of papules that thickens skin, from prolonged intense scratching

Petechiae

tiny punctate hemorrhages, 1 - 3 mm, round and discrete, dark red, purple, or brown

Erythema toxicum

tiny, punctate red macules and papules on the cheeks, trunk, chest, back, and buttocks

Review of systems *

to evaluate the past & present health of each body system, double check for significant data, & assess health promotion practice * for each body system, assess for symptoms & health promoting behaviors

Family history *

to help detect health risks for the patient * draw a pedigree or genogram

Rational for obtaining a family history

to highlight those diseases and conditions for which a particular pt. may be at increased risk for.

erbs point

traditional auscultatory area in the 3rd left intercostal space

Cutis marmorata

transient mottling on trunk and extremities

pterygium

triangular opaque tissue on the nasal side of the conjunctiva that grows toward the center of the cornea

pronation

turning the forearm so that the palm is down

Supination

turning the forearm so that the palm is up

kussmaul respiration

type of hyperventilation that occurs with diabetic ketoacidosis

palpitation

uncomfortable awareness of rapid or irregular heart rate

the first step to cultural competency by a nurse is to:

understand your own heritage and its basis in cultural values.

"culturally competent" implies that the nurse:

understands the cultural context of the patient's situation.

cryptorchidism

undescended testes

diopter

unit of strength of the lens settings on the ophthalmoscope that changes focus on the eye structures. Red numbers are for focusing on objects further away. Black numbers are for focusing on objects nearby.

pain

unpleasant sensory and emotional experience associated w/ actual or potential tissue damage, or described in terms of such damage. Pain is always subjective

malocclusion

upper or lower dental arches out of alignment

Describe facial charecteristcs that occur with Down Syndrome

upslanting eyes flat nasal bridge thick tongue flat nose

16. Describe the method of everting the upper eyelid for examination.

use a QTip and then pull eyelashes up and around it

Follow up database

used in all setting to monitor progress on short/chronic health problems

Medical diagnosis

used to evaluate the cause of disease; focus is on the function or malfunction of a specific organ system

Medical Diagnosis

used to evaluate the cause of the etiology of disease; focus is on the function or malfunction of a specific organ system.

Nursing diagnosis

used to evaluate the responses of the whole person to health problems

language -

using the voice to communicate one's thoughts and feelings

arrhythmia

variation from the hearts normal rhythm

Bartholin glands

vestibular glands, located on either side of the vaginal orifice, that secrete a clear lubricating mucus during intercourse.

profile sign

viewing the finger from the side to detect early clubbing

pain can be classified by its source

visceral pain, deep somatic pain, cutaneous pain, referred pain

9. Contrast rigidity with voluntary guarding.

voluntary guarding - Person is cold, tense, or ticklish. Bilateral and will feel the muscles relax slightly during exhalation rigidity - Constant, board-like hardness of the muscles, Unilateral; Painful in localized area when person attempts a sit-up.

Waist-hip-ratio

waist circumference / hip circumference

Waist-to-hip ratio (WHR)

waist or abdominal circumference divided by the hip or gluteal circumference; method for assessing fat distribution

Hernia

weak spot in abdominal muscle wall, usually near the inguinal canal or femoral canal, where a loop of bowel protrudes through

Calculate BMI

weight / height^2 x 703

Body mass index

weight in kilograms divided by height in meters squared (W/H^2); values of 30 or more is indicative of obesity; value of less than 18.5 is indicative of under-nutrition

Objective data

what the health professional observes by inspecting, palpating, percussing and auscultation during the physical examination

Subjective data

what the person says about themselves during history taking

thought content -

what the person thinks - specific ideas, beliefs the use of words

provisions of Title VI of the Civil Rights Act of 1964

when people with LEP seek healthcare in settings such as hospitals, nursing homes, clinics, day care centers, and mental health centers, services cannot be denied to them

gynecomastia

Excessive breast development in a male

Confluent

skin lesions that run together

Ephelides

"freckles" - small, flat macules of brown melanin pigment that occur on sun-exposed skin

What is an appropriate recording of a patients' reason for seeking health care?

"grabbing" chest pain for 2 hours

17. A. rests on the diaphragm

Base

Scleroderma

"hard skin". a chronic connective tissue disorder associated with decreased mobility

2. During examination of the scrotum, a normal finding would be that:

C. The left testicle hangs lower than the right.

7. A bimanual technique may be the preferred approach for a woman:

C. with pendulous breasts.

herpes simplex

"cold sores"; clear vesicles with red base that evolve into pustules, usually at lip-skin junction

Seborrheic dermatitis

"cradle cap"

8. The hearing receptors are located in which region?

D. Cochlea

Electronic health recording

Direct computer entry of the patient health record while in the patient's presence

A patient is in the emergency department with nausea and vomiting. What would you include in the database?

A diet and GI history

A patient is in the emergency department with nausea and vomiting. Which would you include in the database?

A diet and GI history

Wellness

A dynamic process and view of health; a move toward optimal functioning.

An area of thin shiny skin with decreased visibility of normal skin markings is called: A. lichenification B. plaque C. atrophy D. keloid

C. atrophy

14. A pleural friction rub is best detected by:

C. auscultation

Incus

"Anvil" - middle of the 3 ossicles of the middle ear

Tympanic membrane

"Eardrum" - thin, translucent, pearly gray, oval membrane that stretches across the ear canal and separates the outer ear from the middle ear. When viewed with an otoscope, it shows the light as a cone shaped reflection.

Malleus

"Hammer" - first of the 3 ossicles of the middle ear; The umbo, manubrium, and short process can be seen through the eardrum.

Formulate a response you would make to a patient who has spoken to you in a way you interpret as sexually aggressive. (p. 42)

"I am uncomfortable when you talk to me that way; please don't" "I wonder if the way you're feeling now relates to your illness or to being in the hospital"

Useful phrase to use as a closing when ending an interview

"Is there anything else you would like to mention?"

Stapes

"Stirrup" - inner of the 3 ossicles of the middle ear.

During pregnancy: dependent edema, varicosities in the legs, and hemorrhoids.

"The symptoms are caused by the pressure of the growing uterus on the veins. They are usual conditions of pregnancy".

A patient has recently received health insurance and would like to know how often he should visit the provider. How do you respond?

"Your visits may vary, depending on your level of wellness."

Furuncle

"boil" - suppurative inflammatory skin lesion due to infected hair follicle

amphthous ulcers

"canker sores"; small, painful, round ulcers in the oral mucosa of unknown cause.

Nevus

"mole" - circumscribed skin lesion due to excess melanocytes

18. List the characteristics to explore when you hear a murmur, including the grading scale for murmurs.

(1) Timing: Systolic or diastolic (2) Loudness (Intensity): Example: ii/vi or 2/6 (use roman numerals or numbers) Grading scale of the murmur: -1. Barely audible -2. Clearly audible, but faint -3. Moderately loud, easy to hear -4. Loud, associated with a thrill palpable on the chest wall -5. Very loud, heard with one corner of the stethoscope lifted off the chest wall. -6. Loudest, still heard with entire stethoscope lifted just off the chest wall.

15. Raynaud phenomenon has associated progressive tricolor changes of the skin from blue to white and then to red. State the mechanism for each of these color changes.

(1) white (pallor) in top figure from arteriospasm and resulting deficit in supply; (2) blue (cyanosis) in lower figure from slight relaxation of the spasm that allows a slow trickle of blood through the capillaries and increased oxygen extraction of hemoglobin; (3) finally, red (rubor) in heel of hand due to return of blood into the dilated capillary bed or reactive hyperemia. May have cold, numbness, or pain along with pallor or cyanosis stage; then burning, throbbing pain, swelling along with rubor. Lasts minutes to hours; occurs bilaterally. Several drugs predispose to the episodes, and smoking can increase the symptoms.

Urticaria

(hives). Wheals coalesce to from extensive reaction, intensely pruritic

Objective assessment of hair

* Inspect and palpate the hair, noting the color, texture, distribution. * Inspect for scalp lesions * Inspect for infestations

Objective assessment of nails

* Inspect and palpate the nails, evaluating the shape, contour, consistency, and color. * Assess capillary refill

Objective assessment of skin

* Observe color & general pigmentation. * Observe subtle skin tone changes: symmetrical skin tone * Observe for color changes that may not be visible: mucous membranes, lips, nail beds, sclera * Assess skin temperature, moisture, texture, thickness, edema, mobility, turgor, vascularity or bruising, and lesions. * If a lesion is observed, note: color, elevation, shape, size, location and distribution on the body, and any exudate

Key properties of skin

* Protects the body * Prevents penetration * Allows perception of touch, pain, temperature, and pressure * Regulates temperature * Allows identification of people * Allows communication of emotions * Repairs wounds * Absorbs and excretes substances * Produces vitamin D

Reason for seeking care

* formerly known as chief of complaint * in patient's own words, briefly describe the reason for visit * write this in quotes

Present health or history of present illness

* well person- briefly note the general state of health * sick person- chronologically record the reason for seeking care

5. In examining the ear of an adult, the canal is straightened by pulling the auricle:

C. up and back

8. After assessing the patient's pulse, the practitioner determines it to be "normal." This would be recorded as:

2+

5. Describe the whispered voice test of the hearing acuity.

*Stand arm's length (2 ft.) behind the person *Test one ear at a time while masking hearing in the other ear to prevent sound transmission around the head: done by placing one finger on the tragus and pushing in and out of the auditory meatus. *Exhale fully and whisper slowly a set of 3 random numbers and letters (ex. "5,B,6") * The person repeats each number/letter correctly * If the response is not correct, repeat the whispered test using a different combination of 3 numbers/letters *A passing score is at least 3 correct out of 6 items whispered

Give an example of objective data

2- x 5-cm scar present on the right lower forearm

PHYSICAL APPEARANCE

- Age -Sex -LOC -Skin Color -Facial Features

7. List and describe common thoracic deformities.

- Barrel chest: Note equal anteroposterior-to-transverse diameter and that ribs are horizontal instead of the normal downward slope. This is associated with normal aging and also with chronic emphysema and asthma as a result of hyperinflation of lungs. - Pectus excavatum: markedly sunken sternum and adjacent cartilages (also called funnel breast). Depression begins at second intercostal space, becoming depressed most at junction of xiphoid with body of sternum. More noticeable on inspiration. Congenital, usually not symptomatic

17. List 5 factors that can cause extraneous noise during auscultation.

- Examiner's breathing on stethoscope tubing - Stethoscope tubing bumping together - Patient shivering - Patient's hairy chest; movement of hairs under stethoscope sounds like crackles (rales) - Rustling of paper gown or paper drapes

BEHAVIOR

- Facial Expression: - Mood and Affect - Speech - Dress - Personal hygiene

MOBILITY

- Gait - Range of motion

1. Identify appropriate history questions to ask regarding the breast examination.

- Is there any history of breast disease? What type? How was it diagnosed? When? - Is there a family history of breast disease? What ages?

9. List the symptom areas to address during history taking of the peripheral vascular system.

- Leg pain or cramps - Skin changes on arms or legs - Swelling in the arms or legs - Lymph node enlargement - Medication

11. Contrast the appearance of nasal turbinates versus nasal polyps.

- Nasal polyps: are smooth, pale gray, avascular, mobile, and non-tender - Turbinates: warm the inhaled air: highly vascular.

8. Name the related organs in the lymphatic system.

- Spleen; (1) to destroy old red blood cells (2) to produce antibodies (3) to store red blood cells (4) to filter micro-organisms from the blood - Tonsils; respond to local inflammation - Thymus; important in developing the T lymphocytes if the immune in children

BODY STRUCTURE

- Stature - Nutrition - Symmetry - Posture - Position - Body build, Contour

2. Describe the pleura and it's function.

- Thin, slippery pleura form an envelope between the lungs and chest wall - Outside layer visceral; inside layer parietal - Function: form cushion for lungs - Pleural cavity has negative pressure or vacuum which holds the lungs tightly against chest wall - Helps lungs slide smoothly

12. Describe the appearance and clinical significance of these findings on the tongue: ankyloglossia, fissured tongue, geographic tongue, black hairy tongue, macroglossia.

- ankyloglossia: congenital defect, a short lingual frenulum, here fixing the tongue tip to the floor of the mouth and gums. -fissured tongue or scrotal: Deep furrows divide the papillae into small irregular rows, increase with age. The condition occurs in 5% of the general population and Down Syndrome. -geographic tongue: pattern of normal coating interspersed with bright red, shiny, circular bllad area with raised pearly borders. Pattern resembles a map and changes in a few days. Cause is unknown. -black hairy tongue: this is not hair, rather, the elongation of filiform papillae and painless overgrowth of mycelial threads of fungus infection on the tongue. -macroglossia: enlarged tongue- The tongue is enlarged and may protrude from the mouth. The condition is not painful but may impair speech development. Occurs with Down syndrome, cretinism, myxedema, acromegaly.

3. Name the functions of the ciliary body, the pupil, and the iris.

-Ciliary body: controls the thickness of the lens. -Iris: functions as a diaphragm to change the amount of light allowed into the eye. -Pupil: constricted by contractions of the iris.

8. List the changes observed during the peri-menopausal period.

-1-2 years of decline in ovarian function- irregular menses that gradually become farther apart and produce lighter flow. -Ovaries stop producing progesterone and estrogen. -Uterus shrinks in size. -ovaries atrophy to 1-2 cm and not palpable after menopause. -Sacral ligaments relax, pelvic muscles weaken, uterus drops, cervix shrinks and looks paler with thick, glistening epithelium. -Vagina becomes shorter and narrower and less elastic -Decrease in secretions. -pH becomes more alkaline. -Mons pubis looks smaller because fat pad atrophies. -Labia and clitoris gradually decrease in size. -Pubic hair becomes thin and sparse.

18b. Study the lobes of the lungs and label their landmarks on the following illustration

-4th rib -horizontal fissure -5th rib midaxillary line -right oblique fissure -6th rib midclavicular line -left oblique fissure

6. Describe the appearance of a deviated nasal septum and a perforated septum.

-A deviated nasal septum looks like a hump or shelf in one nasal cavity. -A perforated septum is seen as a spot or light from a penlight that is shining in the other naris.

4. Describe how an image formed on the retina compare, the posterior chamber, and the vitreous body.

-Anterior chamber: behind the cornea but in front of the lens. -Posterior chamber: behind the iris and around the lens; contains watery aqueous humor that is produced by the ciliary body. -Vitreous body: soft gelatinous humor

Ways to modify interview for hearing-impaired.

-Ask preferred method of communication. -Interpreter is then important for health history. -If able to read lips, face them. -Facial hair/ accents make interview difficult. -Be sure to make sure they understand.

19. Explain the rationale for testing the strabismus during early childhood.

-Brain begins to suppress data from the weak eye and causes visual acuity the deteriorate in the weak eye.

18. Describe the 3 types of normal breath sounds.

-Bronchial (Tracheal) High pitch; Loud Amplitude; Duration- inspiration < expiration; Quality - harsh, hollow tubular; Normal Loc - Trachea and Larynx -Bronchovesicular Moderate pitch; Moderate Amp; Duration - inspiration = expiration; Quality - mixed; Normal Loc - over major bronchi, where fewer alveoli are located -Vesicular Low pitch; Soft Amp; Duration - inspiration > expiration; Quality - rustling, like the sound of the wind in the trees; Normal Loc - Over peripheral lung fields, where air flows through smaller bronchioles and alveoli

8. List and describe 3 types of normal breath sounds.

-Bronchial (Tracheal) High pitch; Loud Amplitude; Duration- inspiration < expiration; Quality - harsh, hollow tubular; Normal Loc - Trachea and Larynx -Bronchovesicular Moderate pitch; Moderate Amp; Duration - inspiration = expiration; Quality - mixed; Normal Loc - over major bronchi, where fewer alveoli are located -Vesicular Low pitch; Soft Amp; Duration - inspiration > expiration; Quality - rustling, like the sound of the wind in the trees; Normal Loc - Over peripheral lung fields, where air flows through smaller bronchioles and alveoli

10. Discuss the most common causes of decreased visual function in the older adult.

-Cataract formation: Lens opacity, proteins clumping in the lens; hard for light to get through. -glaucoma: increase in ocular pressure; gradual loss of peripheral vision; more common in men than women. -macular degeneration: Age related MD causes vision loss in older adults.

Overnutrition

-Caused by the consumption of nutrients, especially calories, sodium, and fat, in excess of body needs. -Leads to obesity, heart disease, type II diabetes, hypertension, stroke, gallbladder disease, sleep apnea, certain cancers, and osteoarthritis

21. Describe the following 4 types of red eye, and explain their significance: conjunctivitis, subconjunctival hemmorrhage, iritis, acute glaucoma.

-Conjunctivitis is pink eye: red, beefy-looking vessels at periphery of eye, usually clearer around the iris; commonly from viral or bacterial infection, allergy, or chemical irritation. -Subconjunctival hemorrhage: red patch on the sclera; looks alarming, but isn't serious; occurs from increased intraocular pressure. -Iritis: deep, dull red halo around iris and cornea; marked photophobia, constricted pupil, blurred vision, and throbbing pain. -Acute glaucoma: redness around iris , dilated and oval pupil, steamy cornea, and shallow anterior chamber; results from sudden increase in intraocular pressure from blocked outflow from anterior chamber.

1. Describe the most important points about the health history for the respiratory system.

-Cough -Shortness of breath -Chest pain with breathing -History or respiratory infections -Smoking history -Environmental exposure -Self-care behaviors

9. Describe the appearance of these nodules that could be present on the external ear:

-Darwin's tubercle - small painless nodule at helix. -sebaceous cyst - filled with sebaceous material and can be painful if infected. -tophi - small, whitish, yellow, hard, non-tender nodules in or near helix that contains greasy, chalky, material. -chondrodermatitis - painful nodules develope on rim of helix. --small, indurated, dull red, poorly defined and very painful. --result of repetitive mechanical pressure or environmental trauma. -keloid- overgrowth of scar tissue which invades original site of trauma. -carcinoma - ulcerated, crusted nodule with indurated base that fails to heal bleeds intermittently.

5. After tooth loss in middle or older adult, describe the consequences of chewing with the remaining maloccluded teeth.

-Excessive bone resorption with further tooth loss. -Muscle imbalance resulting from a mandible and maxilla now out of alignment, which produces muscle spasms, tenderness of muscles of mastication, and chronic headaches. -The temporomandibular joint is stressed, leading to osteoarthritis, pain, and inability to fully open the mouth.

12a. Fill in the labels indicated in the following illustrations (outer ear).

-External auditory meatus -tragus -antitragus -lobule -location of mastoid process -antihelix -helix

13a. Fill in the labels indicated on the following illustrations (nasal cavity)..

-Frontal sinus -superior turbinate/concha and meatus -middle turbinate and meatus -Interior turbinate and meatus -hard/bony palate -palatine tonsil in oropharynx -soft palate -opening of the Eustachian tube in the nasopharynx -pharyngeal tonsil -location of the opening of the frontal sinus -sphenoid sinus -cranial nerve 1, olfactory nerve

1. Discuss ways of creating an environment that will provide psychological comfort for both the woman and practitioner during the female genitalia examination.

-Have them empty their bladder before -Position table so perineum not exposed to inadvertent door open -If they want, friend, family, chaperone present. But still maintain privacy -Elevate head and shoulders to semi-fowlers to maintain eye contact -Place stirrups so legs are not abducted to far -Explain each step before doing it -Assure she can say stop at any time -Be gentle but firm -Communicate

Adverse outcomes related to patient malnutrition

-Impaired wound healing -Increased infection risk -Suppression of immune system -functional loss with increased falls risk -Increased risk of pressure ulcers -longer hospital stay -Increased mortality

8. What age-specific considerations would you make for the examination of the: Infant? Toddler? Preschooler? School-age child? Adolescent? Older adult? Acutely ill person?

-Infant - establish trust, parent must be present, perform steps while baby sleeps; assess head, eye, ear, nose, and throat last. -Toddler - fear of invasive procedures and being restrained, toddler must be in parent's lap for the examination, may not like undressing; assess head, eye, ear, nose, and throat last, -Preschooler - toddler may sit in adult lap; 4-6 year old may sit on the exam table, use short and simple explanations; assess head, eye, ear, nose, and throat last, -School-age child - Break the ice with conversation, child can undress self, demonstrate equipment, comment on the body & how it works; work head to toe, -Adolescent - examine alone, focus on health promotion, give positive feedback about development, -Older adult - adjusting to changes in physical strength and health, allow rest periods when needed, touch is important because other senses may be diminished, be aware that aging years contain more life stress, -Acutely ill person - alter the position depending for relief/comfort, exam problem areas first, collecting a mini-database, then finish complete assessed after first problem is resolved

Mild malnutrition

85% to 95% of normal body weight

11. How many degrees is the normal costal angle?

90 degrees

3. Differentiate among light, deep, and bimanual palpation.

-Light palpation evaluates surface characteristics and identifies areas of tenderness. -Deep palpation assesses an organ or mass deeper in a body cavity. -Bimanual palpation is the use of both hands to envelop or capture certain body parts or organs, such as the kidneys.

13b. Fill in the labels indicated on the following illustrations (nasal structures).

-Nasal bridge -nasal tip -ala -vestibule -columella -nares

3. Describe the appearance or sketch these normal variations of the cervix and os:

-Nulliparous: small and round in nulliparous woman -Parous: has a horizontal irregular slit and also may show healed unilateral or bilateral lacerations -Stellate Lacerations; star shaped -Cervical eversion: endocervical canal is everted of "rolled out". Ectropion may normally occur after vaginal deliveries. -Nabothian cysts: benign growths that commonly appear on the cervix after childbirth; they are small, smooth, yellow nodules that may be single or multiple

18c. Study the lobes of the lungs and label their landmarks on the following illustration

-Oblique fissure -expiration -inspiration -T12 -T10 -T3

Define undernutrition

-Occurs when nutritional reserves are depleted and/or when nutrient intake is inadequate to meet day-to-day needs or added metabolic demands. -vulnerable groups are infants, children, pregnant women, recent immigrants, people with low incomes, hospitalized people, and aging adults.

4. List the 3 pairs of salivary glands, including their location and the locations of their duct openings.

-Parotid -Submandibular -Sublingual secrete saliva --clear fluid that moistens and lubes food bolus, starts digestion and cleans and protects mucosa ADD LOCATION

13. Describe the procedure and rationale for determining costovertebral angle/CVA tenderness.

-Place hand over 12th rib at the costovertebral angle on the back. -Thump the hand with the ulnar edge of the other first Pain occurs with inflammation of the kidney or paranephric area

Normal physiologic changes in the elderly that affect nutritional status

-Poor dentition -decreased visual acuity -decreased saliva production -slowed GI motility -decreased GI absorption -diminished olfactory and taste sensitivity

Major risks factors for undernutrition or overnutrition in the elderly?

-Poor physical or mental health -Social isolation -Alcoholism -Limited functional ability -Poverty -Polypharmacy

14. Describe the technique for using the Doppler ultrasonic probe to detect peripheral pulses.

-Position person supine, with legs externally rotated so you can reach the medial ankles easily -Place a drop of coupling gel on the end of the handheld transducer -Place the transducer over a pulse site, swivelled at a 45o angle. -Apply very light pressure, locate the pulse site by the swishing, whooshing sound

12c. Fill in the labels indicated in the following illustrations (tempanic membrane).

-Posterior fold -incus -umbo -annulus -pars tensa -cone of light -manubrium of malleus -short process of malleus -anterior fold -pars flaccida

9. Identify common age related changes in the eye.

-Pupil size decreases. -Lens elasticity decreases. -Decrease in accommodation. -Presbyopia. -The vitreous humor does not recycle as frequently.

10. Describe the appearance and clinical significance of these findings in the infant: sucking tubercle, Epstein pearls, Bednar aphthae.

-Sucking tubercle: a normal finding in infants, a small pad in the middle of the upper lip from friction of breastfeeding or bottle-feeding. - Epstein pearls: a normal finding in newborns and infants, small, white, glistening, pearly papules on the hard palate, and on the gums, where they look like teeth. They are small retention cysts and disappear in the first few weeks. -Bednar aphthae: traumatic areas or ulcers on the posterior hard palate on either side of the midline that result from abrasions while sucking.

11. Explain the statement that normal visual acuity is 20/20.

-The number on top is the distance that you are away from the chart. -The bottom number is the distance that a normal person with normal vision will stand from the chart and see clearly.

Define optimal nutritional status

-When sufficient nutrients are consumed to support day-to-day body needs and any increased metabolic demands caused by growth, pregnancy, or illness. -People having optimal nutritional status are more active, have fewer physical illnesses, and live longer than people who are malnourished

13. In the space below, define a cleft palate and a bifid uvula.

-bifid uvula: complete or partial split in uvula -cleft palate: gap in roof of mouth

15. State 4 guidelines to distinguish S1 from S2.

-cadence or relative length of the phase -location -timing of S1 with apical beat or carotid -pulse

18a. Fill in the labels indicated on the following illustrations (ribs).

-clavicle -2nd intercostal space -dome of the diaphragm -7th intercostal space -costal margin -costal angle -xiphiod process -costochondral junction -body of sternum -Sternal angle of Louis -manubrium of sternum -suprasternal notch

initial pain assessment

-clinician ask patient to answer 8 questions concerning location, duration, quality, intensity, and aggravating/relieving factors -further questions are asked about the manner of expressing pain and the effect of pain that impair ones quality of life

4. Discuss ways of creating an environment that will provide psychological comfort for the man and the examiner during examination of the male genitalia.

-confident, relaxed demeanor -unhurried yet business like -do not discuss GU history or sexual practices while performing exam -firm deliberate touch -if erection occurs, reassure that that is a normal physiologic response to touch

20. Describe these findings, and explain their significance: epicanthic fold, pseudostrabismus, ophthalmia neonatorum, brushfield spots.

-epicanthic fold: skin across the inner corner of the eye (canthus). -pseudostrabismus: the appearance of strabismus because of epicanthic fold but is normal for a young child. -Ophthalmia neonatorum, conjunctivitis of the newborn: purulent discharge caused by a chemical irritant or a bacterial or viral agent from the birth canal. -brushfield spots: white specks around the edge of the iris.

12b. Fill in the labels indicated in the following illustrations (inner ear).

-external auditory canal -mastoid process -tympanic membrane -round window -eustachian tube -inner ear -conchlea -cranial nerve VIII -vestibule -semicircular canals -stapes in oval window -incus -malleus -skull bone -cartilage

8. Contrast the physical appearance and clinical significance of the following: leukoedema; candidiasis; leukoplakia, Fordyce granules.

-leukoedema: a large patch, grayish opaque that may be present along the buccal mucosa. It is more common in blacks and East Indians. When it is mild, the patch disappears as you stretch the cheeks. The severity increases with age, and it looks grayish white and thickened. The cause is unknown. -candidiasis: infection, a white cheesy, curdlike patch on the buccal mucosa and tongue. It scrapes off, leaving a raw, red surface that bleeds easily. -leukoplakia: chalky, white, thick, raised patch with well-defined borders. The lesion is firm and does not scrape off, and is due to chronic irritation. These lesions are precancerous. -Fordyce granules: small, isolated white or yellow papules on the mucosa of the cheek, tongue, and lips. They are painless sebaceous cysts and not significant.

2. Discuss selection, preparation, and insertion of the vaginal speculum.

-lubricate/warm it with warm water and gel lubricant -Insertion: Push the introitus down and open to relax the pubococcygeal muscle. Ease insertion by asking woman to bear down. Hold speculum with index and middle fingers surrounding blades and thumb under thumbscrew; this prevents blades from opening painfully during insertion. Tilt width of blades obliquely and insert speculum past your fingers, applying any pressure in a downward slope toward small of back; this avoids pressure on sensitive urethra above it.

2. Describe the size and components of the nasal cavity.

-much larger than the external nose. -it extends back over the roof of the mouth. -the anterior edge of the cavity is lined with numerous coarse nasal hairs. lined with blanket of ciliated mucous membrane

18. Contrast the use of the negative diopter or red lens settings with the positive diopter or black lens settings on the ophthalmoscope.

-negative diopter/red, concave lens: used for near sighted, corrects focal point -positive diopter/black, convex lens: use for farsighted, corrects focal point

21c. Fill in the labels indicated on the following illustrations (dilated pupil).

-optic disc -physiologic cup -vein -artery -macula -fovea centralis

10. Describe the appearance of these conditions that could appear in the ear canal:

-osteoma - single, stoney hard, rounded nodule that obscures the drum: nontender, benign. -exostosis - small, boney hard, rounded nodules of hypertrophic bone, convered with normal epithelium; frequently in cold water swimmers; no treatment needed -furuncle - painful, reddened, infected hair follicle. -polyp - arises in canal from mucosal tissue; bleeds easily; indicates chronic ear disease; Benign.

14. Concerning malalignment of the eye axes, contrast phoria with tropia.

-phoria: Mild weakness when fusion is blocked. -tropia: More severe constant malalignment.

6. List laboratory tests to assess urinary function.

-serum analysis of kidney function measuring creatinine -blood urea nitrogen, BUN

21a. Fill in the labels indicated on the following illustrations (palpebral fissure).

-upper eyelid, -palpebral fissure, -lateral canthus, -lower lid, -limbus - border between cornea & sclera, -caruncle, -medial canthis, -sclera, -Iris, -pupil.

18d. Label the normal location of the three types of breath sounds on the posterior and anterior chest walls.

-vesicular -bronchovesicular

18e. Study the lobes of the lungs and label their landmarks on the following illustration

-vesicular -bronchovesicular -bronchial/trachea

11. List the disease state suggested by the following descriptions of the appearance of the eardrum.

-yellow-amber color tympanic membrane - otitis media with effusion. -pearly gray color tympanic membrane - normal tympanic membrane. -air-fluid level tympanic membrane - serous otitis media. -distorted light reflex tympanic membrane - acute otitis media. -red color tympanic membrane - actute purulent otois media. -dense white areas on tympanic membrane - sequelae of infections. -oval dark areas tympanic membrane - drum rupture. -black or white dots on drum - fungal functions. -blue drum - Trauma, skull fracture.

Rationale for obtaining a systems review

. evaluate the past and present health state of each body system 2. double check incase any significant data were omitted in the present illness section. 3. evaluate health promotion practices.

Knowledge of the use of personal space is helpful for the health care provider. Personal distance is generally considered to be:

1 1/2 to 4 feet

9. List the 4 point grading scale for the size of tonsils.

1+ Visible. 2+ Halfway between tonsillar palate and uvula. 3+ Touching the uvula. 4+ Touching each other.

4 point grading scale for pitting edema:

1+ mild 2+ moderate 3+ deep pitting 4+ very deep pitting, very swollen

5. List points to include in teaching the breast self-examination.

1. Affirm she is healthy 2. Regular exam -right after menstrual period or 4 -7 days after the 1st day of mensturation. 3. Emphasize the absence of lumps 4. The majority of women will never get cancer 5. Most lumps are benign 6. Early detection is important. 7. Keep teaching simple. 8. Teach to do in front of a mirror or in shower then on back. 9. Assess technique 10. give out info.

3. List the 4 sets of paranasal sinuses, and describe their function.

1. Frontal Sinuses 2. Ethmoidal Sinuses 3. Maxillary Sinuses 4. Sphenoid Sinuses ADD FUNCTIONs

List questions you could ask a patient that would screen for suicide ideation. -

1. Have you ever felt that life is not worth living? 2. Have you ever felt so blue that you thought of hurting yourself? 3. Do you feel like hurting yourself now? 4. Do you have a plan to hurt yourself? 5. How would you do it? 6. What would happen if you were dead? 7. How would other people react if you were dead? 8. Whom could yu tell if you felt like killing yourself?

1. Name the functions of the nose.

1. Olfaction 2. Respiration 3. Warming of Air 4. Humidification of Air 5. Filtering of Air

12b. Fill in the name of each cranial nerve, and then write S for sensory, M for motor, or MX for mixed.

1. Olfactory 2. Optic 3. Oculomotor 4. Troclear 5. Trigeminal 6. Abducens 7. facial 8. Acoustic 9. Glossopharyngeal 10. vagus 11. Accessory 12. Hypoglossal

List situations in which it would be necessary to preform a complete mental status examination. -

1. Patients whose initial brief screening suggests an anxiety disorder or depression. 2. Family members concerned about a person's behavioral changes such as memory loss or inappropriate social interactions. 3. Brain Lesions (trauma, tumor, stroke). A Mental status assessment documents any emotional cognitive change associated with the lesion. Not recognizing these changes hinders care planning and creates problems with social readjustment. 4. Aphasia (the impairment of language ability secondary to brain damage). A mental status examination assesses language dysfunction and any emotional problems associated with it such as depression or agitation.

14. List 3 factors that affect the normal intensity of tactile fremitus?

1. Relative location of bronchi to the chest wall 2. Thickness of the chest wall 3. Pitch and intensity

3. Describe 3 mechanisms that help return venous blood to the heart.

1. The contracting skeletal muscles that milk the blood proximally, back toward the heart 2. The pressure gradient caused by breathing, in which inspiration makes the thoracic pressure decrease and the abdominal pressure increase 3. The intraluminal valves, which ensure unidirectional flow.

List various sources of pain.

1. Visceral pain: originates from larger interior organs. *That is why visceral pain often presents with autonomic responses such as vomiting, nausea, pallor, and diaphore 2.Deep somatic pain: comes from sources such as the blood vessels, joints, joints, tendons, muscles, and bone. Injury may result from pressure, trauma, or ischemia 3. Cutaneous pain: Linking pain to a mental disorder (psychogenic pain) negates person's pain report 4. Somatic Pain: originates from musculoskeletal tissues or the body surface. 5. Referred Pain: Pain that is felt at a particular site but originates from another location.

5. List the 5 components of a deep tendon reflex arc.

1. an intact sensory nerve (afferent) 2. a functional synapse in the cord 3. an intact motor nerve fiber (efferent) 4. neuromuscular junction 5. a competent muscle

6. List 4 situations in which you clean your hands promptly and thoroughly.

1. before and after every physical patient encounter, 2. after contact with blood, body fluids, secretions, and excretions, 3. after contact with any equipment contaminated with body fluids, 4. after removing gloves.

Severe Malnutrition

< 75% of normal body weight

B. pulmonic

12. You will hear a split S2, most clearly in which area?

14. A murmur

14. A murmur is heard after S1, and before S2. This murmur would be classified as systolic, possibly benign

E. Pericardium

17. Tough, fibrous, double-walled sac that surrounds and protects the heart.

C. Endocardium

18. Thin layer of endothelial tissue that lines the inner surface of the heart chambers and valves.

F. Atrium

19. Reservoir for holding blood

12. When comparing the anteroposterior diameter of the chest with the transverse diameter, what is the expected ratio? What is the significance of this?

1:2 or 5:7 are the expected ratios. It has a role in the breathing cycle; inspire and exhale fully.

A. Pericardial fluid

20. Ensures smooth, friction-free movement of the heart muscle.

Visual acuity

20/20 is normal vision; This means that a patient can stand at 20 feet and read what a person with normal vision can read at 20 feet.

B. Ventricle

21. Muscular pumping chamber

D. Myocardium

22. Muscular wall of the heart

23. D. Sunken sternum and adjacent cartilages

23. pectus excavatum

States Convenient Ways to assess a person's recent memory within the context of the initial health history. -

24-hour diet recall or by asking the time the person arrived at the agency. Ask questions you can corroborate.

24. F. forward protrusion of the sternum with ribs sloping back at either side

24. pectus carinatum

25. C. lateral S-shaped curvature of the thoracic and lumber spines

25. Scoliosis

26. B. exaggerated posterior curvature of the thoracic spine.

26. kyphosis B. exaggerated posterior curvature of the thoracic spine.

5. State the body mass index for a male weighing 190 lb who is 5' 10" tall___, female weighing 136 lb who is 5' 4" tall ___.

28, 23

16. Normal findings for diaphragmatic excursion are:

3 to 5 cm or 7 to 8 cm in well-conditioned people; it may be somewhat higher on the right side (1- 2 cm) because of the presence of the liver.

9. Define the 4-point grading scale for deep tendon reflexes.

4+ very brisk, hyperactive with clonus, indicative of disease 3+ brisker than average, may indicate disease, probably normal 2+ average, normal 1+ diminished, low normal, or occurs only with reinforcement 0 no response

rating scales can be introduced

4-5 yrs of age

Tanner staging

5 stages of breast development: 1. preadolescence 2. Breast buds 3. Overall growth 4. Areola mound 5. Mature breasts

Epitrochlear node

5. Inspection of a person's right hand reveals a red swollen area. To further assess for infection, you would palpate the:

List situations in which it would be necessary to preform a complete mental status examination. Continued: -

5. Symptoms of psychiatric mental illness, especially with acute onset.6. Any known illnesses or health problems such as alcohol use disorders or chronic renal disease. 7. Current medications with side effects that may cause confusion or depression. 8. The usual educational and behavioral level - note that factor as the normal baseline and do not expect performance on the mental status examination to exceed it. 9. Responses to personal history questions indicating current stress, social interaction patterns, sleep habits, drug and alcohol use.

faces pain scale revised

6 drawing of faces that show pain intensity, from no pain on left (score of 0) to very much pain on right (score of 10)

Children usually come for health care with a parent. At about what age should the interviewer begin to question the child himself or herself regarding presenting symptoms?

7 years

Moderate malnutrition

75% to 84% of normal body weight

B. second left interspace

8. The nurse auscultates the pulmonic valve area in this region

Abnormal characteristics of pigmented lesions are summarized in the mnemonic A-B-C-D-E:

A - asymmetry (not regularly round or oval, two halves of lesion do not look the same) B - border irregularity (notching scalloping, ragged edges, poorly defined margins) C - color variation (areas of brown, tan black, blue, red, white, or combination) D - diameter greater than 6mm (the size of a pencil eraser) E - elevation or enlargement ADDITIONAL: lesion that is rapidly changing, new pigmented lesion, development of itching, burning, bleeding.

5. Discuss the rationale for bimanual examination and list normal findings for the cervix, uterus, adnexa.

A Pap test and pelvic exam are important parts of a woman's routine health care because they can detect cancer or abnormalities that may lead to cancer of the cervix.

14. Define bruit, and discuss what it indicates.

A blowing, swishing sound indicating blood flow turbulence. Can be due to a local vascular cause, such as atherosclerotic narrowing.

AUSCULTATORY GAP -

A brief period when Korotkoff sounds disappear during auscultation of blood pressure; Common with hypertension

A patient asks the nurse, "May I ask you a question?" This is an example of:

A closed question

Factors creating a higher risk for cardiac disease and death?

A combination of increased waist circumference, high blood pressure, and hyperglycemia

Complete database

A complete health history and full physical examination

Complete (Total Health) Database

A complete health history and full physical examination. Most often used in long term care.

Values

A desirable or undesirable state of affairs and a universal feature of all cultures

Title VI of the Civil Rights Act of 1964

A federal law that mandates that when people with limited English proficiency seek health care in health care settings such as hospitals, nursing homes, clinics, daycare centers, and mental health centers, services cannot be denied to them

12. Describe the method of testing for presbyopia.

A hand held vision screening.

Nursing Process

A method of collecting and analyzing clinical information with the 6 main components; 1) Assessment, 2) Diagnosis, 3) Outcome Identification, 4) Planning, 5) Implementation, and 6) Evaluation.

Diagnostic Reasoning

A method of collecting and analyzing clinical information with the following components: 1) attending to initially available cues, 2) formulating diagnostic hypothesis, 3) gathering data relative to the tentative hypothesis, 4) evaluating each hypothesis with the new data collected, and 5) arriving at the final diagnosis.

Semicoma -

A partial or mild comatose state; a coma from which a person may be roused by various stimuli.

What would be included in a holistic model of assessment?

A patient's perception of his or her health status

Cue

A piece of information, a sign or symptom, or a piece of laboratory data.

16. Define pulse deficit, and discuss what it indicates.

A pulse deficit signals a weak contraction of the ventricle; it occurs with atrial fibrillation, premature beats, and heart failure.

Leading question

A question that implies that one answer would be better than another

Vertigo

A sense that either one's own body (subjective) or the environment (objective) is revolving; may indicate inner ear disease

Ishihara test

A series of polychromatic cards with a hidden pattern among the dots.

Cultural and Linguistic Competence

A set of congruent behaviors, attitudes, and policies that come together in a system among professionals that enables work in cross-cultural situations

Health Promotion

A set of positive acts we can take to help consumers choose a healthier lifestyle.

Ethnicity

A social group within the social system that claims to possess variable traits such as a common geographic origin, migratory status, and religion

Vertigo

A spinning, twirling sensation. Objective- feels as if the room spins. Subjective-feels as if he or she spins

The practitioner, entering the examining room to meet a patient for the first time, states: "Hello, I'm M.M., and I'm here to gather some information from you and to perform your examination. This will take about 30 minutes. D.D. is a student working with me. If it's all right with you, she will remain during the examination." Which of the following must be added to cover all aspects of the interview contract?

A statement regarding confidentiality, patient costs, and the expectations of each person.

Hypothesis

A tentative explanation for a cue or a set of cues that can be used for a basis for further investigation.

3. During a visit for a school physical, the 13-year-old girl being examined questions he asymmetry of her breast. What is the nurse's best response.

A. "One breast normally may grow faster than the other during development."

1. The gastrocolic reflex is:

A. a peristaltic wave.

During a routine visit, M.B. age 78, asks about small, round, flat, brown macules on the hands. The best response is: A. "these are the result of sun exposure and do not require treatment" B. "these are related to exposure to the sun. they may become cancerous." C. "these are the skin tags that occur with aging. No treatment is required." D. "I'm glad you brought this to my attention. I will arrange for a biopsy."

A. "age spots" - the result of sun exposure and do no require treatment

To determine if a dark skinned person is pale, the nurse should assess the color of the: A. Conjunctivae B. Ear lobes C. Palms of the hands D. Skin in the antecubital space

A. Conjunctivae

To assess for early jaundice, you will assess: A. sclera and hard palate B. nail beds C. lips D. all visible skin surfaces

A. sclera and hard palate

Milia occur because: A. sebum occludes skin follicles B. of a vascular occlusion in the skin C. excess carotene is ingested D. of a genetic variation in skin tone

A. sebum occludes skin follicles

An older person needs to be assessed before going home as to whether he or she is able to go outside alone safely. Which test is best for this assessment? A. up and go test B. performance of activities of daily living test C. older americans resources and services multidimensional functional assessment questionnaire D. Lawton IADL instrument

A. up and go test

8. During the examination of a full-term male newborn, a finding requiring investigation would be:

A. An absent testes.

13. Pulse oximetry measures:

A. Arterial oxygen saturation of hemoglobin

An appropriate tool to assess an individual's instrumental activities of daily living is a tool by: A. Katz B. Lawton C. Tinetti D. Norbeck

B. Lawton

Functions of the skin include: A. Production of Vit. C B. Temperature regulation C. The production of new cells by melanocytes D. The secretion of a drying substance called sebum

B. Temperature regulation

12. Which is the most common bacterial sexually transmitted infection in the United States?

A. Chlamydia.

The components of a nail examination include: A. Contour, consistency, and color B. Shape, surface, circulation C. Clubbing, pitting, and grooving D. Texture, toughness, and translucency

A. Contour, consistency, and color

10. A common cause of a conductive hearing loss:

A. Impacted cerumen

8. A positive Babinski sign is:

A. Dorsiflexion of the big toe and fanning of all toes

13. Gynecomastia is

A. Enlargement of the male breast.

15. A patient has a barrel-shaped chest, characterized by:

A. Equal anteroposterior transverse diameter and ribs being horizontal

4. Foods that may be beneficial to reduce the risk for colon cancer include:

A. Foods high in fiber.

The bell of the stethoscope is used:

A. For soft, low-pitched sounds

5. Examination of the shoulder includes 4 motions. These are:

A. Forward flexion, internal rotation, abduction, and external rotation.

Lyme disease is more prevalent from: A. from May through September B. along the West Coast C. in children younger than 3 years D. in those participating in water sports

A. May through September

1. A function of the venous system includes:

A. Holding more blood when blood volume increases

3. Note aspects of normal gait

Base is as wide as the shoulder width. Foot placement is accurate, walk is booth, coordinated, even and well balanced, symmetrical arm swing present

6. During an initial home visit, the patient's temperature is noted to be 97.4 F. How would you interpret this?

A. It cannot be evaluated without knowledge of the person's age.

4. Differentiate testing of active range of motion versus passive range of motion.

Active range of motion is done entirely by the patient. Passive range of motion is where you help the patient or solely manipulate the body to make the movements.

11. Which of the following is normal, common findings on inspection and palpation of the valve and perineum?

A. Labia majora that are wide apart and gaping.

10. auscultation technique for pericardial friction rub.

A. Listen with the diaphragm, patient sitting up and leaning forward, breath held in expiration.

2. Which retinal structures can be viewed through the ophthalmoscope?

A. Optic disk, the retinal vessels, the general background, and the macula

normal jugular venus pressure.

A. Patient elevated to 30 degrees, internal jugular vein pulsation at 1 cm above sternal angle. B. Patient elevated to 30 degrees, internal jugular vein pulsation at 2 cm above sternal angle. C. Patient elevated to 40 degrees, internal jugular vein pulsation at 1 cm above sternal angle.

4. Which of the following assessments best confirms symmetric chest expansions?

A. Placing hands on the posterolateral chest wall with thumbs at the level of T9 or T10 and then sliding the hands up to pinch a small fold of skin between the thumbs.

15. Documentation of an eye examination can include the term PERRLA. What does this mean?

A. Pupils B. Equal C. Round D. Reactive to E. Light F. Accommodation

A flat macular hemorrhage is called a(n): A. Purpura B. Ecchymosis C. Petechiae D. Hemangioma

A. Purpura

8. When using the ophthalmoscope, you would:

A. Remove your own glasses and approach the patient's left eye with your left eye.

differentiate a split S2 from S3

A. S3 is lower pitched and is heard at the apex

9. The function of the nasal turbinates is to:

A. warm the inhaled air.

2. A young woman has come for her first gynecologic examination. Because she has not had any children, the examiner expects the cervical os to appear:

A. Smooth and circular

6. Visual acuity is assessed with:

A. Snellen eye chart

10. Which of the following would be a normal sensitivity to pressure for the testes?

A. Somewhat sensitive

10. A patient states that he has frothy, foul-smelling stools that float on the surface of the water in the toilet bowl. What type of stool is this patient describing.

A. Steatorrhea

6. The bulge sign is a test for:

A. Swelling in the supra-patellar punch

14. A 65-year-old man has noticed a change in his personality and his ability to understand. He also cries and becomes angry very easily.

A. The cerebral lobe responsible for these behaviors is the frontal Lobe.

4. The largest salivary gland is located:

A. within the cheeks in front of the ear.

10. Pulse pressure is described as:

A. The difference between the systolic and diastolic pressure.

11. A nasal polyp is distinguished from the nasal turbinates by 3 of the following. Which reason is false?

A. The polyp is (not) highly vascular.

1. The manubriosternal angle is:

A. the articulation of the manubrium and the body of the sternum

13. When using the ophthalmoscope, the interruption of the red reflex occurs when:

A. There is opacity in the cornea or lens.

15. You are assessing an African-American patient and note a flat, 3-cm, non-tender, grayish-white lesion on the left buccal mucosa. Which of the following is most likely?

A. This lesion is leukoedema and is common in darkly pigmented persons.

13. A newborn infant has a dark green stool 2 days after birth. How should you interpret this?

A. This type of stool would indicate anal patency.

18. Supination:

A. Turning the forearm so that the palm is up.

microaneruysom

abnormal findings of round red dots on the ocular fundus that are localized dilations of small vessels

13. Why is it important to match the appropriate size of blood pressure cuff to the person's arm and shape and not the person's age?

A. Using a cuff that is too narrow will give a false reading that is high.

What term refers to a linear skin lesion that runs along a nerve route? A. Zosteriform B. Annular C. Dermatome D. Shingles

A. Zosteriform

Pleural effusion

abnormal fluid between the layers of the pleura

An older adult with new-onset delirium usually has: A. a short attention span. B. trouble naming common objects. C. outbursts of violent behavior. D. vertigo.

A. a short attention span. RATIONALE: Delirium manifests as an acute change in cognition that affects the domain of attention. A person with Alzheimer disease may have alterations in word finding and naming objects in addition to memory problems. A patient with posttraumatic stress disorder may exhibit outbursts of violent behavior. Vertigo is rotational spinning caused by neurologic disease in the vestibular apparatus in the ear or in the vestibular nuclei in the brainstem.

22. A. anteroposterior - transverse diameter

A. anteroposterior - transverse diameter

1. The medical record indicates that a person has an injury to Broca's area. When meeting this person, you expect:

A. difficulty speaking

5. You are providing health promotion teaching for a 40-year-old woman. What is the current recommendation for women 40 years of age and older for breast cancer screening with mammography?

A. every year

Assessment of the social domain includes: A. family relationships B. ability to cook meals C. ability to balance the checkbook and pay bills D. hazards found in the home

A. family relationships

3. the midline fold of tissue that connects the tongue to the floor of the mouth.

A. frenulum

2. Sensorineural hearing loss may be related to:

A. gradual nerve degeneration

11. The congenital displacement of the urethral meatus to the inferior surface of the penis is:

A. hypospadias

Select the statement that is true regarding headaches A. may be precipitated by alcohol and daytime napping B. usual occurrence is two per month each lasting 1 to 3 days C. characterized by throbbing D. tend to be supraorbital, retro-orbital or frontotemporal

A. may be precipitated by alcohol and daytime napping

26. Hypoglossal:

A. movement of the tongue

When you perform a functional assessment of an older patient, which is most appropriate? A. observe the patient's ability to perform the tasks B. ask the patient's wife or husband how he or she does when performing tasks C. review the medical record for information on the patient's abilities D. ask the patient's physician for information on the patient's abilities

A. observe the patient's ability to perform the tasks

13. When assessing the tongue, you should:

A. palpate the U-shaped area under the tongue

An older adult's advanced activities of daily living would include: A. recreational activities B. meal preparation C. balancing the checkbook D. self-grooming activities

A. recreational activities

List the conditions associated with parotid gland enlargment

AIDS Mumps

Subjective assessment of skin, hair & nails:

ASK THE FOLLOWING: * Past history of skin disease (hives, allergies, psoriasis, eczema) * Change in pigmentation * Change in mole * Excessive dryness or moisture * Pruritus * Excessive bruising * Rash or lesion * Medications * Hair loss * Change in nails * Environmental or occupational hazards * Self-care behaviors * FOR INFANT OR CHILD: ask about birthmarks, changes in skin color, diaper rash

12. Distinguish abdominal wall masses from intra-abdominal masses.

Abdominal wall masses are most commonly umbilical, epigastric, incisional, or spigelian hernias; benign and malignant neoplasms; infections; and hematomas. An intra-abdominal mass is any localized enlargement or swelling in the human abdomen.

Amenorrhea

Absence of menstruation; termed secondary amenorrhea when menstruation has begun and then stops; most commonly due to pregnancy.

Adenexa

Accessory organs of the uterus such as ovaries and fallopian tubes

Describe the charecteristics of lymph nodes often associated with: Acute infection, Chronic Inflammation, Cancer

Acute Infection- firm bilateral- enlarged, tender Chronic Inflammation- enlarged able to slightly move Cancer- hard, matted, fixed

Explain how acute and chronic pain differ in terms of nonverbal behaviors

Acute Pain: is short-term and self-limiting, often follows a predictable trajectory, and dissipates after an injury heals. Ex. incident pain is an acute pain Chronic Pain: Is a diagnosed when the pain continues for 6 months or longer. It is divided into malignant and nonmalignant.

Angina pectoris

Acute chest pain that occurs when myocardial demand exceeds its oxygen supply

Special considerations when interviewing an older adult.

Address the person by last name and title. -Allow more time. -Adjust pace comfortably for patient. -Keep in mind physical limitations. -Touch is very important.

Gender-

After puberty, females usually have a lower BP reading than males. After menopause females have higher BP

arterial deficit in the lower extremities. After raising the legs 12 inches off the table and then having the person sit up and dangle the leg, the color should return in.

After raising the legs 12 inches off the table and then having the person sit up and dangle the leg, the color should return in 10 seconds or less.

13. List factors that affect blood pressure.

Age-normally a gradual rise occurs through childhood to adult years

Differentiate between air conduction and bone conduction.

Air Conduction, AC: the normal pathway of hearing; it is the most efficient. Bone Conduction, BC: Alternate route of hearing; bone of the skull vibrate; these vibrations are transmitted directly to the inner ear and to cranial nerve VIII.

When assessing mental status in children, what statement is true?

All aspects of mental status in children are interrelated.

Name the borders of two regions in the neck

Anterior triangle- in Front between the sternomastoid/ up and lower mandible Posterior Triangle - Trapezoid muscle base along the clavicle.

For older adults postoperative patients, poorly controlled acute pain places them at higher risk for: A. Atelectasis B. Increased myocardial oxygen demand C. Impaired wound healing D. All the above

All of the above

What would be included in the database for a new patient admission to a surgical unit?

All subjective and objective, and data gathered from a patient and the results of any laboratory or diagnostic studies completed

Trochlear CN4

Allows the eye to look directly down towards the lips; superior oblique muscle.

Ocular motor CN3

Allows the eye to look up, down and center towards the nose; superior, inferior, medial rectus & inferior oblique muscles.

Abducens CN6

Allows the eyes to look sideways towards the ears; lateral rectus muscle.

STROKE VOLUME -

Amount of blood pumped out of the heart with each heartbeat

17. Define preload and afterload.

Amount ventricular muscle is stretched at the end of diastole prior to contraction. Afterload is the opposing pressure the ventricle must generate to open aortic valve against the higher aortic pressure.

summation gallop

abnormal mid-diastolic heart sound heard when both the pathologic S3 and S4 are present

Goiter

An enlargement / tumor of the thyroid gland caused by hypothyroidism (can be brought on by a lack of iodine in the body)

Otoscope

An instrument that illuminates the ear canal, enabling the examiner to look at the ear canal and tympanic membrane.

Ophthalmoscope

An instrument that illuminates the internal eye structures, enabling the examiner to look through the pupil at the fundus/background of the eye.

11b. Fill in the labels indicated in the following illustrations (male inguinal anatomy).

Anterior superior iliac spine, inguinal ligament, Inguinal lymph node, Inguinal canal, External ring, Fascia over symphysis pubis, Femoral artery, Femoral vein, Femoral canal, Spermatic cord, Cremaster muscle, Internal ring, Transverse abdominis muscle, Internal oblique muscle, External oblique muscle.

neuropathic pain

abnormal processing of pain message; burning, shooting in nature

16. E. 3 to 4 cm above the inner third of the clavicles

Apex

Although a full mental status examination may not be required, you must be aware of the 4 main headings of the assessment while performing the interview of the physicalexamination. These Headings are:

Appearance, Behavior, Cognition, and Thought processes, or A, B, C, T

Frontal lobe

Area Concerned with personality, behavior, emotions, and intellectual function.

Fovea centalis

Area inside the macula with the sharpest and keenest vision.

macula

Area of sharpest vision containing the fovea centralis; round darker area of the ocular fundus that mediates vision only from the central visual field.

optic disc

Area where fibers from the retina converge to form the optic nerve; area of ocular fundus where blood vessels exit and enter.

Squamous cell carcinoma

Arise from actinic keratosis or de novo. Erythematous scaly patch with sharp margins, >1cm. Develops central ulcer and surrounding erythema. Usually on hands or head, areas exposed to UV radiation. Grows rapidly.

Body Build-Proportions are:

Arm span equals height, Body length from crown to pubis roughly equal to length from pubis to sole of foot

12. Compare the characteristics of leg ulcers associated with arterial insufficiency with ulcers with venous insufficiency.

Arterial deficit ulcers occur on tips of toes, metatarsal heads, and lateral malleoli. Venus deficit is in the medial malleolus and lower leg; uneven edges superficial ulcer base, granulation tissue, beefy red to yellow, bleeding, may or may not be painful.

Recall questions for an initial pain assessment.

Ask the patient to answer 8 questions concerning location, duration, quality, intensity, and aggravating/relieving factors

You are preparing the discharge plan for a Patient with aphasia. What assessment should you include in the plan?

Ask the patient to demonstrate word comprehension by naming articles in the room or on the body as you point to them.

Open-ended questions

Asks for longer narrative information; Unbiased; Leaves the person free to answer in any way

6. List the major symptom areas to assess when collecting a health history for the neurologic system.

Assess: headache, head injury, dizziness/vertigo, seizures, tremors, weakness, incoordination, numbness or tingling, difficulty swallowing, difficulty swallowing, difficulty speaking, significant past history (stroke, spinal cord injury, meningitis), and environmental/occupational hazards

Wernicke's area

Associated with language comprehension. When damaged, receptive aphasia results meaning the person hears sound, but it has no meaning. Ex. Like hearing a foreign language

One of the critical thinking skills is identifying assumptions. Explain how the following statement contains an assumption. How would you get the facts in this situation? "Ellen, you have to break up with your boyfriend. HE is too rough with you. He is no good for you."

Assuming boyfriend is the problem -Interview Ellen and friends, do complete check up

6. For serial weight measurement, what time of the day would you instruct the person to have weight measured?

At the same time each day

peripheral arterial insufficiency include:

Atrophic skin changes; thin, shiny skin with loss of hair

Breast quadrants

Axillary tail of spence, Upper outer quad, Upper inner quad, Lower inner quad, Lower outer quad.

hernia

abnormal protrusion of bowel through weakening in abdominal musculature

15. A patient has soft, moist, fleshy, painless papules around the anus. The examiner suspects this condition is:

B. HPV

6. You are auscultating breath sounds on a patient. Which of the following best describes how to proceed?

B. How the diaphragm of the stethoscope against the chest wall; listen to one full respiration in each location, being sure to do side to side comparisons.

Herpes zoster: A. caused by bacteria B. lesion on only one side of body; does not cross midline C. has absence of pain or edema D. forms pustular, umbilicated lesions

B. "shingles". Is a lesion on only one side of the body - does not cross the midline

For a health assessment, which assessment technique will you use first?

B. Inspection

9. The cremasteric response:

B. Is positive when the ipsilateral testicle elevates on stroking of the inner aspect of the thigh.

2. Which population has the highest incident of benign prostatic hypertrophy (BPH)?

B. African Americans.

15. The nurse is conducting a health fair for older adults. Which statement is true regarding vital sign measurements in aging adults?

B. An increased respiratory rate and a shallower inspiratory phase are possible findings.

7. Select the best description of an accurate assessment for a patient's pulse.

B. Begin counting with zero; count for 30 seconds.

13. Flexion:

B. Bending a limb at a joint.

12. A patient is known to be blind in the left eye. What happens to the pupils when the right eye is illuminated by a penlight beam?

B. Both pupils constrict Consensual pupil constriction

7. Normal stool is described as:

B. Brown in color and soft in consistency.

20. Posterior apex

B. C7

10. In placing a finger on either side of the cervix and moving it side to side, you are assessing.

B. Cervical motion tenderness.

6. During an inspection of a patient's nares, a deviates septum is noted. What should you do next?

B. Document the deviation in the medical record in case the person needs to be suctioned.

4. Atrial systole occurs:

B. During ventricular diastole

12. The Glasgow Coma Scale, GCS, is divided into three areas; including:

B. Eye opening, motor response to stimuli, and verbal response.

10. When assessing an infant, the examiner completes the Ortolani maneuver by:

B. Gently lifting and abducting the infant's flexed knees while palpating the greater trochanter with the fingers.

14. One of the causes of visual impairment in aging adults includes:

B. Glaucoma

7. Select the best description of bronchovesicular breath sounds.

B. Moderate-pitched, inspiration equal to expiration.

2. Select the correct description of the left lung.

B. Narrower than the right lung with two lobes.

1. The palpebral fissure is

B. Open space between the eyelids

11. Which of these is the correct technique to assess patency of the anal sphincter?

B. Palpate the anus with a gloved finger to elicit sphincter control.

A patient requests to be discharged to home instead of a rehabilitation hospital after a hip fracture. Which of the following is true about the difference between home care and hospital care? A. Home care is more expensive than hospitalization. B. Patients have less risk for infection in the home setting. C. Patients have been shown to recover more slowly at home than in the hospital. D. Physical therapy is available only in the hospital setting.

B. Patients have less risk for infection in the home setting. RATIONALE: Older adults may avoid the risk of infection exposure when at home. Home care is less expensive than hospitalization. Older adults have been shown to recover more quickly when at home than when placed in an institution. Home care services include skilled nursing care; primary care; physical, occupational, and speech therapy; social work; nutrition; case management; assistance with activities of daily living; and some durable medical equipment.

7. To insert the speculum as comfortably as possible, the examiner:

B. Presses the introitus down with one hand and inserts the blades obliquely with the other.

3. You assess a patient who reports a cough. The characteristic timing of the cough of chronic bronchitis is described as:

B. Productive cough for at least 3 months of the year for 2 consecutive years.

12. Which is a structure that secretes a thin, milky alkaline fluid to enhance the viability of sperm?

B. Prostate gland

2. Pronation and supination of the hand and forearm are the result of the articulation of the:

B. Radius and ulna

10. Conjunctivitis is associated with:

B. Reddened conjunctiva

5. A 70 -year-old woman reports dry mouth. The most frequent cause of this is:

B. Related to medications she may be taking.

2. description of tricuspid valve.

B. Right atrioventricular valve

10. Senile tremors may resemble Parkinsonism, except that senile tremors do not include:

B. Rigidity and weakness of voluntary movement.

11. To use the rechnique of egophony, ask the patient to:

B. Say "eeeeeee" each time the stethoscope is moved.

3. The examiner records "positive consensual light reflex" This indicates:

B. Simultaneous constriction of the other pupil when one eye is exposed to bright light

7. To elicit the Babinski reflex:

B. Stroke the lateral aspect of the sole of the foot from heel to across the ball.

The best description of the pitch of a sound wave obtained by percussion is:

B. The number of vibrations per second

13. What does the notation in a health record indicating the patient is a "G2 P3 Ab0" mean?

B. The woman has been pregnant twice with 3 children; twins and another child, and all her children living.

15. During the examination of a 30-year-old woman, she asks about "the 2 large moles" that are below her left breast. After examining the area, how do you respond?

B. This is a normal finding

3. An 18-month-old child is brought into the clinic for a health screening visit. To assess the height of the child:

B. Use a horizontal measuring board.

16. Optic:

B. Vision

11. You are assessing a patient's tympanic membrane and suspect an infection of acute purulent otitis media. Which of the following findings supports this?

B. absent light reflex, reddened drum, bulging drum

14. A patient with a head injury has clear, watery drainage from the ear. How should you proceed?

B. assess for the presence of glucose in the drainage

The Get Up and Go Test would be used to: A. determine a patient's ability to get dressed without assistance. B. assess functional activity of the patient along with safety determination. C. assess swallowing status of the patient. D. assess adults with dementia.

B. assess functional activity of the patient along with safety determination. RATIONALE: The Get Up and Go Test is a reliable and valid test used to assess functional ability and safety aspects. Determination of functional assessment would be able to determine if the individual could get dressed without assistance. This test would not be used to evaluate a patient's swallowing status. The Direct Assessment of Functional Abilities assesses adults with dementia.

14. Which is the first physical change associated with puberty in girls?

B. breast bud development

An older adult has had surgery for a fractured hip and has a history of dementia. You should keep in mind that older adults with cognitive impairment: A. experience less pain B. can provide a self-report of pain C. cannot be relied on to self-report pain D. will not express pain sensations

B. can provide a self-report of pain

Skin turgor is assessed by picking up a large fold of skin on the anterior chest under the clavicle. This is done to determine the presence of: A. edema B. dehydration C. vitiligio D. scleroderma

B. dehydration

6. Discuss the significance of a barrel chest.

Barrel chest: Note equal anteroposterior-to-transverse diameter and that ribs are horizontal instead of the normal downward slope. This is associated with normal aging and also with chronic emphysema and asthma as a result of hyperinflation of lungs.

Altered cognition in older adults is commonly attributed to: A. an infection or injury. B. dementia, delirium, or depression. C. the normal aging process. D. medication side effects.

B. dementia, delirium, or depression. RATIONALE: Altered cognition in older adults is commonly attributed to three disorders: dementia, delirium, and depression.

The best description of the secretion of the eccrine glands: A. thick, milky B. dilute saline solution C. protective lipid substance D. keratin

B. dilute saline solution

7. A 20-year-old man has indicated that he does not perform a testicular self-examination. One of the facts that should be shared with him is that testicular cancer, although rare, does happen:

B. in men ages 15 to 34

9. During a beast examination you detect a mass. Which of the following is most consistent with cancer rather than benign breast disease?

B. irregular, poorly defined, fixed

The Lawton IADL instrument is described by which of the following? A. the nurse uses direct observation to implement this tool B. it is designed as a self-report measure of performance rather than ability C. it is not useful in the acute hospital setting D. it is best used for those residing in an institutional setting

B. it is designed as a self-report measure of performance rather than ability

8. A 2-year-old child comes to the clinic for a health examination. A common finding for this age group is:

B. lordosis

9. The sensation of vertigo may indicate:

B. pathology in the semicircular canals

9. Select the best description of the uterus.

B. pear-shaped, thick-walled organ flattened anteroposteriorly.

12. An adhesion of the prepuse to the head of the penis, making it impossible to retract, is:

B. phimosis

4. During the otoscopic examination of a child younger than 3 years, the examiner:

B. pulls the pinna down

5. During a neurologic examination, the tendon reflex fails to appear. Before striking the tendon again, you use the technique of:

B. reinforcement

When using the various instruments to assess an older person's activities of daily living (ADLs), remember that a disadvantage of these instruments includes: A. the reliability of the tools B. self or proxy report of functional activities C. lack of confidentiality during the assessment D. insufficient detail about the deficiencies identified

B. self or proxy report of functional activities

14. What problems are associated with smoking and the use of oral contraceptives?

B. thrombophlebitis and pulmonary emboli.

7. When assessing a patient's ear with an otoscope, the patient's head should be positioned:

B. tilted away from the examiner

2. What is the most common site of breast tumors?

B. upper outer quadrant

When completing a spiritual assessment, you should: A. use "yes" and "no" questions as the foundation for future dialogue B. use open-ended questions to help the patient understand potential coping mechanisms C. try to complete this assessment as soon as possible after meeting the patient D. wait until a member of the clergy can be involved in the assessment

B. use open-ended questions to help the patient understand potential coping mechanisms

The Katz Index of Independence in ADL would measure the functional ability to: A. clean the house and take out the garbage. B. wash the face and hands and comb hair. C. pay the electric and telephone bills. D. do laundry and put away the clothes.

B. wash the face and hands and comb hair. RATIONALE: The Katz Index of Independence in ADL is a functional assessment of a person's ability to complete activities of daily living (e.g., eating/feeding, bathing, grooming, dressing, toileting, walking, using stairs, and transferring). Cleaning the house and taking out the garbage are instrumental activities of daily living (e.g., abilities necessary for independent community living). Paying bills is an instrumental activity of daily living. Doing the laundry and putting away clothes are instrumental activities of daily living.

Exercise-

BP is elevated

Stress-

BP is elevated

Weight-

BP is higher in obese persons

List 5 points to consider when using an interpreter during an interview

Be aware of differences from patient and interpreter Plan your words Be patient Longer than expected Avoid using family (children)

12. Define and describe the relationships among the terms blood pressure, systolic pressure. Diastolic pressure, pulse pressure, and mean arterial pressure.

Blood Pressure - Force of blood pushing against the side of the vessel wall

Bruit

Blowing, swooshing sound heard through the stethoscope over an area of abnormal blood flow

What is Magsicoreligions causation of illness?

Belief in the struggle between good and evil is reflected in the regulation of health and illness

4. List the two endpieces of the stethoscope and the conditions for which each is best suited.

Bell - best for soft, low-pitched sounds: extra heart sounds or murmurs: hold lightly Diaphram - used most often; best for high-pitched sounds: breath, bowel, normal heart sounds; hold firmly.

8. Discuss pathologic changes that may occur in the breast:

Benign breast disease Abscess Acute mastitis Fibroadenoma Cancer Paget disease

12. List the 3 areas of assessment on the Glasgow Coma Scale.

Best eye: opening response: spontaneously, to speech, to pain, no response Best motor response: obeys, verbal command, localizes pain, flexion-withdrawal, flexion-abnormal, extension-abnormal, no response. Best verbal response: conversation-confused, speech, sounds, no response A fully normal person has a score of 15. 7 or less reflects a coma.

S1

Best heard at the apex of the heart; coincides with the pulse in the carotid artery and coincides with the R wave if the patient is on an ECG monitor.

Infant heart location

Between rib 2 and the 4th intercostal space, up to age 1.

Child heart location

Between rib 3 and the 5th intercostal space from toddlerhood to age 7

10. State the vertebral level whose intactness is accessed when eliciting each of these reflexes:

Biceps reflex: C5-C6 - blow to biceps tendon triceps reflex: (C5-C6)--blow to biceps tendon quadriceps reflex: (C2-C4)--(knee jerk) strike tendon just below patella Achilles reflex: (C5-S2): ankle jerk

External auditory canal

Bridges the pinna to the eardrum.

Facial characteristics of hyperthyroidism

Bulging eyeballs goiter

13. When assessing hearing acuity in a 6-month-old, the examiner would:

C. watch for head turning when saying the child's name

11. Which woman should not be referred to a physician for further evaluation?

C. A 25 year old with asymmetric breasts and inversion of nipples since adolescence.

Identify the blood vessel that runs diagonally across the sternomastoid muscle A Temporal artery B Carotid artery C External jugular vein D Internal jugular vein

C External jugular vein

The capillary beds should refill after being depressed in: A. <1 second B. >2 seconds C. 1-2 seconds D. time is not significant as long as color returns

C. 1 - 2 seconds

You examine nail beds for clubbing. The normal angle between the nail base and the nails is: A. 60 degrees B. 100 degrees C. 160 degrees D. 180 degrees

C. 160 degrees

3. Select the best description of the anal canal.

C. 3.8cm long outlet of the gastrointestinal tract

11. A patient has blurred peripheral vision. You suspect glaucoma and assess the visual fields. A patient with normal vision would see your moving finger temporarily at:

C. 90 degrees

Risk factors that may lead to skin disease and breakdown include: A. Loss of protective cushioning of the dermal skin layer B. Decreased vascular fragility C. A lifetime of environmental trauma D. Increased thickness of the skin

C. A lifetime of environmental trauma

Which would be included in a holistic model of assessment A. Nursing goals for the patient B. Anticipation growth and development patterns C. A patient's perception of his or her health status D.The nurses perception of the disease related to the patient

C. A patient's perception of his or her health status

When performing indirect percussion, the stationary finger is struck:

C. At the distal interphalangeal joint

1. The examiner is going to inspect and palpate for a hernia. During this examination, the man is instructed to:

C. Bear down when the examiner's fingers are at the inguinal canal.

5. Which changes regarding height and weight occur during the 80s and 90s?

C. Both decrease

9. A positive Phalen test and Tinel sign are found in a patient with:

C. Carpal tunnel syndrome

9. On examining a patient's nails, you will note that the angle of the nail base is >160 degrees and that the nail base feels spongy to palpation. These findings are consistent with:

C. Congenital heart disease and COPD

12. Mean arterial pressure is:

C. Diastolic pressure plus one third of the pulse pressure.

8. Which symptoms suggest benign prostatic hypertrophy?

C. Difficulty initiating urination and week stream.

2. You are assessing a patient's gait. What do you expect to find?

C. Gait is as wide as the shoulder width.

1. precordium

C. The area on the anterior chest overlying the heart and great vessels.

When the examiner enters the examining room, the infant patient is asleep. Which assessment should the examiner perform next?

C. Heart, lung, and abdomen

8. Before withdrawing the speculum, the examiner swabs the cervix with a swab soaked in acetic acid. This examination is done to assess for:

C. Human papillomavirus.

9. The 6 eye muscles that control eye movement are Innervation by cranial nerves:

C. III, IV, VI

6. Which is true regarding the bulbouretral gland?

C. It can be palpated during an examination of a male patient.

4. A 70-year old woman has come for a health examination. Which of the following is a common age-related change in the curvature of the spinal column.

C. Kyphosis

20. Abducens:

C. Lateral movement of the eye

16. What is the cause of the red reflex?

C. Light reflecting from the retina.

7. The cover test used to assess for:

C. Muscle weakness

12. When examining for tactile fremitus, it is important to:

C. Palpate the chest symmetrically.

5. A 64-year-old man has come for a health examination. A normal age-related change in the scrotum would be.

C. Pendulous scrotum.

5. Absence of diaphragmatic excursion occurs with:

C. Pleural effusion or atelectasis of the lower lobe.

3. The function of the pulmonic valve

C. Protects the orifice between the right ventricular and the pulmonary artery.

During the evaluation phase of the nursing process, which action would be included A.Validating the nurses diagnosis B. Establishing priorities related to the patient's care C. Providing information to the patient and family members D.Establishing a timeline for planning outcomes

C. Providing information to the patient and family members

14. A patient is being seen in the clinic for complaints of "fainting episodes that started last week." How should you proceed with the examination?

C. Record the blood pressure in the lying, sitting, and standing positions.

4. During the examination of an infant, use a cotton-tipped applicator to stimulate the anal sphincter. The absence of a response suggests a lesion of:

C. S2

6. You are going to inspect a female patient's breasts for retraction. The best position for this part of the examination is:

C. Sitting with hand pushing into hips

11. Patients who have Parkinson's disease usually have which of the following characteristic styles of speech?

C. Slow, monotonous.

The ophthalmoscope has 5 apertures. Which aperture is used to access the eyes of a patient with undilated pupils?

C. Small

4. Prostatic hypertrophy occurs frequently in older men. The symptoms that may indicate this problem are:

C. Straining, loss of force, and sense of residual urine.

5. You must be alert for which eye emergency symptoms?

C. Sudden onset of vision change

4. Which changes in head circumference measurements in relation to chest measurements will occur from infancy through early childhood?

C. The newborn's head will be 2 cm larger than that chest circumference, but between 6 months and 2 years, the chest size will increase and remain the way.

The sequence of an examination changes from beginning with the thorax to that of head to toe when the patient is in what age category?

C. The school-aged child

9. A patient states that he is frequently constipated, and when he has a bowel movement he has rectal bleeding and pain. He does not feel any mass at his anal opening. "Do I have hemorrhoids, or is there something else wrong with me?" The examiner completes a rectal examination and explains that:

C. The symptoms are consistent with internal hemorrhoids.

12. The examiner notes small, round, white, shiny papules on the hard palate and gums of a 2-month-old infant. What is the significance of this finding?

C. This is a normal finding called Epstein pearls.

8. The tonsils are graded as a 3+. The tonsils would be:

C. Touching the uvula.

An example of a primary lesion is a(n): A. Erosion B. Ulcer C. Urticaria D. Port-wine stain

C. Urticaria (hives)

7. The examiner measures a patients legs for length discrepancy. Which is a normal finding.

C. Within 1 cm of each other

Which is an example of a formal social support network for the aging adult? A. a neighbor who drops by with newspapers and magazines on a regular basis B. An area church that offers a weekly activity and luncheon for seniors in the neighborhood C. a home health care agency that provides weekly blood pressure screenings at the church luncheon D. a senior citizen chess club whose members hold classes at the local Boys' Club

C. a home health care agency that provides weekly blood pressure screenings at the church luncheon

An appropriate use of the caregiver strain index would be which situation? A. a daughter who is taking her older father home to live with her B. an older patient who lives alone C. a wife who has care for her husband for the past 4 years at home D. A son whose parents live in an assisted living facility

C. a wife who has care for her husband for the past 4 years at home

Which statement is true regarding an individual's functional status? A. functional status refers to one's ability to care for another person B. an older adult's functional status is usually static over time C. an older adult's functional status may vary from independence to disability D. dementia is an example of function status

C. an older adult's functional status may vary from independence to disability

The configuration for individual lesions arranged in circles or arcs, as occurs with ringworm, is called: A. linear B. clustered C. annular D. gyrate

C. annular

The "A" in the ABCDE rule stands for: A. accuracy B. appearance C. asymmetry D. attenuated

C. asymmetry SKIN SELF EXAMINATION: A - asymmetry B - border C - color D - diameter E - elevation and enlargement

When completing a health assessment of an older adult with mobility problems, the sequence should: A. begin with the physical examination followed by the health history. B. be from head to toe to prevent missing any important assessments. C. be arranged to minimize the number of position changes for the patient and the examiner. D. start with the most invasive assessments.

C. be arranged to minimize the number of position changes for the patient and the examiner. RATIONALE: If an older adult patient has limited mobility, the examiner should arrange the sequence to minimize the number of position changes for the patient. The health history should be collected before the physical examination. A head-to-toe approach may include numerous position changes and should be avoided for an older adult with mobility problems. Completing invasive assessments at the end of the examination decreases anxiety and embarrassment for an older adult patient.

6. Cerebellar function is assessed by which of the following:

C. coordination- hopping on one foot.

14. You are assessing a 75-year-old patient's oral cavity. Which of the following would most likely be present?

C. decreased ability to identify odors

Prevention and treatment of ____________ may be one of the most effective interventions aimed at reducing functional decline in an older adult. A. visual disturbances B. muscle weakness C. depression D. bladder and bowel incontinence

C. depression RATIONALE: Prevention and treatment of depression may be one of the most effective interventions aimed at reducing functional decline in an older adult.

5. For a woman, history of her mothers' health during pregnancy is important. A medication that requires frequent follow-up is:

C. diethylstilbestrol

Signs of caregiver burnout include: A. going to church every week. B. weight gain. C. headaches and epigastric pain. D. using an adult daycare facility.

C. headaches and epigastric pain. RATIONALE: Signs of possible caregiver burnout include multiple somatic complaints, increased stress and anxiety, social isolation, depression, and weight loss. Social isolation is a sign of caregiver burnout. Weight loss is a sign of caregiver burnout. Use of an adult daycare facility may prevent caregiver burnout.

26. Depression:

C. lowering a body part

Clubbing can be assessed by: A. observing for transverse ridges in the nails B. the presence of pits in the nails C. noting a change in the angle of the nail base D. palpating a rigid nail base

C. noting a change in the angle of the nail base

4. When teaching the beast self-examination, you would inform the woman that the best time to conduct breast self examination is:

C. on the 4th to 7th day of the cycle

1. Using the otoscope, the tympanic membrane is visualized. The color of the membrane is:

C. pearly gray

3. To test for stereognosis, you would:

C. place a coin in the person's hand and ask them to identify it.

8. During the examination of a 70-year-old man, you note gynecomastia. You would of supernumerary nipples that are not developed.

C. review the medications for drugs that have gynecomastia as a side effect.

A risk factor for melanoma is: A. brown eyes B. darkly pigmented skin C. skin that freckles or burns before tanning D. use of sunscreen products

C. skin that freckles or burns before tanning

4. During the examination of the genitalia of a 70-year-old woman, a normal finding would be:

C. thin and sparse pubic hair.

It is dangerous for a cognitive change to be attributed to the normal aging process because: A. cognitive change is not associated with aging. B. nurses are not trained properly to make these types of judgments. C. this may delay the diagnosis of an underlying disease process. D. the client could be saying confusing comments to avoid detection of addictions.

C. this may delay the diagnosis of an underlying disease process. RATIONALE: Cognitive impairment resulting from disease may be attributed by patients, families, and health care providers to normal changes with aging, which can delay diagnostic workup.

emphysema

COPD characterized by enlargement of alveoli distal to terminal bronchioles

Secondary skin lesions:

CRUST: thickened, dried out exudate left when vesicles or pustules burst or dry up SCALE: compact desiccated flakes of skin. dry or greasy, silvery or white, dead skin FISSURE: linear crack with abrupt edges, extends into dermis, dry or moist EROSION: scooped out, but shallow depression, superficial ULCER: deeper depression extending into dermis, irregular shape, may bleed

Pulmonary congestion symptoms

Can happen from abnormally high pressure on the left side of the heart.

Distended neck and abdominal veins

Can happen from abnormally high pressure on the right side of the heart.

7. List the characteristics of vaginal discharge associated with the following conditions of vaginitis:

Candidiasis, yeast infection: thick, white, curdy, not malodorous. Trichomoniasis: pruritus, watery, and often malodorous. Bacterial vaginosis: thin, creamy, gray-white, malodorous. Chlamydia: yellow or green mucopurulent discharge Gonorrhea: purulent.

6. Describe the appearance of the cervical abnormality carcinoma.

Carcinoma: Chronic ulcer and induration are early signs of carcinoma, although the lesion may or may not show on the cervix.

12. Differentiate between carotid artery pulsation and jugular vein pulsation.

Carotid artery pulsation-Smooth rapid upstroke, a summit that is rounded and smooth and a downstroke that is more gradual and has a dicrotic notch caused by closure of the aortic valve Jugular venous pulsation-results from a back was, a waveform moving backward caused by upstream events

Metabolic syndrome

Carries increased cardiac risk and is diagnosed when a person has 3 out of five biomarkers: *Elevated blood pressure *Increased fasting plasma glucose *Elevated triglycerides *Increased waist circumference *Low high-density lipoprotein (HDL) cholesterol

middle turbinate

Catches sinus drainage, warms, humidifies and filters air inhaled into the nose.

Inferior turbinate

Catches tears from the nasolacrimal duct, warms, humidifies and filters air inhaled into the nose.

6. Describe the appearance of the cervical abnormality Chadwick sign

Chadwick sign: bluish discoloration of the cervix, vagina, and labia resulting from blood flow -Early sign of pregnancy.

Cerebellum

Coiled structure under occipital lobe that is concerned with motor coordination of voluntary movements, equilibrium and muscle tone. Does NOT initiate movement but coordinates and smoothes. Ex. the complex and quick coordination of many different muscles needed in playing piano, swimming, juggling

4. List screening measures that are recommended for early detection of colon-rectal cancer and of prostate cancer.

Colorectal: colonoscopy every 10 years, annual fecal immunochemical test Prostate: blood test for prostate-specific antigen and physical examination

Describe evidence-based nursing

Combining clinical expertise with the use of nursing research to provide the best care for patients while considering the patient's values and circumstances

Which best describes evidence based nursing practice A. Combining clinical expertise with the use of nursing research to provide the best care for patients while considering the patient's values and circumstances B.Appraising and looking at the implications of one or two articles as they relate to the culture and ethnicity of a patient C. Completing a literature search to find relevant articles that use nursing research to encourage nurses to use good practices D.Finding value based resources to justify nursing actions when working with patients of diverse cultural backgrounds

Combining clinical expertise with the use of nursing research to provide the best care for patients while considering the patient's values and circumstances

Which would be considered a risk diagnosis A. Identifying existing levels of wellness B. Evaluating previous problem and goals C. Identifying potential problems the individual may develop D.Focusing on strengths and reflecting an individual transition to higher levels of wellness

Combining clinical expertise with the use of nursing research to provide the best care for patients while considering the patient's values and circumstances

A 30 year old woman reports having persistent intense pain in her right arm related to trauma sustained from a car accident 5 months ago. She states that the slightest touch or clothing can exacerbate the pain. This report is suggestive of: A. Referred Pain B. Psychogenic Pain C. Complex Regional Pain I D. Cutaneous pain

Complex regional pain I

Focused or Problem-Centered Database

Concerns mainly on problem, one cue complex, or one body system. Mini database targeted for a short-term care in all hospital settings.

9. Select the best description of an accurate assessment of a patients' respirations.

D. Count for 30 seconds after pulse assessment.

5. Discuss the procedure and rationale for bimanual examination and list normal findings for the uterus.

Confirm INFO: Examine Uterus: • use abdominal hand to push pelvic organs closer for your intravaginal fingers to palpate. • Place your hand midway between umbilicus and symphysis; push down slowly. • Brace elbow of your pelvic arm against your hip, and keep it horizontal; woman must be relaxed. Normal Uterus findings: feels firm and smooth, with contour of fundus rounded; it softens during pregnancy; it should be freely movable and nontender.

Atresia

Congenital absence or closure of ear canal

Eustachian tub

Connects the middle ear with the nasopharynx and allows passage of air

With older adults, how should the examiner proceed with the interview?

Consider the fatigue of the older person and break the interview into shorter segments.

Consensual light reflex

Constriction of the pupil in an eye NOT receiving direct light. Right 3/1 = 3/1 Left indicates that both pupils measure 3 mm at resting state and both constrict to 1 mm in response to light.

Direct light reflex

Constriction of the pupil in an eye receiving direct light.

3. Describe 3 major motor pathways in the CNS, including the type of movements mediated by each.

Corticospinal or pyramidal tract: permits humans to have very skilled and purposeful movements. Extrapyramidal tract: originates in the motor cortex, basal ganglia brainstem, and spinal cord; these subcortical motor fibers maintain muscle tone and control body movements, especially gross automatic movements such as walking. Cerebellar system: coordinates movement, maintains equilibrium, and helps maintain posture; receives information about the position of muscles and joints, the body's equilibrium, and what kind of motor messages are being sent from the cortex to the muscles on feedback pathways.

Medulla

Continuation of spinal cord in the brain that contains all ascending and descending fiber tracts; has vital autonomic centers of respiration, heart GI function, as well as nuclei for cranial nerves VIII - XII. Pyramidal decussation happens here: crossing of motor fibers.

9. Define 2 types of adventitious breath sounds.

Crackle- Discontinuous, brief, popping sounds Wheezes- Continous musical sounds, high or low pitched, more noticeable on expiration

Rectouterine pouch

Cul-de-sac of Douglas; deep recess formed by the peritoneum between the rectum and cervix

Nail Anatomy

Cuticle Lunula Lateral nail fold Posterior nail fold Nail matrix Nail bed Nail Plate

paresthesia

abnormal sensation, burning, numbness, tingling, prickling, crawling, skin sensation

1. The reservoirs for storing milk in the breast are:

D lactiferous sinuses

While assessing a man for allergies, he states he is allergic to penicillin. Which response is best?

D. "Please describe what happens to you when you take penicillin."

Which is an example of objective data A. Patient history of allergies B.Patient use of medications at home C.Last menstrual, One month ago D. 2-X5 cm Scar present on the right lower forearm

D. 2-X5 cm Scar present on the right lower forearm

6. Darwin's tubercle indicates:

D. A congenital, painless nodule at the helix.

Which would be included in the database for a new patient admission to a surgical unit A. All subjective and Objective data gathered by the health practitioner from the patient B.All objective data obtained from the patient through inspection, percussion, palpation, and auscultation C. A summary of a patient's record, including laboratory studies D. All subjective and objective, and the data gathered from the patient and the results of laboratory and diagnostic studies completed

D. All subjective and objective, and the data gathered from the patient and the results of laboratory and diagnostic studies completed

3. Anterior and posterior stability are provided to the knee joint by the:

D. Anterior and posterior cruciate ligaments

3. H.T. has come to the clinic for a follow-up visit. Six months ago, he started a new medication that may cause impotence as a side effect; therefore medication classes explored by the nurse are:

D. Antihypertensives.

1. Vaginal lubrication is provided during intercourse by:

D. Bartholin glands

10. During the examination of the breasts of a pregnant woman, you would expect to find:

D. Blue vascular pattern over both breasts.

11. Hematopoiesis takes place in which of the following?

D. Bone marrow

3. A woman has come for an examination because of a missed menstrual period and a positive home pregnancy test. Examination reveals a cervix that appears cyanotic. This is referred to as the:

D. Chadwick sign.

Which of the following would be an indication that an older adult should stop driving a vehicle? A. Taking insulin to control type 2 diabetes mellitus B. Difficulty walking and getting in and out of the vehicle C. A pacemaker placed 2 months ago for complete heart block D. Difficulty checking over the shoulder when backing up or changing lanes

D. Difficulty checking over the shoulder when backing up or changing lanes RATIONALE: The American Association of Retired Persons has developed warning signs for when to stop driving. One of the warning signs is difficulty turning around to check over the shoulder while backing up or changing lanes. Type 2 diabetes mellitus and taking insulin are not indications to stop driving a vehicle. Mobility problems (e.g., difficulty walking or getting in or out of a vehicle) are not indications to stop driving a vehicle. Having a pacemaker is not an indication to stop driving a vehicle.

5. Which finding in the prostate gland suggests prostate cancer?

D. Diffuse hardness

When taking the health history, the pt. c/o pruritus. What is a common cause of this symptom? A. Excessive bruising B. Hyperpigmentation C. Melasma D. Drug reactions

D. Drug reactions

12. You are teaching a parent of an infant the health promotion activities to reduce the risk for acute otitis media. Which would you include in the teaching plan?

D. Eliminating smoke in the house and car

A scooped out, shallow depression in the skin is called a(n): A. Ulcer B. Excoriation C. Fissure D. Erosion

D. Erosion

6. During palpation of the testes, the normal finding would be:

D. Firm, rubbery, and smooth.

1. During the assessment of the spine, the patient would be asked to:

D. Flex, extend, abduct, and rotate.

22. Acustic:

D. Hearing and equilibrium

7. Oral malignancies are most likely to develop:

D. In the mucosal "gutter" under the tongue.

12. Any lump found in the breast should be referred for further evaluation. A benign lesion will usually have 3 of the following characteristics. Which one is characteristic of a malignant lesion?

D. Irregular shape

12. Fibrous bands running directly from one bone to another that strengthen the joint and help prevent movement in undesirable directions are known as:

D. Ligaments

Identify the facial bone that articulates at a joint instead of a structure? A. Zygomatic B. Maxilla C. Nasal D. Mandible

D. Mandible

Which should be included while teaching 4th graders about the sweat glands, during a hygiene lesson? A. There are two types of sweat glands: the eccrine and the sebaceous B. The evaporation of sweat, a dilute saline solution, increases body temperature. C. Eccrine glands produce sweat and are mainly located in the axillae, anogenital area, and naval D. Newborn infants do not sweat and use compensatory mechanisms to control body temperature

D. Newborn infants do not sweat and use compensatory mechanisms to control body temperature

1. The 4 areas to consider during the general survey include:

D. Physical appearance, body structure, mobility, and behavior

3. Before examining a patients ear with the otoscope, you would palpate which for tenderness?

D. Pinna, tragus, and mastoid process

4. The thickening and yellowing of the lens due to aging is described as:

D. Senile cataract

To assess a patient's abdomen by palpation, how should the nurse proceed?

D. Start with light palpation to detect surface characteristics and to accustom the patient to being touched.

At the conclusion of the patient examination, the examiner should:

D. Summarize findings to the patient.

Which of the following would be included in an assessment of a patient's ability to perform instrumental activities of daily living? A. Balance, gait, and motor coordination B. Dressing, toileting, and using stairs C. Eating, bathing, and grooming D. Taking medications, shopping, and meal preparation

D. Taking medications, shopping, and meal preparation RATIONALE: Instrumental activities of daily living are functional abilities necessary for independent community living and include shopping, meal preparation, housekeeping, laundry, managing finances, taking medications, and using transportation. Other activities may include yard work or home maintenance and leisure activities (e.g., reading, other hobbies). Mobility or physical performance includes balance, gait, motor coordination, and endurance. Activities of daily living are tasks necessary for self-care and include eating/feeding, bathing, grooming, dressing, toileting, walking, using stairs, and transferring.

Select the symptome that is least likely to indicate a possible malignancy. A Hx of radiation Tx to head neck upper ches B. Hx of using chewing tobacco C. Hx of large alcohol consumption D. Tenderness

D. Tenderness

13. The landau reflex in the infant is seen when:

D. The baby raises his head and arches the back, as in a swan dive.

When inspecting the ear canal of a patient, the examiner chooses which speculum for the otoscope?

D. The largest that will fit

11. The examiner suspects a patient has coarctation of the aorta. Which assessment finding supports this suspicion?

D. The pressure is lower than in the arm.

14. An older man asks if he is able to father children. In the aging male, when does infertility occur.

D. There is no specific age; men may be fertile into their 80's and 90's.

Which statement best describes the purpose of a health history?

D. To provide a database of subjective information about the patient's past and present health.

17. Pronation:

D. Turning the forearm so that the palm is down.

During the assessment, which part of the hand is best for detecting vibration?

D. Ulnar surface of the hand

10. The opening of an adult's parotid gland, stensen's duct, is opposite to:

D. Upper 2nd molar

document 4+ edema of the right.

D. Very deep pitting, indentation lasts a long time.

Providing resistance while the parient shrugs the shoulders is a test of the statis of CN: A. II B.V C.IX D.XI

D. XI

Nevus is the medical term for: A. freckle B. a birthmark C. an infected hair follicle D. a mole

D. a mole

You will use which technique when assessing an older individual who has cognitive impairment? A. ask open-ended questions B. complete the entire assessment in one session C. ask the family members for information instead of the older individual D. ask simple questions that have "yes" or "no" answers

D. ask simple questions that have "yes" or "no" answers

6. A woman has come for health care reporting a thick white discharge with intense itching. These symptoms are suggestive of:

D. candidiasis

You note a lesion during an examination. The most complete description: A. raised, irregular lesion the size of a quarter, located on dorsum of left hand B. open lesion with no drainage or odor, approximately 1/4 inch in diameter C. pedunculated lesion below left scapula with consistent red color, no drainage or odor D. dark brown, raised lesion, with irregular border, on dorsum of right foot, 3 cm in size with no drainage

D. dark brown, raised lesion, with irregular border, on dorsum of right foot, 3 cm in size with no drainage

An older person is experiencing an acute change in cognition. You recognize that this disorder is: A. Alzheimer dementia B. attention deficit disorder C. depression D. delirium

D. delirium

Flattening of the angle between the nail and its base is: A. found in subacute bacterial endocarditis. B. a description of spoon-shaped nails. C. related to calcium deficiency D. described as clubbing

D. described as clubbing

Checking for skin temperature is best accomplished by using: A. palmar surface of the hands B. ventral surface of the hands C. fingertips D. dorsal surface of the hands

D. dorsal surface of the hands

2. Which sinuses can you assess through examination?

D. frontal and maxillary

9. During transillumination of a scrotum, you note a non-tender mass that transilluminates with a red glow. This finding is suggestive of:

D. hydrocele

2. The control of body temperature is located in the:

D. hypothalamus.

8. After examining a patient, you make the following notation: increased respiratory rate, chest expansion decreased on left side, dull to percussion over left lower lobe, breath sounds louder with fine lower lobe, breath sounds louder with fine crackles over left lower lobe, these findings are consistent with:

D. lobar pneumonia

10. On auscultating a patient, you note a coarse, low-pitched sound during both inspiration and expiration. This patent reports pain with breathing. These findings are consistent with:

D. pleural friction rub

Xerosis

DRY: excessive dryness. the skin loses moisture and it may crack and peel. Bathing or hand washing too frequently, especially if one is using harsh soaps, may also contribute to xeroderma. Xeroderma can also be caused by a deficiency of vitamin A, vitamin D, systemic illness, severe sunburn, or some medication

You are reviewing assessment data of a 45-year-old male patient and note pain of 8 on a scale of 10, labored breathing, and pale skin color on the electronic health record. This documentation is an example of:

Data cluster

You are reviewing assessment data of a 45-year-old male patient and noted a pain of eight on the scale of 10, labored breathing, and pale skin on the electronic health record. This documentation is example of

Data cluster

15. Explain the significance of phase I, phase IV, and phase V Korotkoff sounds during blood pressure movement. Phase V, Silence.

Decreased velocity of blood flow. Streamlined blood flow is silent. The last audible sound is diastolic pressure

8. Differentiate between light and deep palpation, and explain the purpose of each.

Deep- press 5-8cm in noting the location, size, consistency, and mobility of organs and the presence of any tenderness (occurs with inflammation of peritoneum or underlying organ), enlarged liver, enlarged spleen, enlarged kidney, and Kidney masses. light - press about 1 cm in to get an overall impression of the skin surface and superficial musculature; Muscle guarding, rigidity, large masses, tenderness.

Hypopituitary Dwarfism -

Deficiency in growth hormone in childhood results in retardation of growth below the 3rd percentile, delayed puberty, hypothyroidism, and adrenal insufficiency.

Normal age- related finding in the lower extremities of an 80-year-old woman would be: A. Crepitus B. Joint Swelling C. Diminished strength bilaterally D. Unilateral muscle atrophy

Diminished strength bilaterally

Chronic heart failue

Develops over time

Acute heart failure

Develops quickly with sudden illness.

10. Explain the position of the valves during the cardiac cycle in diastole, isometric contraction, systole, and isometric relaxation.

Diastole -av valves, i.e. tricuspid and mitral, are open; semilunar valves are closed, Systole - av valves shut/semilunar valves open

Pulse pressure -

Difference between the systolic and diastolic and reflects the stroke volume

Orthopnea

Difficulty breathing especially when laying down

dysphagia

Difficulty in swallowing

11. Contrast the anatomic course and clinical significance of these hernias:

Direct inguinal: -anatomic course- directly behind and through external inguinal ring, above inguinal ligament, rarely enters scrotum; usually painless. Is acquired weakness, heavy lifting, muscle atrophy, etc. Indirect inguinal: -anatomic course- sac herniates through internal inguinal ring; remains in canal or passes into scrotum; most common, painful, congenital or acquired Femoral: -anatomic course- through femoral ring and canal, below inguinal ligament, more often on right side; may have severe pain; more common in women. Acquired, due to increase in abdominal pressure, muscle weakness, or frequent stooping.

8. Concerning the pupillary light reflex, describe and contrast a direct light reflex with a consensual light reflex.

Direct light reflex occurs in the eye that receives the light stimulus. The consensual reflex occurs in the other eye; CN2.

Otorrhea

Discharge from the ear

Acromegaly -

Disease characterized by enlarged features, especially the face and hands, caused by hypersecretion of the pituitary hormone after puberty, when normal bone growth has stopped; most often caused by a pituitary tumor

Carotid artery palpation

Don't apply too much pressure as it slows the heart rate. Touch only one side at a time or you'll occlude blood flow to the brain or they may faint.

Achondroplastic Dwarfism -

Due to genetic disorder in converting cartilage to bone. Congenital skeletal malformation. relatively large head with frontal bossing midplace hypoplasia short stature short limbs often thoracic kyphosis, prominent lumbar lordosis, abdominal protrusion

3. Describe the effect of respiration on the heart sounds.

During inspiration, blood enters the vena cava due to decreasing thoracic pressure/increased right side volume. On the left side more blood is called to the lungs allowing the aortic to close earlier.

14. Extension:

E. Straightening a limb at a joint.

24. Vagus:

E. Talking, swallowing, and sensory information from pharynx and carotid sinus.

13. You are assessing an adolescent boy. The first physical sign of puberty is:

E. Testes enlargement

21. elliptic shape with an anteroposterior to transverse diameter in the ratio of 1:2

E. elliptic shape with an anteroposterior to transverse diameter in the ratio of 1:2

What health belief practice is associated with patients who are of American Indian heritage

Eating compatible foods in one's diet

5. List the risk factors of venous stasis.

Elderly, diabetes, obesity, peripheral vascular disease, pregnancy, smoking, varicose veins, inactivity

Late clubbing

Elevated edge of nail, with an angle >180 degrees, because of chronic obstructive pulmonary disease

Pons

Enlarged area of brainstem, containing ascending sensory and descending motor tracts. Includes the pneuotaxic and apneustic respiratory centers that coordinate with the main respiratory center in the medulla.

Describe the points to consider in preparing the physical setting for the interview (7)

Ensure privacy Refuse interruptions Remove distractions Provide lighting Reduce noise Face to face Reduce note taking

10. Contrast the physical appearance and clinical significance of these scrotal lumps:

Epididymus: enlarged and reddened scrotum -hard to distinguish from testes -WBC and bacteria in urine -Acute infection caused by prostatitis Varicocele: when standing feels soft, irregular mass posterior to and above testes; collapses when supine, refills when upright, feels like bag of worms -dilated, tortuous varicose veins in the spermatic cord cause by incompetent valves; 90% left sided Spermatocele: round, freely movable mass lying above and behind testes -retention cyst in epididymus Testicular tumor: firm nodule or harder than normal section of testicle -swelling usually occurs -most all are malignant -need biopsy to confirm -early detection is important

6. Describe the appearance of the cervical abnormality erosion.

Erosion: Cervical lips inflamed and eroded -Reddened granular surface is superficial inflammation with no ulceration. -Biopsy needed to distinguish erosion from carcinoma.

5. Discuss the procedure and rationale for bimanual examination and list normal findings for the adnexa.

Examine Adnexa: Move both hands to right to explore adnexa. Place abdominal hand on lower quadrant inside anterior iliac spine and place intravaginal fingers in lateral fornix. Push abdominal hand in and try to capture ovary. Move to left to palpate other side. Then, withdraw your hand and check secretions on fingers before discarding the glove. Normal Adnexa findings: Normal adnexal structures often are not palpable. Fallopian tube is not palpable normally; no other mass or pulsation should be felt. Often you cannot feel ovary; normally feels smooth, firm, almond shaped, and movable, sliding through fingers. Normal secretions are clear or cloudy and odorless.

5. Discuss the procedure and rationale for bimanual examination and list normal findings for the cervix.

Examine Cervix: Locate cervix in midline, often near anterior vaginal wall Palpate using palmar surface of fingers; Normal Cervix findings: location: midline, projects 1-3cm into the vagina. Size: diameter is 2.5cm. Consistency: feels smooth and firm; softens and feels velvety at 5 to 6 weeks of pregnancy, Goodell's sign. Contour: evenly rounded. Mobility: with a finger on either side, move cervix gently from side to side; normally, this produces no pain. Palpate all around fornices; the wall should feel smooth.

Reflection

Examiner response that echoes the patient's words; Repeat part of what the patient had just said

Facilitation

Examiner's response that encourages the patient to say more, to continue with the story

Interpretation

Examiner's statement that is not based on direct observation, but is based on the examiner's inference or conclusion; It links events, makes associations, or implies cause

Explanation

Examiner's statements that inform the patient; Examiner shares factual and objective information

10. Describe 3 qualities to consider when assessing the pulse.

Exercise, Emotion, Illness

Position of the heart

Extends from the 2nd to the 5th intercostal space from the right border of the sternum to the left midclavicler line.

Jugular Venous Pressure, JVP

External pulsation of the jugular vein. Measured at 45 degrees. 9 cm is normal and >4cm may indicate heart failure.

List the facial structures that should appear symmetric when inspecting the head

Eyebrows Ears Eyes Nose Mouth

15. Olfactory:

F. Smell

25. Elevation:

F. rising a body part

Differentiate abscess & furuncle

FURUNCLE: infected hair follicles ABSCESS: traumatic introduction of bacteria. Abcesses are usually large and deeper than furuncles.

List the 9 types of examiner responses that could be used during the interview

Facilitation Silence Empathy Clarification Confrontation Interpretation Explanation Summary

9b. Fill in the labels indicated in the following illustrations (pelvis).

Fallopian tube, Fimbria, Ovary, Uterus, Bladder, Symphysis pubis, Mons pubis, Clitoris, Urethral meatus, Labia minora, Labia majora, Vagina and rugae, Rectum, Anterior fornix, Cervix, Cervical Opening, Os, Rectouterine pouch, Posterior fornix, Anterior view of Adnexa, Cervix, Fondus of uterus.

Summary

Final review of what examiner understands patient has said; Condenses facts and presents a survey of how the examiner perceives the health problem or need

2. Explain the mechanism producing normal 1st and 2nd heart sounds.

First heart sound/S1 occurs with closure of the AV valves and signals the beginning of systole; usually loudest at the apex. Second heart sound/S2 occurs with closure of the semilunar valves and signals the end of systole.

Hemorrhoid

Flabby papules of skin or mucous membrane in the anal region caused by a varicose varicose vein of the hemorrhoidal plexus.

Ascites

Fluid in the peritoneal cavity due to liver dysfunction.

Ambulatory, apparently well individual who presents at outpatient clinic with a rash

Focused database

Recently placed on antihypertensive medication

Follow up database

8. Contrast the motions used to straighten the ear canal when using the otoscope with an infant versus an adult.

For infants, the pinna is pulled down and back. For adults, the pinna is pulled up and back.

Pitch

Frequency; the number of vibrations/cycles per second of a note.

Sinuses accessible for examination

Frontal over eyebrows, Maxillary over cheekbones

Sinuses

Frontal over eyebrows. Ethmoid between eyes. Maxillary over cheekbones. Sphenoid in front of ears.

17. Oculomotor:

G. Extraocular movement, pupil constriction, down and inward movement of the eyes

15. Abduction:

G. Moving a limb away from the midline of the body.

Murmur

Gentle blowing or whooshing sound that can be heard on the chest wall; best heard at the apex, apical pulse area with patient laying on left side with timing at the end of the exhale.

4. List the steps in the procedure of obtaining a Vaginal pool specimen:

Gently rub the blunt end of an Ayre spatula over the vaginal wall under and lateral to cervix. -Dip specimen into a liquid vial or wipe it on a glass slide. --If mucosa is very dry, as in post menopausal women, moisten a sterile swab with normal saline to collect specimen.

A patient presents with acute pain of the abdomen. After the initial examination, how would you proceed? A. Withhold analgesic until diagnostic testing is completed B. Give patient medications as ordered C. Withhold analgesic until pain subsides. D. Determine what type of pain it is and proceed accordingly

Give pain medication as ordered.

Extraocular muscles

Give the eye straight and rotary movements with three cranial nerves: Ocular motor CN3 Trochlear CN4 Abducens CN6

Malignant melanoma

Half rise from preexisting nevi. Usually brown, but can be tan, black, pink-red, purple, or of mixed pigmentation. Often have irregular or notched borders. May have scaling, flaking, oozing texture. Commonly located on trunk and back in males & females, legs in females, and on the palms, soles of feet and nails of blacks.

Novice Nurse

Has no experience with a specific patient population and used rules to guide performance.

When auscultating the heart, you hear a blowing swishing sound between the first and second heart sound.How do you validate this

Have another nurse validate what you've heard

5. State the method of promoting anal sphinter relaxation to aid palpation of the anus and rectum.

Have patient take a deep breath and hold it Place pad of index finger gently against anal verge Will feel sphincter tighten then relax As relaxes, as patient to exhale and flex tip of finger and slowly insert it into the anal canal in direction towards umbilicus.

You conduct an admission interview. Because you are expecting a phone call, you stand near the door. Which would be a more appropriate approach?

Have someone else answer the phone so you can sit facing the patient.

Questions asked in a Health Assessment (set A)

Have you noticed any changes? Do you have and chronic illnesses? Are you in pain now? What is your family health history? Any recent surgeries? Are you on any medications? Do you self exam? Any recent trauma?

The concept of health and healing has evolved in recent years. What is the best description of health

Health depends on an interaction of mind, body, and spirit within the environment

Third Level Priority

Health problems that don't fit into more severe categories. Such as lack of activity, rest or family coping.

What is appropriate in a cultural assessment

Health-related beliefs

BRADYCARDIA -

Heart rate fewer than 50 or 60 beats per minute in the adult (depending on agency)

TACHYCARDIA

Heart rate great than 95 beats per minute in adult

What culture would describe illness as hot and cold imbalance

Hispanic-American Heritage

Gigantism -

Hypersecretion of growth hormone from the anterior pituitary before puberty, leading to abnormal overgrowth of body tissues

Contrast hypospadias with epispadias

Hypospadias: ventral location of meatus. Epispadias: dorsal location of meatus.

Cue Recognition - 62 yo male, appears pale, diaphoretic, and anxious: Hypothesis formulation Data gathering for hypothesis testing Hypothesis evaluation

Hypothesis formulation - possible angina pectoris Data gathering for hypothesis testing - Vital signs, pulse oximetry, breath and heart sounds, assess chest pain Hypothesis evaluation - EKG and nitroglycerin dose results

What is considered a risk diagnosis

Identifying potential problems the individual may develop

Animism

Imagining that inanimate objects come alive and have human characteristics

A major characteristic of dementia is:

Impairment of short-and-long-term memory

2. Describe the size, shape, and location of the male prostate gland.

In front of the anterior wall of the rectum, 2 cm behind the symphysis pubis -2.5 cm long, 4 cm in diameter -Bilobed with round or heart shaped, lateral lobes separated by median sulcus

-Broca's area

In frontal lobe, mediates motor speech. When injured, expressive aphasia results meaning the person cannot talk. Ex. Can understand language and know what he wants to say but can't produce the language.

Raynaud phenomenon occurs:

In hands and feet as a result of exposure to cold, vibration, and stress

spleen

In the left upper quadrant, LUQ

Functional assessment

Include activities of daily living Example: bathing dressing, toileting, eating, walking, housekeeping, shopping, cooking

Comprehensive nutritional assessment

Includes dietary history and clinical information, physical examination for clinical signs, anthropometric measures, and laboratory tests.

List the barriers to evidence-based practice, both on an individual level and on an organizational level.

Individual level: nurses lack research skills in evaluating quality of research, are isolated from other colleagues knowledge aide in research, lack confidence to implement change Organizational: nurses lack time to go to library, health care institutions lack research holdings, organizational support for EBP is lacking

Elderspeak

Infantilizing and demeaning language used by a health professional when speaking to an older adult

Which statement best describes interpretation as a communication technique?

Interpretation is based on the interviewer's inference from the data that have been presented.

What would you say to someone who tells you that infants do not remember pain and that they are too little for the pain to have any damaging effect?

Infants have the same capacity for pain as adults it is a myth that infants do not remember pain studies have shown infants remember pain

Otitis media

Inflammation of the middle ear and tympanic membrane; Common in babies and children. Higher rates associated with absence of breast-feeding, second-hand smoke, daycare attendance, male sex, pacifier use, fall and winter seasons, and bottlefeeding in supine position.

Otitis Externa

Inflammation of the outer ear and ear canal

2. Describe the significance of the inguinal and femoral canal.

Inguinal canal: narrow tunnel passing obliquely between layers of abdominal muscle. Femoral canal: inferior to the inguinal ligament. Knowledge of these anatomic areas in the groin is useful because they are potential sites for a hernia.

Marfan's Syndrome -

Inherited condition that results in excessive cartilage formation at the growth plates, forming long arms and legs.

Cochlea

Inner ear structure containing the central hearing apparatus

4. List the steps in the procedure of obtaining a Endocervial specimen using cytobrush specimen:

Insert cytobrush into os. May feel pinch and scant bleeding may occur. -Rotate 720 ONE direction. -stir cytobrush in liquid vial or rotate it gently to deposit cells on a slide.

Examination sequence for the abdomen.

Inspection, auscultation, percussion, palpation

SPHYGMOMANOMETER -

Instrument for measuring arterial blood pressure

4. Describe the characteristic of the 1st heart sound and it's intensity at the apex of the heart and at the base.

Intensity depends on (1) position of AV valve at the start of systole (2) structure of the valve leaflets, and (3) how quickly pressure rises in the ventricle

Amplitude

Intensity; how loud or soft a sound is

The expert nurse differs from the movie nurse by acting without consciously thinking about the actions. This is referred to as:

Intuition

Conductive hearing loss

Involves and mechanical dysfunction of the external or middle ear: commonly caused by impacted earwax.

Esotropia

Inward turning of the eye

pulse amplitude of 3+

Is increased and full

cerebral cortex

Is the center for humans highest functions governing thought, memory, reasoning, sensation and voluntary movement.

23. Protraction:

L. Moving a body part forward and parallel to the ground

21. Facial:

L. Tasting anterior 2/3s of tongue, closing eyes.

Neuropathic pain does not adhere to typical and predictable phases.

It implies an abnormal processing of the pain message due to an injury of the nerve fibers. It is sustained on a neurochemical level that can only be identified by electromyography and nerve-conduction studies.

6. Explain the significance of a supernumerary nipple or breast.

It is a small extra nipple along the embryonic milk line.

5. Describe how an image formed on the retina compares with its actual appearance in the outside world.

It is upside down and reversed from its actual appearance.

You are assessing a 75-year-old man. What is an expected finding?

It may take him a little longer to respond but his general knowledge and abilities should not have declined.

25. Spinal:

J. Movement of trapezius and sternomastoid muscle

19. Circumduction:

J. Moving the arm in a circle around the shoulder.

During an interview with a patient diagnosed with a seizure disorder, the patient stats, "I plan to be an airline pilot." If the patient continues to have this as a career goal after teaching regarding the seizure disorders has been provided, you might question the patient's:

Judgment

18. Troclear:

K. Down and inward movement of the eye

21. Eversion:

K. Moving the sole of the foot outward at the ankle.

Conditions of the nails:

KOILONYCHIA: thin, concave, raised edges. "Spooning". Caused by iron deficiency & anemia. CLUBBING: inner edges of nail bed is elevated. PARONYCHIA: Red, swollen, tender inflammation of nail folds. BEAU'S LINE: transverse furrow or grove; extends down to nail bed SPLINTER HEMORRHAGES: red brown, linear streaks, embolic lesions ONYCHOLYSIS: fungus infection, green, thick, crumbling

patella

Kneecap

Umbo

Knob of the malleus that shows through the tympanic membrane

Describe the technique for measuring head circumference and chest circumference on an infant. To measure chest circumference of the infant:

Lay the infant flat on his or her back. Use the thumb of one hand to hold the zero mark of the tape at the middle of the sternum. Use your other hand to wrap the tape snugly under the axillary area and around the back to meet at the midsternal area. Make sure the tape is at the nipple level of the chest and that it is not twisted. Read the measurement after the infant has exhaaled or during the resting phase between respirations. Read the tape measure to the nearest 1/2 inch or 0.1 centimeters.

Diagnostic positions test

Leading eyes through the six Cardinal positions of gaze, testing cranial nerves 3,4 and 6. Can indicate muscle weakness or dysfunction of cranial nerves.

13. What is the tripod position?

Leaning forward with arms braced against their knees, chair, or bed. This gives them leverage so that their rectus abdominis, intercostal, and accessory neck muscles all can aid in expiration.

11. List the components of the neurologic recheck examination that are performed routinely on hospitalized patients monitored for neurologic deficit.

Level of consciousness: check orientation to the place and time; note the quality and the content of the verbal response: articulation, fluency, manner of thinking, and any deficit in language comprehension or production. Motor function: giving the person specific commands: lift the eyebrows, frown, bare teeth; check upper arm strength by checking hand grasps; ask person to lift hand up; check lower by having them lift their leg. Pupillary response: note the size, shape, and symmetry of both pupils: shine a light into each pupil and note the direct and consensual light reflex. Both pupils should constrict briskly. Vital signs: blood pressure is notoriously an unreliable parameters of CNS deficit.

20. Inversion:

M. Moving the sole of the foot inward at the ankle.

Skin rashes of childhood diseases:

MEASLES (Rubeola): red / purple blotchy rash, coppery, does not blanch GERMAN MEASLES (Rubella): pink, papular rash, paler than measles CHICKEN POX (Varicella): small tight vesicles, become pustules and then crust

Common variations on infant's skin:

MONGOLIAN SPOT: blue--black to purple macular area at sacrum (sometimes: buttocks, thighs, abdomen, shoulders, arms) CAF'E AU LAIT SPOT: large round, oval, patch of light brown pigmentation ERYTHEMA TOXICUM: rash on 3-4 day old newborns. Cheeks, trunk, chest, back, butt CUTIS MARMORATA: transient mottling of skin in trunk & extremities PHYSIOLOGIC JAUNDICE: 1/2 of all newborns, yellowing of skin, sclera, mucous membranes MILIA: tiny white papules or cheeks, nose, forehead, chin, caused by sebum occluding hair follicles

Thalamus

Main relay station where sensory pathways of the spinal cord, cerebellum, and brainstem form synapses on their way to the cerebral cortex.

Hypothalamus

Major respiratory center with basic vital functions: temp, appetite, sex drive, heart rate, and blood pressure control, sleep center, anterior and posterior pituitary gland regulator, and coordinator of autonomic nervous system activity and stress response.

1. Describe the proper positioning and preparation of the patient for examination.

Make sure the room is a warm temp, use good lighting & drape breast and genitals. Ask the patient if their bladder is empty. Position the patient supine, with head on a pillow, the knees bent or on a pillow, and the arms at sides or across the chest. Discourage the pt from placing arms over their head because this tenses abdominal musculature.

7. Differentiate between the female and male examination procedures and findings.

Males can have a shorter exam to detect abnormal breast tissue or gynecomastia benign breast growth.

11. List the skin characteristics expected with arterial insufficiency to the lower legs.

Malnutrition, pallor and coolness; thin, shiny atrophic skin, thick-ridged nails, loss of hair, ulcers, gangrene

Systolic pressure -

Maximum pressure felt on the artery during left ventricular contraction, or systole

screening for deep vein thrombosis

Measure the wildest point with a tape measure

Interview

Meeting between examiner and patient with the goal of gathering a complete health history

Nonverbal communication

Message conveyed through body language--posture, gestures, facial expressions, eye contact, touch, and even where you place your chair

Verbal communication

Messages sent through spoken words, vocalization, and tone of voice

19. Discuss the characteristics of an innocent or functional murmur.

Midsystolic Soft blowing - pulm area 2LCIS Short buzzing - aortic area 4LCIS Soft intensity (Grade I - III) Localised Poorly conducted Musical/vibratory in character Variable with posn/respn Not associated with other signs of heart disease

Emotions-

Momentarily rises with fear, anger and pain as a result of stimulation of the sympathetic nervous system

Intermittent claudication includes

Muscular pain brought on by exercise

22. Rotation:

N. Moving the head around a central axis.

Vibrissae

Nasal hair

While taking a history, the patient describes a burning, painful sensation that moves around the toes and bottoms of the feet. These symptoms suggest: A. Nociceptive pain B. Neuropathic Pain C. Visceral Pain D. Muscular Pain

Neuropathic pain

Identify the differences between nociceptive and neuropathic pain. Which word will people use to describe nociceptive and neuropathic pain?

Nociceptive are specialized nerve endings, design to detect painful sensations from the periphery and transmit them to the CNS. Nociceptive are located primarily within the skin, joints, CT, muscle, and thoracic, abdominal, and pelvic viscera. Neurophatic: Is pain that does not adhere to the typical and rather predictable phases in nociceptive pain. It is pain "caused" by a lesion or disease of the somatosensory nervous system

Describe characteristics of normal cervical lymph nodes during childhood

Nodes do not palpate

7. Describe the function of the lymph nodes.

Nodes filter the fluid before it is returned to the bloodstream and filter out the micro-organisms that could be harmful to the body.

anisocoria

Normal but unequal pupil size; 5% of people

Pupillary light reflex

Normal constriction of the pupils when bright light shines on the retina; sensory link to Optic CN2 and motor link to Oculomotor CN3. Normal adult size is 3 to 5 mm

4. Summarize the mechanics of respiration.

Normally, our breathing pattern changes without our awareness in response to cellular demands. This involuntary control is mediated by the respiratory centre in the brainstem (pons and medulla). The major feedback loop is humoral regulation, or the change in carbon dioxide and oxygen levels in the blood, and less, importantly, the hydrogen ion level. Normal stimulus to breathe for most of us is an increase of carbon dioxide in the blood, or hypercapnia.

2. Discuss inspection of the abdomen, including findings that should be noted.

Note: Contour, Symmetry, Umbilicus Skin, Pulsation, Hair distribution and Demeanor.

7. List and describe 3 tests of cerebellar function.

Observe as the patient walks 10 to 20 feet, turns, and returns to the starting point. Observe a heel-to-toe walk and a walk on tip-toes to assess balance. The normal gait should be smooth, rhythmic, and effortless with a coordinated opposing arm swing and smooth turns. An uncoordinated or unsteady gait are abnormal. The Romberg Test has the patient stand up with feet together and arms at the sides. Once in a stable position, ask them to close the eyes and to hold the position. Test the shallow knee bend or to hop in place, first on one leg, then the other followed by rapid alternating movements and pats on the knees with hands. The finger-to-finger test asks the client to use their index finger to touch your finger then or his own nose as the position of your finger changes. The finger-to nose test ask the patient to touch the tip of his nose with each index finger while alternating hands and increasing speed. The heel-to-shin test places the heel on the opposite knee followed by running it down the shin from the knee to the ankle.

You are performing a mental status examination. Which assessments would be most appropriate?

Observing the patient and inferring health or dysfunction.

Retina

One of the three concentric coats covering the spear of the eye. Innermost nervous layer under the choroid. Visual receptive layer of the eye that light waves are changed into nerve impulses.

Choroid

One of the three concentric coats covering the spear of the eye. Middle vascular, dark layer under the sclera.

Sclera

One of the three concentric coats covering the spear of the eye. Outermost fibrous layer seen through the bulbar conjunctiva; tough protective and white.

For what or with whom should touch be used during the interview

Only if the interviewer knows the person well

Contrast open-ended versus closed direct questions and explain the purpose of each during the interview. Give examples of both

Open-ended: -allows client to have free rein -used for narrative information -calls for long paragraph answers -elicits feelings, opinions, ideas -builds and enhances rapport -"tell me all about your headaches" Close-ended: -ask for SPECIFIC information (past medical history or during review of systems) -elicit a short, one- or two-word answer, a "yes" or "no," or as a forced choice -useful to fill in any details that were initially left out after the person's opening narrative -need to speed up interview -elicits cold facts -limits rapport and leaves interaction neutral -"are your headaches on one side or both?"

Afterload

Opposing pressure the ventricle must generate to open the aortic valve in spite of high aortic pressure.

cotton wool area

abnormal soft exudates visible as gray-white areas on the ocular fundus

Purpuric lesions

PETECHIAE: superficial capillaries, not caused by mechanical injury BRUISE: contusion, mechanical injury, hemorrhage to tissues HEMATOMA: bruise that can be felt, mechanical injury, raised, swollen

Distinguish skin lesion terms:

PRIMARY: lesion develops on previously unaltered skin SECONDARY: when a lesion changes over time, and changes because of a factor (scratching, infection)

3. List the pros and cons of circumcision of the male newborn.

PROS: -decrease risk of acquiring HIV through sexual contact, genital herpes, genital ulcers, oncogenic HPV -female partners decrease risk for HIV, bacterial vaginosis, trichomoniasis and cervical cancer -decrease incidence for infant UTI, phimosis, paraphimosis, and penile cancer CONS: -pain -bleeding -swelling -inadequate skin removal -wound infection -urinary retention

List the pros and cons to note-taking during the interview.

PROS: You don't have to rely on memory CONS: -breaks eye contact too often -shifts your attention away from person, diminish his/her sense of importance -trying to record everything a person says may cause to ask him or her to slow down, or person may slow his or her tempo to allow for you to make take notes. Either way, client's natural mode of expression is lost -impedes observation of client's nonverbal behavior -threatening to client during discussion of sensitive issues (amount of alcohol and drug use, etc.)

Primary skin lesions:

PUSTULE: Circular, elevated cavity filled with fluid or pus. MACULE: color change, flat. "Freckle" PAPULE: solid, elevated. Circumscribed PLAQUE: elevated. Wider than 1cm. NODULE: solid, elevated. hard or soft. larger than 1cm. TUMOR: firm or soft. larger than a few cm. Benign or malignant. WHEAL: superficial, raised, slightly irregular due to edema. VESICLE: elevated, contains free fluid. "blister"

positive Murphy's sign

Pain felt when taking a deep breath when the examiner's fingers are on the approximate location of the inflamed gallbladder.

Otalgia

Pain in the ear; Can be referred pain coming from teeth or oralpharynx.

8. Describe the method of testing the sensory system for pain, temperature, touch, vibration, and position.

Pain: break a tongue blade for a "sharp" and "dull" side. Lightly apply the sharp point or the dull end to the person's body in a random, unpredictable order. Ask the person to say "sharp" or "dull" depending on the sensation felt. --Abnormal findings: ---hypoalgesia- decreased pain sensation ---hyperalgesia-increased pain sensation Temperature: place the flat side of the tuning fork on the skin; its metal always feels cool. Light touch: wisp of cotton over the arms, forearms, hands, chest, thighs and legs. Ask the person to say "now" or "yes" when touch is felt. ---abnormal findings: ---hypoesthesia- decreased touch sensation ---anesthesia- absent touch sensation ---hyperesthesia- increased touch

Papanicolaou test

Pap test; painless test used to detect cervical cancer

Salivary glands

Parotid in cheeks with Stensen's duct. Sublingual, almond shaped, under tongue, multiple ducts. Submandibular, walnut sized under jaw, Wharton's duct

Ocular fundus

Part of the retina that you see with the ophthalmoscope.

Parietal lobe

Postcentral gyrus: Primary center for sensation

At which phase does the individual become aware of a painful sensation? A. Modulation B. Transduction C. Perception D. Transmission

Perception

On a patient's second day in an acute care hospital, the patient complains about the "bugs" on the bed. The bed is clean. This would be an example of altered:

Perception

3. State the rationale for performing auscultation of the abdomen before palpation or percussion.

Percussion and palpation can increase peristalsis and give a false interpretation of bowel sounds.

5. List the elements included in the inspection of respiratory system.

Posterior and lateral thorax; anterior chest

Pain issues should be anticipated in a cognitively impaired older adult with a history of: A. Constipation B. Peripheral vascular disease C. COPD D. Parkinson Disease

Peripheral vascular disease

7a. Fill in the labels indicated in the following illustrations (rectum).

Peritoneum, Anal Canal: -column, -valve, -crypt, Interphincteric groove, External sphincter, Internal sphincter, Anorectal junction, Levator ani muscles, Rectal ampula, Valves of Houston, Rectum, Rectosigmoid junction, sigmoid colon.

8. Contrast phimosis with paraphimosis

Phimosis: narrowed opening of prepuce so cannot retract foreskin. Paraphimosis: painful constriction of glands by retracted forskin.

List the significant information considered in each of the 4 areas of a GENERAL SURVEY.

Physical Appearance, Body structure, Mobility, and Behavior.

State 7 types of nonverbal behaviors

Physical appearance Posture Gestures Facial expressions Eye contact Voice Touch

Describe the technique for measuring head circumference and chest circumference on an infant. To measure head circumference of an infant:

Position the infant on the examination table or ask the parent to hold the infant. Use a thumb or finger to hold the zero mark of the tape measure against the infant's forehead just above the eyebrows. Use your other hand to bring the tape around the infant's head, just above the ears, over the occipital bone at the back of the head, and back to the forehead to move the zero mark on the tape. Pull the tape snug to compress the hair, but not too tight. Read the tape measure to the nearest 1/2 inch to 0.1 centimeters. Record the reading.

12a. Fill in the labels indicated in the following illustrations (the brain).

Plane of coronal section B Hypothalamus Pituitary Brainstem Spinal cord Medulla Cerebellum pona thalamus corpus callosum internal capsule basil ganglia hypothalamus

You are reviewing concepts related to steps in the nursing process for determining prioritization and developing patient outcomes. To what are these actions attributed?

Planning

You are reviewing concepts related to steps in the nursing process for determining prioritization and developing patient outcomes. To which are these actions attributed

Planning

6. Describe the appearance of the cervical abnormality polyp.

Polyp: Bright red, soft, pedunculated growth emerges from os. -Benign but must be determined by biopsy.

2. Describe normal posture and body build?

Posture-a person stands comfortably erect as appropriate for age

What is a priority influence on a patient's health status

Poverty

Mean arterial pressure -

Pressure-forcing blood into the tissues, averaged over the cardiac cycle. Close to diastolic pressure plus one-third of pulse pressure

CRIES is an inappropriate pain assessment tool for: A. Cognitively impaired older adults B. Children ages 2 to 8 years C. Infants D. Preterm and term neonates

Preterm and term neonates

Temporal lobe

Primary auditory receptor center with functions in hearing, taste, and smell

Geographic privacy

Private room or space with only patient and examiner present

Collaborative Problems

Problems in which the approach to treatment involves multiple disciplines.

Second Level Priority

Problems next in urgency-those requiring your prompt intervention to forestall further deterioration. EX: Mental status change, untreated medical problems, acute pain, acute urinary elimination problems, abnormal lad values, or risk of infection.

If you are unable to palpate the popliteal pulse.

Proceed with the examination. It is often impossible to palpate this pulse.

Cultural Care

Professional health care that is culturally sensitive, appropriate, and competent

Cystocele

Prolapse of urinary bladder and it's vaginal mucosa into the vagina with straining or standing

Five functions of the skin:

Protection Perception Identification Communication Temperature regulation Wound repair Absorption & excretion Production of Vit. D

7. Define pupillary light reflex, fixation and accommodation:

Pupillary reflex: Normal constriction of the pupils when bright light shines on the retina. Fixation: Direction of the eye toward an object attracting a persons attention. accommodation: adaptation of the eye for near vision by increasing the curvature of the lens.

PERRLA

Pupils Equal, Round, Reactive to Light, and Accommodating

Clarification

Questions that ask for specific information; Elicit a short, one or two word answer, a "yes" or "no", or a forced choice

Emergency Database

Rapid collection or the database, often compiled concurrently with life-saving measures in the emergency room.

Mr. B tells you "Everyone here ignores me." You respond, "Ignores you?" This technique is best described as:

Reflecting

Corneal light Reflex

Reflection of light on the corneas; should be in the exact same place on each eye.

11. Relate the qualities of normal respirations to the appropriate approach for counting them.

Relaxed, -Regular, Automatic, Silent, Count without the patient being aware

What is the difference between religion and spirituality.

Religion is characterized by identification of a higher being shaping one's destiny while spirituality reflects an individual's perception of one's life having worth or meaning

Confrontation

Response in which examiner gives honest feedback about what he or she has seen or felt after observing a certain patient action, feeling, or statement

6. Describe the function of the lymphatic system.

Retrieves excess fluid from the tissue spaces and returns it to the bloodstream. During circulation of the blood, somewhat more fluid leaves the capillaries than the veins can absorb. Without lymphatic drainage, fluid would build up in the interstitial spaces and produce edema.

11. Describe screening mammography and clinical breast examination, CBE, for the diagnosis of breast lesions.

Screening mammography starts at age 40, monthly breast self-examination (BSE), and regular clinical breast examination (CBE).

Confrontation test

Screens for a loss of peripheral vision.

Right side of the heart

Sits agains the sternum. Gets blood from the inferior and superior vena cava and pumps it into the lungs; contains the tricuspid AV valve that opens when the heart fills for diastole. The AV valves close when the heart pumps for systole making the first heart sound, S1; this prevents regurgitation of blood into the atria. Has the pulmonic semilunar valve that opens during pumps for systole allowing blood to leave the heart.

Left side of the heart

Sits against the back side of the body. Pumps oxygenated blood to the body; contains the mitral AV valve that opens when the heart fills for diastole. The AV valves close when the heart pumps for systole; this prevents regurgitation of blood into the atria. Has the aortic semilunar valve that opens during pumps for systole allowing blood to leave the heart.

6. Describe the normal physical characteristics of the prostate gland that would be assessed by palpation:

Size: 2.5 cm long, 4 cm in diameter, should not protrude more than 1 cm into rectum Shape: heart shaped with palpable central groove Surface: smooth Consistency: elastic, rubbery Mobility: slightly movable Sensitivity: nontender

Pars flaccida

Small, slack, superior section of tympanic membrane

hypoactive bowel sounds

Sounds following abdominal surgery or with inflammation of the peritoneum.

Telegraphic speech

Speech used by age 3 or 4 in which three or four word sentences contain only the essential words

3. Correlate changes in the female breast with normal developmental stages.

Stage 1: Pre-adolescenct: only small, elevated nipple Stage 2: Breast buds- small mound of breast and nipple develops; areola widens Stage 3: Breast and areola enlarge; nipple is flush with breast surface Stage 4: the areola and nipple form a secondary mound over the breast Stage 5: Mature breast: only nipple protrudes; the areola is flush with breast contour

Basal cell carcinoma

Starts as a skin-colored papule (may be deeply pigmented) with a pearly translucent top and overlying telangiectasia (broken blood vessel). Develops rounded, pearly borders with central red ulcer, or looks like a large open pore with central yellowing. This is the most common form of skin cancer. Occur on sun exposed areas of face, ears, scalp, shoulders. Grows slowly.

The major neck muscle are the:

Sternomastoid and Trapezius

Corneal light reflex

Stimulates the eyes to blink; Trigeminal CN5 carries the afferent sensation to the brain and facial CN7 carries the efferent message that stimulates a blink. Asymmetry can indicate muscle weakness or paralysis.

Cardiac output

Stroke volume x heart rate is cardiac output

Explain 4 factors that could affect a patient's response to the mental status examination but have nothing to do with mental disorders. -

Strokes, Alzheimer, Economic Status, Deliberate Fashion trend, cultural norm

A patient says she is very nervous and nauseated, and she feels like she will vomit. This data would be what type of data?

Subjective

4. Describe the components of the breast examination.

Subjective: -Breast: Pain, Lump, Discharge, Rash, Swelling, Trauma, Hx of breast dx, surgery or radiation, medications. -Axilla: tenderness, lump, swelling, rash Objective: -Inspection: General appearance, skin, lymphatic drainage areas, nipple, maneuvers for screen retration -Inspect and Palpate the Axilla -Palpate breasts -Teach breast self-examination

1. Name the 6 sets of extraocular muscles and the cranial nerve that innervates each one.

Superior Rectus- III oculomotor. Inferior Rectus- III oculomotor. Lateral Rectus- VI abducens. Medial Rectus- III oculomotor. Superior Oblique- IV trochlear. Inferior Oblique-III oculomotor.

9. Define the fourth heart sound. When it the cardiac cycle does it occur? Describe its intensity, quality, location in which it is heard, and method of auscultation.

The S4 is a ventricular filling sound. It occurs when the atria contract late in diastole. It is heard immediately before S1. This is a very soft sound, of very low pitch. You need a good bell, and you must listen for it. It is heard best at the apex, with the person in left lateral position.

8. Draw and describe swan neck deformity and boutonniere deformity in rheumatoid arthritis.

Swan-Neck: Flexion contracture of meta-carpophalangeal joint. Boutonniere deformity: the knuckle looks like being pushed through a button-hole.

1. Differentiate synovial from non-synovial joints.

Synovial joints allow sliding of opposing surfaces, cartilage covers the surface of opposing bones. Nonsynovial joints are united by fibrous tissue or cartilage and are immovable like the sutures in the skull or slightly movable like the vertebrae.

Causes of changes in skin:

TEMPERATURE: hypothermia, hyperthermia MOISTURE: diaphoresis, dehydration TEXTURE: hyperthyroidism, hypothyroidism MOBILITY: edema, scleroderma (hard skin) TURGOR: dehydration or extreme weight loss

Biomedical Model

The Western European/ Northern American tradition that views health as the absence of disease.

Flow of blood

TO THE HEART without oxygen. -Lower limbs through the inferior vena cava and upper limbs/head through the superior vena cava, -Into the right atrium, -Through the tricuspid valve, -Into the right ventricle, -through the pulmonic valve, -into the pulmonic artery, -into the lungs for oxygen. TO THE BODY with oxygen. -through pulmonic veins, -into the left atrium, -through the mitral valve, -into the left ventricle, -through the aortic valve -into the aorta, -to artery for left brachial, to artery for left carotid, to right artery that splits to the right brachial and right carotid.

Which is considered a common physiologic change that occurs with pain? A. Polyuria B. Hyperventilation C. Hyperactive Bowel Sounds D. Tachycardia

Tachycardia

What does active listening NOT include?

Taking detailed notes during the interview

Describe how you would validate the following data. The primary counselor tells you Ellen is depressed and angry about being admitted to residential treatment in the clinic.

Talk to Ellen and comfort her, explain to her why she's there

2. List the pulse sites accessible to examination.

Temporal artery, carotid artery, brachial artery, femoral artery, popliteal artery, dorsalis pedis, posterior tibial.

Ethnocentrism

Tendency to view your own way of life as the most desirable, acceptable, or best and to act superior to another culture's lifeways

palpation the sigmoid colon

Tenderness expected during abdominal palpation

9a. Fill in the labels indicated on the following illustrations (foot anatomy).

Tendo calcaneous, Achilles tendon Talus bursae Flexor muscles flexor tendon extensor tendon Synovial joint- -Capsular ligament -synovial cavity -synovial membrane -cartilage -Tibia

Select the finding that most accurately describes appearance of a patient.

Tense posture and restless activity. Clothing clean but not appropriate for the season e.g., patient wearing T-shirt and shorts in cold weather

12. List the findings that may appear during the whispered voice test and audiometric screening.

Test on the ear while masking hearing in the other to prevent sound transmission around the head

Biomedical Model

The European/American tradition that view health as an absence of disease

7. Define the 3rd heart sound. When in cardiac cycle does it occur? Describe its intensity, quality, location in which it is heard and method of auscultation.

The S3 is a ventricular filling sound. It occurs in early diastole during the rapid filling phase. Your hearing quickly accommodates to the S3, so it is best heard when you listen initially. It sounds after S2 but later than an opening snap would be. It is heard best at the apex with the bell held lightly (just enough to form a seal) and with the person in the left lateral position.

8. Differentiate a physiologic S3 from a pathologic S3.

The S3 may be normal (physiological) or abnormal (pathological). The physiological S3 is heard frequently in children and young adults; it occasionally may persist after age 40 years, especially in women. The normal S3 usually disappears when the person sits up. In adults, the S¬3 is usually abnormal. The pathological S3 is also called a ventricular gallop or an S3 gallop, and it persists when sitting up. The S3 indicates decreased compliance of the ventricles, as in heart failure.

Health

The balance of the person, both within one's being (physical, mental, and/or spiritual) and in the outside world (natural, communal, and/or metaphysical)

Adult heart location

The base (top) and apex (bottom) of the heart in adults. Between intercostal spaces 2 and 5

Religion

The belief in a divine or superhuman power or powers to be obeyed and worshiped as the creator (s) and ruler (s) of the universe; and a system of beliefs, practices, and ethical values

The Skin

The body's largest organ system. Acts as the sentry that protects the body from environmental stress and adapts to other environmental influences. THREE LAYERS: Epidermis - the thin, tough outer layer Dermis - the inner supportive layer Subcutaneous - the adipose tissue below the dermis

Assessment

The collection of data about an individual 's health state.

Assessment

The collection of data about an individual's health state

Diastolic pressure -

The elastic recoil, or resting pressure that the blood exerts constantly between each contraction

1. Identify the structures that provide transport of sperm.

The epididymus is a very coiled duct system and the main storage site. Vas Deferens approximates with arteries, veins, lymphatics, and nerves ascend along the posterior border of the testes, run through the tunnel of the inguinal canal into the abdomen. Vas deferens continues behind the bladder and joins the seminal vesicle to form the ejaculatory duct, which empties into the urethra.

Culture

The nonphysical attributes of a person - the thoughts, communications, actions, beliefs, values, and institutions of racial, ethnic, religious, or social groups

Air conduction, AC

The normal pathway of hearing; most efficient.

What can be determined when the nurse clusters data as apart of the critical-thinking process

The nurse recognizes relevant information among the data

What can be determined when the nurse clusters data as part of the critical-thinking process?

The nurse recognizes relevant information among the data

Which demonstrates a good understanding of the interview process?

The nurse spends more time listening to the patient than talking.

The nurse uses health promotion activities with a new patient. What would this focus include?

The nurse would empower the patient to choose a healthier lifestyle

While caring for a preterm infant, you are aware that: A. Inhibitory neurotransmitters are in sufficient supply y 15 weeks gestation. B. The fetus has less capacity to feel pain C. Repetitive blood draws have minimal long-term consequences. D. The preterm infant is more sensitive to painful stimuli.

The preterm infant is more sensitive to painful stimuli.

Acculturation

The process of adapting to and acquiring another culture

Socialization

The process of being raised within a culture and acquiring the characteristics of that group

1. Define the apical impulse and describe its normal location, size, and duration.

The pulsation created as the left ventricle rotates against the chest wall during systole. When visible, it occupies the 4th or 5th intercostal space, at or inside the midclavicular line. Size; normally 1 x 2 cm. Duration; short, normally occupies only first half of systole.

2. List the primary sensations mediated by the 2 major sensory pathways of the CNS.

The spinothalamic tract transmits sensations of pain, temperature, and crude or light touch. The posterior (dorsal) column allows position (proprioception) without looking resulting in knowledge of where your body parts are in space and in relation to each other. Vibration is the feeling of vibrating objects. Stereognosis is finely localized touch where you can identify familiar objects by touch (travel to thalamus).

Which mental function is the Four Unrelated Words Test intended to Test?

This test the person's ability to lay down new memories.

Quality

Timbre; a subjective difference in a sound as a result of the sound's distinctive overtones

List 8 items of information that should be communicated to the patient concerning the terms or expectations of the interview

Time and place Introductions Purpose Duration Expectations Other people Confidentiality Cost

5. List teaching points to include with the teaching of testicular self-examination.

Timing: once a month Shower: warm water relaxes the scrotal sac Examine: check for changes, report changes immediately

9. Describe the following lesions of the penis and genital area:

Tinea cruris: forms red-brown, half-moon shape with well defined borders Herpes simplex type 2: clusters of small vesicles with surrounding erythema Syphilitic chancre: small, solitary, silvery papule that erodes to a red, round or oval, superficial ulcer with a yellowish serous discharge Penile warts: soft, pointed, moist, fleshy, painless papules; may be singular or multiple in a cauliflower-like patch; color may be gray, or pale yellow

5. Explain the method for measuring leg length.

To determine if one leg is shorter than the other; Normally these measurements are equal or within 1 cm, indicating no true bone discrepancy. True leg lengths are equal, but apparent leg lengths unequal - this condition occurs with pelvic obliquity or adduction or flexion deformity in the hip.

7. Describe the appearance of a torus palatinus, and explain it's significance.

Torus palatinus is a normal variation, a nodular bony ridge down the middle of the hard palate. It is a benign growth that occurs after puberty and is a more common finding in Native Americans, Inuits, and Asians.

3. List the structures that compose the respiratory dead space.

Trachea & Bronchi

Conjunctiva

Transparent protective covering over the exposed part of the eye.

Presbycusis

Type of hearing loss involving gradual sensorineural deficit caused by nerve degradation in the inner ear; starts in the in the 50s age group. Starts with the loss of high frequency sounds such as consonants in speech.

Proficient Nurse

Understands a patient situation as a whole rather than a list of tasks.

Recommended weight gain during pregnancy

Underweight mother 28 - 40lbs. Normal weight 25 - 35lbs. Overweight 15 - 25lbs. Obese 11 - 20lbs.

7. Discuss the rationale for making certain that the testes have descended in the male infant.

Undescended testes have a histologic change by 6 years, causing decreased spermatogenesis and infertility. Increased risk for testicular cancer.

Pain in the aging adult is considered to be: A. Part of the normal degenerative process B. Perceives to lesser degree C. An expected finding D. Unrelated to the aging process

Unrelated to the aging process

Mr. Quinn tells you his weight this morning on the clinic scale was 165lbs . How do you evaluate this

Validate by weighing Mr. Quinn on scale

4. Differentiate an upper motor neuron from a lower motor neuron.

Upper motor neuron: located in the CNS, they are a complex of descending motor fibers that can influence or modify the lower motor neurons. They convey impulses from motor areas of the cerebral cortex to the lower motor neurons in the anterior horn. Lower motor neuron: located in the PNS, the cell body of the lower motor neuron is located in the anterior gray column of the spinal cord, with nerve fiber extending from to the muscle. Many neural signals spark here providing the final direct contact with muscles.

11a. Fill in the labels indicated in the following illustrations (male genitilia).

Ureters, Bladder, Symphysis pubis, Urethra, Frenulum, Glans penis, Rugae, Scrotum, Scrotal skin lined with dartos fascia, Cremaster muscle, Tunica vaginalis, Testis, Epididymis, Vas deferens, Spermatic cord, Inguinal canal, Seminal vesicles.

When interviewing a patient who does not speak English, the examiner should:

Use a qualified medical interpreter who is culturally literate.

During an interview, a patient denies having any anxiety. The patient frequently changes position in the chair, and has little eye contact with the interviewer. The interviewer should:

Use confrontation to bring the discrepancy between verbal and nonverbal behavior to the patient's attention.

4. List the steps in the procedure of obtaining a Cervical scrape using spatula specimen:

Use cytobrush or insert the bifid end of the Ayre spatula into the vagina with the more pointed bump in the cervical opening, OS. -Rotate it 360-720 firmly. -Dip into liquid vial or spread specimen from both sides of spatula on glass slide. --Use single stroke, not back and forth, to thin out specimen. --Important for the adolescent whose endocervical cells have not yet migrated into the endocervical canal-

Your are assessing short-term cognitive function. Which assessment shows assessment shows the ability to lay down new memories?

Use of Four Unrelated Word Test

4. Describe the procedure for auscultation of bowl sounds.

Use the diaphragm of the stethoscope to press lightly on the skin. Listen to all four quadrants starting in the RLQ at the ileocecal valve. Note characteristic and frequency of bowel sounds. Listen for 5 minutes to determine that bowel sounds are completely absent. Vascular sounds can be heard using the bell to listen for bruits/ARIF, aorta, renal, iliac, and femoral.

Valve auscultation

Use the diaphram. Aortic area - second right interspace, Pulmonic area - second left interspace, Erbs point - left lower sternum border, 3rd space, Tricuspid area - left lower sternum border, 4th space, Mitral area - left midclavicler 5th interspace APEs To Man

A patient is requesting pain medication and expresses a pain level of 9/10; however, the patient is up and smiling. How should you proceed? A. Complete a full physical examination. B.Check to see when the patient last received the pain medication and administer it if the time is appropriate. C. Call the provider and suggest a substance abuse consult. D. Use therapeutic communication Technique to determine the patient's current prescription drug use.

Use therapeutic communication Technique to determine the patient's current prescription drug use.

Follow-Up Database

Used to monitor progress of short-term and chronic health problems used in all hospital settings.

Ad hoc interpreter

Using a patient's family member, friend, or child as an interpreter for a limited LEP

Jargon

Using medical vocabulary with patient in an exclusionary and paternalistic way

Two types of human hair:

VELLUS HAIR: fine, faint, covers most of the body except - palms, soles, dorsa of distal parts of fingers, umbilicus, glans penis, inner labia. TERMINAL HAIR: darker, thicker hair on scalp, eyebrows, axillae, pubic area, and face and chest in a male

Malnutrition Screening Tool

Validated for use in adult acute-care patients

Expert Nurse

Vaults over the first steps to treat the patient much faster.

4. Define the term capacitance vessels, and explain its significance.

Veins have a larger diameter and are more distensible; they can expand and hold more blood when blood volume increases. This is a compensatory mechanism to reduce stress on the heart; this ability to stretch, veins are called capacitance vessels.

inspection of a patient's abdomen findings

Venous pattern, peristaltic waves, and abdominal contour

11. Define venous pressure and jugular venous pulse.

Venous pressure - the pressure exerted on the walls of the veins by the circulating blood Jugular Venous Pulse - The pulse in the right internal jugular vein at the root of the neck; pressure of right atrium

Preload

Venous return of blood that builds as the heart fills up during diastole.

succussion splash bdominal sounds

Very loud splash heard over the upper abdomen when an infant is rocked side to side; indicates increased air and fluid in the stomach as seen with pyloric obstruction or large hiatus hernia.

9a. Fill in the labels indicated in the following illustrations (vulva).

Vestibule: Urethral meatus, Opening of Skene gland, Hymen, Opening of Bartholin gland, Perineum, Anus, Frenulum, Vaginal orifice, Labia minora, Labia majora, Clitoris, Mons pubis,

Empathy

Viewing the world from the other person's inner frame of reference while remaining yourself; Recognizing and accepting the other person's feelings without criticism

Questions asked in a Health Assessment (set B)

What do you know about this condition? Does it interfere with your daily life? When did it first start (onset)? What does it feel like? Do you feel safe at home? Do you have enough money for food? Is there anything else you are concerned about?

Subjective Data

What the person says about themselves during history taking.

Competency Nurse

Where the nurse sees actions in the context of arching goals or plans for patients.

14. Related the use of the wrong size blood pressure cuff to the possible findings that might be obtained.

Wider cuffs deliver false low reading and narrow cuffs give a false high reading (because it takes extra pressure to compress the artery.

13. To test for accommodation, the person focuses on a distant object then shifts the gaze to a near object about 6 inches away.

With distance vision, you expect the pupils to dilate. With close vision, you expect the pupils to constrict and the axes of the eyes to converge.

While examining the broken arm of a 4-year-old boy; select the appropriate assessment tool to evaluate his pain status. A. 0 to 10 numeric rating scale B. Wong-Baker Scale C. Simple descriptor scale D. 0 to 5 numeric rating scale

Wong-Baker Scale

Cerumen

Yellow waxy material that lubricates and protects the ear canal: Often sticky and brown or honey colored in Caucasians and blacks; often dry, gray and flaky in American Indians and Asians.

Lid lag

abnormal white rim of sclera visible between the upper eyelid and the iris when a person moves the eyes downward. Occurs with hyperthyroidism.

15. During percussion, which sound would you expect to predominate over normal lung tissue?

You will expect resonance.

A patient has recently received Health insurance and would like to know how often he should visit the provider. How do you respond

Your visit may vary, depending on level of wellness

Older adults may respond best to

a Descriptor Scale.

coarctation of aorta

a congenital heart defect of a severe narrowing of the descending aorta

friction rub

a course, grating, adventitious lung sound heard when the pleurae are inflamed

Diet history

a detailed record of dietary intake obtainable from 24-hour recalls, food frequency questionnaires, food diaries, and similar sources

Wellness

a dynamic process and view of health toward optimal functioning

chronic obstructive pulmonary disease, COPD

a functional category of abnormal respiratory conditions characterized by airflow obstruction: emphysema, chronic bronchitis.

Pedigree or genogram

a graphic family tree in at least 3 generations such as parents, grandparents, and siblings.

glaucoma

a group of eye diseases characterized by increased ocular pressure Three-times more common in blacks. Comes with night blindness and scotoma A.k.a. a blind spot. Never dilate due to risk of acute episode.

Nursing process

a method of collection and analyzing clinical information with the following components: (1) assessment, (2) diagnosis, (3) outcome identification, (4) planning, (5) implementation, and (6) evaluation

fremitus

a palpable vibration from teh spoken voice felt over the chest wall

Spirituality

a person's personal effort to find purpose and meaning in life

Nutrition screening

a process used to identify individuals at nutritional risk or with nutritional problems

Anorexia Nervosa -

a psychological disorder characterized by somatic delusions that you are too

When reading a medical record, you see the following notation: Patient states "I have had a cold for about a week, and now I am having difficulty breathing." This is an example of:

a reason for seeking care

When taking a health history for a child, what information, in addition to that for an adult, is usually obtained?

a review of the immunizations recieved

Herpes genitalis

a sexually transmitted infection characterized by clusters of small painful vesicles, caused by a virus.

Stupor -

a state of near-unconsciousness or insensibility

Coma -

a state of unconsciousness in which a person: cannot be awakened; fails to respond normally to painful stimuli, light, or sound; lacks a normal wake-sleep cycle; and does not initiate voluntary actions.

Evidence-based practice

a systematic approach emphasizing the best research evidence, the clinician's experience, patient preferences and values, physical examination, and assessment

Macrocephalic

abnormally large head

Microcephalic

abnormally small head

scaphoid

abnormally sunken abdominal wall, as with malnutrition or underweight

15. Explain the significance of phase I, phase IV, and phase V Korotkoff sounds during blood pressure movement. Phase IV,

abrupt muffling. Sound mutes to a low pitched, cushioned murmur, blowing quality. Artery no longer closes in any part of cardiac cycle. Chang in quality, not intensity

hepatomegaly

abnormal enlargement of liver

splenomegaly

abnormal enlargement of spleen

It is normal to palpate a few lymph nodes in the neck of a healthy person what are the characteristics of these nodes? a. mobile, soft, non tender b. large clumped tender c. matted. fixed, tender, hard d. matted fixed non tender

a. mobile, soft, non tender

Normal cervical lymph nodes are: a. smaller than 1 cm b. warm palpation. c fixed d. firm

a. smaller than 1 cm

Cephalhematoma is associated with: a. subperiosteal hemorrhage b. increased intracranial pressure c. down syndrome d. mental retardation

a. subperiosteal hemorrhage

Bells palsy is characterized by a. unilateral paralysis of half the face. b. bulging eyeballs c. a face that appears mask like d. a puffy, edematous face

a. unilateral paralysis of half the face.

15a. Fill in the labels indicated in the following arteries and name the pulse sites.

abdominal aorta common iliac artery external iliac artery femoral pulse site popliteal artery popliteal pulse site anterior tibial artery posterior tibial artery dorsalis pedis artery dorsal arch dorsal pedis pulse site posterior tibial pulse site

Dysmenorrhea

abdominal cramping and pain associated with menstruation

Endometriosis

aberrant growths of endometrial tissue scattered throughout pelvis

point localization

ability of the person to discriminate exactly where on the body the skin has been touched

graphesthesia

ability to "read" a number by having it traced on the skin

orthopnea

ability to breathe easily only in an upright position

Two-point discrimination

ability to distinguish the separation of two simultaneous pinpricks on the skin

memory -

ability to lay down and store experiences and perceptions for later recall

stereognosis

ability to recognize objects by feeling their form, size and weight while the eyes are closed

ascites

abnormal accumulation of serous fluid within the peritoneal cavity, associated with heart failure, cirrhosis, cancer, or portal hypertension.

orchitis

acute inflammation of the testis, usually associated with the mumps

pain can also be classified by its duration

acute pain and persistent (or chronic pain)

accommodation

adaptation of the eye for near vision by increasing the curvature of the lens

6 to 12 cm

adult normal range of liver span in the right midclavicular line

numeric scale is used for

adults and older children.

During an initial interview, the examiner says, "Mr. J., tell me what you do when your headaches occur." With this question, the examiner is seeking information about:

aggravating or relieving factors

Subcutaneous layer

aids protection by cushioning. Adipose tissue.

Aortic regurgitation

aka aortic insufficiency is incompetent aortic valve that allows backward flow of blood into the left ventricle during diastole.

Pinna

aka auricle, is the outer ear. Has flexible cartilage and skin in the shape of a funnel to guide sound waves into its opening.

Describe tonic neck reflex in infant

an Infant at two weeks can turn head from side to side

asthma

an abnormal respiratory condition associated with allergic hypersensitivity to certain inhaled allergens, characterized by inflammation, brochospasm, wheezing, and dyspnea.

atelectasis

an abnormal respiratory condition characterized by collapsed, shrunken, deflated sections of alveoli.

List at least 5 names for various fold healers and the culture they represent

asian folk healers herbalist, acupuncture, bone setter hispanic folk healers curradero, espiritualista, yerbo, partera, sabedor african american folk healers hougan, old lady, spiritualist american indian folk healers shaman, medicin woman amish folk healers braucher, powowwing african american culture bound syndrome thin blood, high blood, low blood hispanic culture bound syndrome mal ojo "evil eye" american indian culture bound syndrome ghost (sense of danger) european american culture bound syndrome hysteria, involutional paraphrenia

numeric rating scales

ask patient to choose a # that rates level of pain for each painful site, w/ 0 being no pain and highest anchor 10 meaning worst pain ever experienced

short form McGill pain questionnaire

ask patient to rank a list of descriptions in terms of their intensity and to give an overall intensity rating to their pain

brief pain inventory

ask patients to rate the pain within the past 24 hours using 0-10 scale with respect to its impact on areas such as mood, walking, ability, and sleep

Ototoxic drugs

aspirin, aminoglycosides: -gentamicin, -tobramycin -amikacin, ethacrynic acid, furosemide, Indomethacin, Naproxen, quinine, vancomycin.

Use an overall pain assessment tool, such as the Initial Pain Assessment or the Brief Pain Inventory, to

assess chronic pain or problematic acute pain.

Nutritional monitoring

assessment of dietary or nutritional status at intermittent times with the aim of detecting changes in the dietary or nutritional status of a population

Eczema

atopic dermatitis

striae

atrophic pink, purple, or white linear streaks on the breasts, associated with pregnancy, excessive weight gain, or rapid growth during adolescence

perceptions -

awareness of objects through any of the five senses

orientation -

awareness of the objective world in relation to the self

base of the heart

broader area of heart's outline located at the 3rd right and left intercostal space

abdomen Auscultation may reveal

bruits of the aortic, renal, iliac, and femoral arteries.

The examiner suspects an infant's head is of abnormal size and can use which procedure to verify these findings? a. palpation b. measuring tape c. observing for symmetry of facial features d. noting absence of the tonic neck reflex

b. measuring tape

nystagmus

back and forth oscillation of the eyes

Alopecia

baldness - hair loss

Epidermis layer

basal cell layer. Uniformly thin. Stratum corneum. The thin, tough, outer layer of the skin

pulsus paradoxus

beats have weaker amplitude with respiratory inspiration, stronger with expiration

Discuss the impact that racial and cultural diversity of individuals has on the US health care system

because the US is perceived as the center of advanced health care and technology, foreign nationals who return home after treatment have been inflating the census reports at many US hospitals, with more popular types of US interventions being CV, neurologic, and cancer treatment cultural conflicts between nurses and patients from diverse backgrounds are related to diff. time and relationship perceptions -ex) in some cultures, the past may influence health practices -when making decisions about health, patients may rely on relationships with others, and their behavior may depend on the opinion of others

Thelarche

beginning of prepubertal breast development

Auscultating the abdomen

begins in the right lower quadrant, RLQ because bowel sounds are always normally present

consciousness -

being aware of ones own existence, feelings, and thoughts and being aware of the environment

flexion

bending a limb at a joint

fibroadenoma

benign breast mass

drusen

benign deposits on the ocular fundus that show as round yellow dots and occur commonly with aging

Lipoma

benign fatty tumor

Anasarca

bilateral edema, or edema that is generalized over the whole body

describe three major theories on the ways people view the cause of illness

biomedical (bacteria and viruses are responsible); naturalistic (laws of nature create imbalances); magicoreligious (evil eye - supernatural forces)

athetosis

bizarre, slow, twisting, writhing movement, resembling a snake or worm

Things to know for the test:

blood flow: pattern Pericardium Myocardium SA node What's an EKG. Cardiac output - stroke volume X heart rate Abnormal Jugular Venous Pressure- at 45 degrees JVP tells us the right heart is straining. Child, baby, adult heart position Pattern for listening to heart Palpating carotid matches to apical pulse, apex S1 Murmur - whooshing sound not lub dub Listen for the carotid - breath held, 3 positions with bell Black folks and Higher BP than other populations Active filling phase - presystole Radial and Ulna pulses should be equal bilaterally 2+ normal Popliteal pulse behind the knee Femoral pulse in groin Dorsalis pedis pulse top of foot Epitrochlear - abnormally if things felt there

Melena

blood in the stool

bruit

blowing, swooshing sound heard through a stethoscope when an artery is partially occluded

bruit bowel sounds

blowing, swooshing sound heard through a stethoscope when an artery is partially occluded

Acrocyanosis

bluish color around the lips, hands, fingernails, feet, and toenails

Chadwick sign

bluish discoloration of cervix that occurs normally in pregnancy at 6 to 8 weeks' gestation

Cyanosis

bluish, dusky blue, mottled color to skin or mucous membranes, that signifies decreased perfusion - increased amount of unoxygenated hemoglobin. Dark skinned normally have bluish lips.

olecranon process

bony projection of the ulna at the elbow

Evidence

collected body of data from lab tests and medical history

spermatic cord

collection of vas deferens, blood vessels, lymphatics, and nerves that ascend along the testis and through the inguinal canal into the abdomen.

1. What is the most common site of nosebleeds?

c. Kiessellbach plexus

Upon examination, the fontanels should feel: a. tense or bulging b. depressed of sunken c. firm slightly concave and well defined d. Pulsating

c. firm slightly concave and well defined

A throbbing unilateral pain associated with nausea vomiting and photophobia is characteristic of a. cluster headache b. subarachnoid hemorrhage c. migraine headache d. tension headache

c. migraine headache

aortic stenosis

calcification of aortic valve cusps that restricts forward flow of blood during systole

pulmonic stenosis

calcification of pulmonic valve that restricts forward flow of blood during systole

mitral stenosis

calcified mitral valve impedes forward flow of blood into left ventricle during diastole

child re 2 yrs of age

can report pain and point to its location

Organic Brain Disorder -

caused by brain disease of known specific organic cause [e.g., delirium, dementia, alcohol and drug intoxication, and withdrawal]

nucleus pulposus

center of the intervertebral disc.

Polyp

cervical polyp is bright red, soft, predunculated growth emerging from os

apnea

cessation of breathing

menopause

cessation of menses, usually occurring around 48-51 yrs of age

leukoplakia

chalky white, thick, raised patch on sides of tongue; precancerous

rheumatoid arthritis

chronic systemic inflammatory disease of joints and surrounding connective tissue.

Annular

circular shape to skin lesion

15. Explain the significance of phase I, phase IV, and phase V Korotkoff sounds during blood pressure movement. Phase I-Soft,

clear tapping, increasing in intensity. Systolic pressure-as the cuff pressure lowers to reach intrluminal systolic pressure, the artery opens, and blood first sputs into the brachial artery. Blood is at a very high velocity because of small opening of artery and large pressure difference across opening. This creates turbulent flow, which is audible

chonanal artesia

closure of nasal cavity due to congenital septum between nasal cavity and pharynx.

talipes equinovarus

clubfoot, congenital deformity of the foot where it is plantar flexed and inverted.

Dermis layer

collagen. elastic tissue

purpose of a complete health history

collect subjective data and combine it with objective data from physical examination and diagnostic tests

silvery white

color that indicates long-standing striae

Marasmic kwashiorkor

combination of chronic energy deficit and chronic or acute protein deficiency

Sarcopenic obesity

combined loss of muscle mass with weight gain occurring in old age

Scale

compact desiccated flakes of skin from shedding of dead skin cells

paralytic ileus

complete absence of peristaltic movement that may follow abdominal surgery or complete bowel obstruction

attention -

concentration, ability to focus on one specific thing

Nullipara

condition of first pregnancy

multipara

condition of having 2 or more pregnancies

epispadias

congenital defect where the urethra opens on the dorsal/upper side of the penis instead of the tip

hypospadias

congenital defect where the urethra opens on the ventral/under side of the penis instead of the tip

pyloric stenosis

congenital narrowing of pyloric sphinctor, forming outflow obstruction of stomach

perception

conscious awareness of painful sensation

miosis

constricted pupils; controlled by CN3

spasticity

continuous resistance to stretching by a muscle due to abnormally increased tension, with increased deep tendon reflexes

lactiferous

conveying milk

crepitus

course, crackling sensation palpable over the skin when air abnormally escapes from the lung and enters the subcutaneous tissue

A-V crossing

crossing paths of an artery and vein in the ocular fundus

bell of the stethoscope

cup-shaped end-piece used for soft, low-pitched heart sounds; best for abnormal heart sounds, murmurs.

Endogenous Obesity -

cushing syndrome: is due to excess cortisol secretion by adrenals. Endogenous or exogenous ACTH stimulates the adrenals to release cortisol. It is characterized by central trunk and cervical obesity

hydrocele

cystic fluid in tunica vaginalis surrounding the testis

ligament

fibrous band running directly from one bone to another bone that strengthens the joint.

retraction

dimple or pucker on the skin

dorsal

directed toward or located on the surface

If the thyroid gland is enlarged bilaterally, which of the following maneuvers is appropriate? a. check for a deviation of the trachea b. listen for a bruit over the carotid arteries c. listen for a murmur over aortic area. d. listen for a bruit over the thyroid area

d.Listen for bruit over the thyroid lobes

extinction

disappearance of conditioned response

areola

darkened area surrounding nipple

caries

decay in the teeth

presbyopia

decrease in power of accommodation that occurs with aging; the lens loses elasticity becoming glasslike, decreasing the ability of the lens to change shape to accommodate for near vision.

Constipation

decrease in stool frequency, with difficult passing of very hard, dry stools.

hypoxemia

decreased level of oxygen in the blood

paralysis

decreased or loss of motor function due to problem with motor nerve or muscle fibers

aneurysm

defect of sac formed by dilation in artery wall due to atherosclerosis, trauma, or congenital defect

aneurysm

defect or sac formed by dilation in artery wall due to atherosclerosis, trauma, or congential defect.

ischemia

deficiency of arterial blood to a body part due to constriction of obstruction of a blood vessel

the major factor contributing to the need for cultural care nursing is:

demographic change

umbilicus

depression on the abdomen marking site of entry of umbilical cord

Mini Nutritional Assessment (MNA)

designed and validated for use in older adults long-term care and community settings

What two sections of the child's health history become separate sections because of their importance to current health status.

developmental and nutritional history

dyspnea

difficult, labored breathing

dysphagia

difficulty swallowing

mydriasis

dilated pupils

varicocele

dilated tortuous varicose veins in the spermatic cord

varicose veins

dilated tortuous veins with incompetent valves

Functional assessment measures how a person manages day-to-day activities. The impact of a disease on the ADL's in older adults is referred to as:

disease burden

3. Differentiate the following:

dislocation -Displacement of a body part, especially the temporary displacement of a bone from its normal position. subluxation-Incomplete or partial dislocation of a bone in a joint. Contracture -abnormal shortening of muscle tissue, rendering the muscle highly resistant to passive stretching. Ankylosis - immobility, consolidation, and fixation of a joint because of disease, injury, or surgery; most often due to chronic rheumatoid arthritis.

Bloody show

dislodging of thick cervical mucus plug at the end of pregnancy, which is a sign of the start of labor.

Skinfold thickness

double fold of skin and underlying subcutaneous tissue that is measured with skinfold calipers at various body sites

diplopia

double vision

7. Describe your own preparation as you encounter the patient for examination: Your own dress, Your demeanor, Safety/universal precautions, Sequence of examination steps, Instructions to patient.

dress professionally, have an open/friendly demeanor, Safety/universal precautions, hand hygiene before/after patient encounters after contact with body fluids after contact w/contaminated equipment after removing gloves. Sequence of examination steps, vital signs change to exam gown hand hygiene explain each step of exam move slow encourage questions summarize findings Instructions to patient explain the procedure what you'd like them to do what they may feel

ptosis

dropping of the upper eyelid over the iris and possibly covering the pupil

Somnolent -

drowsy or sleepy acting

cretitation

dry cracking sound or sensation due to grating of the ends of damaged bone.

vas deferens

duct carrying sperm from the epididymis through the abdomen and then into the urethra

indicative of splenic enlargement

dull percussion note forward of the left midaxillary line

Obtunded -

dull the sensitivity, blunt, deaden, sleeps most of the time and are difficult to arouse

cyanosis

dusky blue mottling of the skin and mucous membrane due to excessive amount of reduced hemoglobin in the blood

Bulla

elevated cavity containing free fluid larger than 1cm diameter

Vesicle

elevated cavity containing free fluid up to 1 cm diameter

Pustule

elevated cavity containing thick, turbid fluid

Oxycontin overdose

emergency database

bursa

enclosed sac filled with viscous fluid located in joint areas of potential friction.

Lymphadenoapathy

enlargement of the lymph nodes due to infection, allergy, or neoplasm

absence of bowel sounds

established after listening for 5 full minutes

use a pain rating scale to

evaluate pain severity

Past health

events illnesses, injuries, hospitalizations, allergies, & current medications

Android obesity

excess body fat that is placed predominantly (upper body, mostly in the abdomen)

Hirsutism

excess body hair

Obesity

excessive consumption of protein and energy, resulting in a gradual body wasting and increased susceptibility to infection

Steatorrhea

excessive fat in the stool, as in gastrointestinal malabsorption of fat

Euphoria

excessive well-being

Menorrhagia

excessively heavy menstrual flow

tail of Spence

extension of breast tissue into the axilla

range of motion, ROM

extent of movement of a joint

Vulva

external genitalia of the female

A genogram is useful in showing information concisely. It is used specifically for:

family history

arcus senilis

gray-white arc or circle around the limbus of the iris that is common with aging

Pediculosis capitis

head lice

Torticollis

head tilt due to shortening or spasm of one sternomastoid muscle

The concept of health and healing has evolved in recent years. Which is the best description of health? a. health is the absence of disease b. health is the dynamic process toward optimal functioning c. health depend on an interaction of mind, body, and spirt within the environment

health depends on an interaction of mind, body, and sprit within the environment

pyrosis

heartburn; burning sensation in upper abdomen due to reflux of gastric acic

eversion

moving the sole of the foot outward at the ankle.

lordosis

inward or concave curve of the lumbar spine

pulsus bigeminus

irregular rhythm; every other beat is premature; premature beats have weakened amphlitude

Delirium -

is an acute confusional state, potentially preventable in hospitalized persons. Characterized by disorientation, disordered thinking and perceptions, illusions and hallucinations, defective memory, agitation, inattention.

Define the term Mental Disorder.-

is apparent when a person's response is much greater than the expected reaction to a traumatic life event. It is a clinically significant behavioral, emotional, or cognitive syndrome that is associated with significant distress (a painful symptom) or disability (impaired functioning) involving social, occupational, or key activities.

Pruritus

itching or burning sensation in the skin

Pruritus

itching. The most common skin symptom. Occurs with dry skin, aging, drug reactions, allergy, obstructive jaundice, uremia, lice

Basal ganglia

large bands of gray matter buried deep within the 2 cerebral hemispheres that form the subcortical associated motor system. They help to initiate and coordinate movement and control automatic associated movements of the body. Ex. Arm swing alternating with legs while walking.

Cafe' au lait

large round or oval patch of light brown usually present at birth

hallux valgus

lateral or outward deviation of the great toe.

list the four basic characteristics of culture

learned, shared, adapted, dynamic

mitral valve

left atrioventricular valve separating the left atrium and ventricle

Diagnostic reasoning

method of collecting and analyzing clinical information: 1. attending to available cues 2. formulating diagnostic hypotheses 3. gathering data relative to the hypotheses 4. evaluating each hypothesis with the collected data 5. arriving at a final diagnosis

rectus abdominis muscles

midline abdominal muscles extending from rib cage to pubic bone

diastasis recti

midline longitudinal ridge in the abdomen, a separation of abdominal rectus muscles

linea alba

midline tendinous seam joining the abdominal muscles

cilia

millions of hairlike cells lining the trachobronchial tree

supernumerary nipple

minute extra nipple along the embryonic milk line

mitral regurgitation

mitral insufficiency; incompetent mitral valve allows regurgitation of blood back into left atrium during systole

candidiasis

moniliasis; white, cheesy, curdlike patch on buccal mucosa due to superficial fungal infection

Midbrain

most anterior part of brainstem that still has the basic tubular structure of the spinal cord; merges into the thalamus and hypothalamus; contains motor neurons and tracts.

lower motor neuron

motor neuron in the peripheral nervous system with its nerve fiber extending out to the muscle and only its cell body in the central nervous system

abduction

moving a body part away from an axis or the medial line.

Retraction

moving a body part backward and parallel to the ground

protraction

moving a body part forward and parallel to the ground

adduction

moving a body part toward the center or toward the medial line.

curcumduction

moving the arm in a circle around the shoulder.

inversion

moving the sole of the foot inward at the ankle.

Fibroid

myoma; hard, painless nodule in uterine wall that cause uterine enlargement

epigastrium

name of abdominal region between the costal margins

suprapubic

name of abdominal region just superior to pubic bone

myopia

nearsided; refractive error in which near vision is better than far vision

Nociceptive pain develops when

nerve fibers in the peripheral and central nervous systems are functioning and intact. It starts outside the nervous system and results from actual or potential tissue damage. Nociception occurs in four phases: transduction, transmission, perception, and modulation. This pain typically is predictable and time limited based on the extent of the injury.

Upper motor neuron

nerve located entirely within the central nervous system

sciatica

nerve pain along the course of the sciatic nerve that travels down from the back or thigh through the leg and into the foot.

inverted nipples

nipples that are depressed or invaginated; sunken in

peyronie disease

nontender hard plaques on the surface of the penis, associated with painful penis bending during erection.

Epulis

nontender, fibrous nodule of the gum

Describe physical examination findings that might indicate pain.

nonverbal behaviors of pain trachycardia hypertension tachypnea

6. Explain the physiologic mechanism for normal splitting of S2, in the pulmonic valve area.

normal variation in S2 heard as two separate components during inspiration

physiologic splitting

normal variation in S2 heard as two separate components during inspiration

B. Approximately 1x2 cm

normal-sized impulse for palpating the apical impulse.

epistaxis

nosebleed, usually from anterior septum

The physical examination can reveal objective data that support subjective data.

o Assess the patient's joints, muscles, skin, and abdomen to detect injuries or other signs of painful disorders. o When a person cannot verbally communicate the pain, look for nonverbal behaviors of pain, such as guarding, grimacing, moaning, agitation, restlessness, stillness, diaphoresis, or vital sign changes. Keep in mind that individuals react to painful stimuli very differently. o Also assess for physiologic changes in various body systems. These changes can result from poorly controlled acute or chronic pain.

recall questions for an initial pain assessment

o Do you have pain, discomfort, or soreness? o Where is your pain? o When did it start? o What does your pain feel like? o How much pain do you have right now? o What makes you pain better or worse? o How does pain limit your function or activities? o How do you usually react when you are in pain? o What does this pain mean to you?

venous ulcer risk factor

obesity

Blood in urine

objective

Unconscious

objective

weight gain

objective

incident pain

occurs predictably after specific movements

1st heart sound, S1

occurs with closure of the atrioventricular valves, AV, signaling the beginning of systole; heard at the apex or bottom of the heart; point of maximal impact for the apical pulse.

2nd heart sounds, S2

occurs with closure of the semilunar valves SL, aortic and pulmonic; signals the end of systole; best heard at the base or top of the heart. Paradoxical Split can be heard here, lub, dub-dub; most commonly in kids when they breathe in about every 4th heartbeat.

Telangiectasia

skin lesion due to permanently enlarged and dilated blood vessels that are visible

pulse

pressure wave created by each heartbeat, palpable at body sites where the artery lies close to the skin and over a bone

Kwashiorkor

primarily a protein deficiency characterized by edema, growth failure, and muscle wasting

nociception

process whereby noxious stimuli are perceived as pain; central and peripheral nervous systems are intact

Diaphoresis

profuse perspiration

Rectocele

prolapse of rectum and its vaginal mucosa into vagina with straining or standing

opisthotonos

prolonged arching of back, with head and heels bent backward, and meningeal irritation

mood -

prolonged display of a person's feelings

priapism

prolonged, painful erection of penis without sexual desire

17. Contrast pinguecula with pterygium.

pterygium: is a triangular growth on the sclera pinguecula: yellowish patch or bump on the conjunctiva that results in a deposit of protein, fat and/or calcium; can often lead to a pterygium.

pulmonic regurgitation

pulmonic insufficiency; backflow of blood through incompetent pulmonic valve into the right ventricle

argyll Robertson pupil

pupil does not react to light; does constrict with accommodation

Closed questions

questions that ask for specific information and elicit a short, one- or two- word answer, a "yes" or "no" or a forced choice

2. Define the characteristics of the following percussion notes.

resonance - amplitude - medium-loud, pitch - low, quality - clear/hollow, duration - moderate duration, Ex - normal lung tissue sound; hyperresonance - amplitude - louder, pitch - lower quality - booming duration - longer Ex - normal child lung sound or abnormal, emphysema like adult lung sound tympany - amplitude -loud, pitch - high, quality - musical, drumlike, duration - sustained longest, Ex - over air filled tissue (stomach, intestines), dull - amplitude - soft, pitch - high, quality - muffled thud duration - short, Ex - dense organ as liver or spleen, flat - amplitude - very soft pitch - high, quality - dead stop of sound, duration - very short, Ex - no air is present (thy muscle, bone, tumor)

Marasmus

results from energy and protein deficiency, manifesting with significant loss of body weight, skeletal muscle, and adipose tissue mass, but with serum protein concentrations relatively intact

spermatocele

retention cyst in epididymis filled with milky fluid that contains sperm

tricuspid valve

right atrioventricular valve separating the right atrium and ventrcle

pulmonic valve

right semilunar valve separating the right ventricle and pulmonary artery

cor pulmonale

right ventricular hypertrophy and heart failure due to pulmonary hypertension.

Tinea pedis

ringworm of the foot

peritoneal friction rub

rough grating sound heard through the stethoscope over the site of peritoneal inflammation

papillae

rough, bumpy elevations on dorsal surface of tongue

Normocephalic

round symmetric skull that is appropriately related to body size

ganglion

round, cystic, non-tender nodule overlying a tendon sheath or joint capsule, usually on dorsum of wrist.

Depression

sad, gloomy, dejected

Plaque

skin lesion in which papules coalesce or come together

Erosion

scooped out, shallow depression in skin

montgomery glands

sebaceous glands in the areola that secrete protective lipid during lactation; also called tubercles of Montgomery

transmission

second phase of nociception whereby the pain impulse moves from the spinal cord to brain

Excoriation

self-inflicted abrasion on skin due to scratching

rationale and components of cultural care nursing (SAC)

sensitive, appropriate, and competent

proprioception

sensory information concerning body movements and position of the body in space

intraductal papilloma

serosanguineous nipple discharge

Valves of Houston

set of three semilunar transverse folds that cross half the circumference of the rectal lumen

Gonorrhea

sexually transmitted infection characterized by purulent vaginal discharge or may have no symptoms

acute pain

short term, self limiting, often predictable trajectory; stops after injury heals

13. The stethoscope bell

should be pressed lightly against the skin so that the bell does not act as diaphragm

Critical thinking

simultaneously problem solving while self improving one's own thinking ability

synapses

sites of contact between two neurons

Hemangioma

skin lesion due to benign proliferation of blood vessels in the dermis

Items included in a functional assessment

spiritual resources, activity/ exercise, sleep/rest, nutrition/ elimination, coping/ stress management, self esteem, self concept, interpersonal relationships, personal habits, occupational health, environmental hazard, and alcohol or illicit street drugs.

strabismus

squint, crossed eye - disparity of the eye axes To avoid double vision, the brain begins to suppress data from the weak eye causing vision to deteriorate from disuse.

palilledema

stasis of blood flow out of the ocular fundus; sign of increased intracranial pressure

coma

state of profound unconsciousness from which person cannot be aroused.

nuchal rigidity

stiffness in cervical neck area

Striae gravidarum

stretch marks. May develop in the skin of the abdomen, breasts, thighs, upper arms

percussion

striking over the chest wall with short, sharp blows of the fingers to determine the size and density of the underlying organ

tendon

strong fibrous cord that attaches a skeletal muscle to a bone.

epididymis

structures composed of coiled ducts located over the superior and posterior surface of the testes, which store sperm

hordeolum

stye - red, painful pustule that is a localized infection of hair follicle at eyelid margin

Hair-line linear markings:

white linear markings that normally are visible through the and on the pink nail bed.

Leukorrhea

whitish or yellowish discharge from vaginal orifice

Peau d'orange

with edema, hair follicles are more prominent, giving a pigskin or orange-peel look

Fear

worried about known external danger

torticollis

wryneck; contraction of the cervical neck muscles, producing torsion of the neck.

Carotenemia

yellow-orange color in light-skinned persons from large amounts of foods containing carotene

Physiologic jaundice

yellowing of skin, sclera, and mucous membranes due to increased numbers of red blood cells hemolyzed following birth

yin/yang theory of health and illness

yin=cold; yang=hot (hot illness eat cold foods and cold illness eat hot food)

11. When auscultating the heart,

your first step is to Identify S1 and S2

13. Fill in the description of the grading scale for pitting edema: 1+, 2+, 3+, 4+.

• 1+ Mild pitting, slight indentation, no perceptible swelling of the leg • 2+ Moderate pitting, indentation, subsides rapidly • 3+ Deep pitting, indentation remains for a short time, leg looks swollen • 4+ Very deep pitting, indentation lasts a long time, leg is very swollen

7. When performing a functional assessment for an older adult, state the common adaptations that the aging person makes when attempting these maneuvers:

• Walking: shuffling pattern and swaying • Climbing up stairs: person holds on to handrail • Walking down stairs: person holds on to handrail with both hands • Picking up object from floor: often bends at waist instead of knees • Rising up from sitting in chair: uses arms to push off chair arms • Rising up from lying in bed: may roll to one side, push arm to lift up torso


संबंधित स्टडी सेट्स

MGMT4010 - Strategic Management Exam 1

View Set

Prep Agent- Transfer of Property

View Set

Chapter 21: Nonproteobacterial Gram-negative Bacteria

View Set

Foodservice Organizations Chapter 3

View Set